Pharmacology Krok 1 Questions, Dentistry

Download as pdf or txt
Download as pdf or txt
You are on page 1of 92

GENERAL PHARMACOLOGY (46) After anaesthetic application during tooth

extraction the patient developed marked soft tissue A patient was prescribed a drug with apparent
Anaphylactic reaction edema of the upper and lower jaw, skin rash on the lipophilic properties. What is the main mechanism
A 50-year-old patient has been referred for face, reddening, and itching. What pathological of its absorption?
treatment of neck lymphadenitis. His individual process results insuchreaction to the anaesthetic? A. Passive diffusion
penicillin sensitivity was tested. In 30 seconds full- A. Allergy B. Filtration
body fever raised in the patient and his arterial B. Toxic action of a drug C. Active transport
blood pressure dropped to 0 mm Hg, followed by C. Inflammation D. Pinocytosis
cardiac arrest. Resuscitation was unsuccessful. D. Circulatory deficiency E. Binding with transport proteins
Autopsy revealed acute venous hyperemia of E. Disturbed lymph drainage
viscera. Histological study revealed mast cells For pain relief a patient has taken simultaneously a
(tissue basocytes) degranulation in the skin (at the A patient has been administered conduction tablet of paracetamol and a tablet of diclofenac
area of injections), myocardium and lungs. What anesthesia with novocaine in preparation for tooth sodium. What type of drug interaction did the
kind of hypersensitivity reaction occurred in extraction. After the anesthesia administration the patient use for self-treatment?
patient? patient developed swelling and hyperemy around A. Additive synergism
A. Anaphylactic the injection site, skin itch, general fatigue, motor B. Non-competitive antagonism
B. Delayed-type hypersensitivity agitation. Name the developed complication: C. Potentiated synergism
C. Complement-mediated cytotoxic A. Allergy D. Synergic antagonism
D. Immune complex-mediated B. Tachyphylaxis E. Competitive antagonism
E. – C. Idiosyncrasy
D. Drug dependence While under barbituric anaesthesia a 65-year-old
For the purpose of anaesthetization a patient got E. Inflammation male patient developed respiratory inhibition.
injection of local anesthetic. A few minutes later Anesthesiologist made him a 10 ml intravenous
the patient got dyspnea and tachycardia; he lost A patient with chronic hepatitis complains of injection of 0,5% bemegride solution. The patient’s
consciousness. What type of shock is it? increased sensitivity to barbiturates that previously condition got better, the pulmonary ventilation
A. Anaphylactic induced no symptoms of intoxication. What volume increased. What phenomenon underlies the
B. Cardiogenic hepatic function is disrupted and primarily interaction of these medications?
C. Haemorrhagic responsible for such reaction in this patient? A. Direct antagonism
D. Traumatic A. Metabolic B. Indirect antagonism
E. Burn B. Hemodynamic C. Unilateral antagonism
C. Phagocytic D. Direct synergism
Allergy D. Hemopoietic E. Indirect synergism
E. Bilification
Tachyphylaxis E. Sensitization C. Sensibilization
A patient used an indirect-acting adrenergic agonist D. Functional cumulation
to treat rhinitis. After the patient has been putting A patient who has been taking a certain drug for a E. Tachyphylaxis
in the nose drops for several days, the long time cannot discontinue the use of it because
vasoconstrictive effect of the drug gradually this causes psychic and somatic disfunctions. The A patient with heart failure and tachycardia has
diminished. Name this phenomenon: syndrome occuring at refraining from the use of a been prescribed digoxin. After 5 days of taking
A. Tachyphylaxis drug is called: digoxin the patient’s heart rate was normalized.
B. Idiosyncrasy A. Abstinence After 2 weeks the patient addressed a doctor due to
C. Teratogenicity B. Sensitization continuous decrease of heart rate down to 52/min.
D. Allergy C. Idiosyncrasy What phenomenon has caused such changes of
E. Cumulation D. Tachyphylaxis heart rate?
E. Cumulation A. Cumulation
A patient noticed symptoms of approaching attack B. Tachyphylaxis
of bronchial asthma and took several tablets one by A patient diagnosed with morphinism has been C. Allergy
one at short intervals out of the doctor’s control. admitted to the narcological department. A doctor D. Tolerance
Short-term improvement of his condition came noted a decrease in pharmacological activity of E. Idiosyncrasy
only after taking the first two tablets. Next intakes morphine. Repetitive use of a drug may result in
of a drug didn’t improve his condition. Reduction tolerance to its effect, and this phenomenon is Teratogenesis
of the drug effectiveness was caused by: called: A woman who had been consuming alcohol
A. Tachyphylaxis A. Addiction excessively during her pregnancy had a child with
B. Cumulation B. Cumulation cleft palate and upper lip. These presentations are
C. Addiction C. Tachyphylaxis indicative of some chromosomal anomalies. What
D. Dependence D. Antagonism process do they result from?
E. Idiosyncrasy E. Allergy A. Teratogenesis
B. Ontogenesis
A patient has been taking bisacodyl for a long time Cumulation C. Mutagenesis
to treat chronic constipation. However, several A patient with chronic heart failure has been taking D. Carcinogenesis
weeks later the aperient effect of the drug digoxin for several months on an outpatient basis. E. Phylogenesis
diminished. What is the possible cause of this? At a certain stage of treatment, he got symptoms of
A. Acquired tolerance drug overdose. What effect underlies the A woman who had taken alcohols during her
B. Drug dependence development of this complication? pregnancy had a child with cleft palate and upper
C. Material cumulation A. Material accumulation lip. These presentations are indicative of some
D. Functional cumulation B. Adaptation
chromosomal anomalies. What process do they E. Cumulation.
result from? A patient who has been treated with diazepam on
A. Teratogenesis account of neurosis complains of toothache. Doctor What is the mechanism of absorption of drug
B. Carcinogenesis administered him an analgetic, but its dose was which is lipophilic and unionized?
C. Mutagenesis lower than average therapeutic dose. What A. Active transport
D. Phylogenesis phenomenon did the doctor take into account while B. Passive diffusion*
E. Ontogenesis prescribing the patient an underdose? C. Filtration
A. Summation D. Pinocytosis
A patient with angina pectoris was helping himself B. Potentiation* E. Binding to protein.
with nitroglycerin. One day the patient used C. Cumulation
nitroglycerin several times because of frequent D. Tolerance Which route of administration results in 100%
episodes of pain. So as the last sublingual E. Drug dependence. bioavailability of drug?
administration of nitroglycerin did not relieve pain, A. Oral
the patient addressed his doctor. Doctor advised A patient with frequent attacks of stenocardia was B. Subcutaneous
him to withdraw nitroglycerin for 8 hours and prescribed sustak-forte to be taken one tablet twice C. Inhalational
prescribed another drug. After 8 hours the effect of a day. At first the effect was positive but on the D. Intravenous*
nitroglycerin was restored. Which phenomenon did second day stenocardia attacks resumed. What can E. Intramuscular.
take place in this case? explain inefficiency of the prescribed drug?
A. Increase in nitroglycerin metabolism A. Tachyphylaxis* Chloramphenicol is highly toxic in newborn. What
B. Increase in protein binding B. Sensibilization is the cause of such toxicity?
C. Decrease of absorption C. Dependence A. Intensive binding to protein
D. Drug dependence D. Cumulation B. Deficiency of metabolizing enzymes in
E. Tolerance*. E. Idiosyncrasy. liver*
C. Penetration into the cell
Metabolism can render pharmacological activity to A man who has been taking a drug for a long time D. Inhibition of protein synthesis
some initially inert substances (pro-drug). Point out cannot withhold it because this causes impairment E. Inhibition of nucleic acid synthesis.
the substance, which is pro-drug, used for of psychic, somatic and vegetative functions. Name
treatment of hypertension. the syndrome of different disturbances caused by What is the result of tubular reabsorption of drug?
A. Enalapril* drug discontinuation: A. Increase in duration of action*
B. Salbutamol A. Idiosyncrasy B. Increase in excretion
C. Verapamil B. Sensibilization C. Increase in distribution
D. Reserpine C. Tachyphylaxis D. Decrease in duration of action
E. Propranolol. D. Abstinence* E. Loss of pharmacologic activity.
B. Cumulation intoxication (bradycardia, extrasystole, nausea)
Anticoagulant drug warfarin is bounding to plasma C. Potentiation* were developed. Point out the phenomenon
albumin. How its effect can be changed in case of D. Tolerance responsible for intoxication.
hypoalbuminemia? E. Additive synergism. A. Tachyphilaxis
A. Effect unchanged B. Material cumulation*
B. Increased* Patient with infection of GIT was treated with C. Idiosyncrasy
C. Decrease antibiotic. Which type of therapy is it? D. Functional cumulation
D. Significant delayed onset of effect A. Symptomatic E. Tolerance.
E. Abolishing of effect . B. Pathogenetic
C. Ethiotropic* The patient with heart failure was using digitoxin.
Passive reabsorption from tubular urine (pH of 5) D. Replacement His condition initially improved, but with time the
of a drug administered IV would be reduced if the E. Stimulating. manifestations of heart failure have increased
drug were again, and symptoms of glycoside intoxication
A. A weak base* Alongside with therapeutic effect drugs can cause apeared. What is the cause of patient's condition
B. A weak acid side effects. In which dose they can occur? worsening?
C. Water soluble A. In overdose A. Potentiation
D. Protein bound B. In therapeutic dose* B. Cumulation*
E. None of the above. C. In threshold dose C. Tachyphylaxis
D. In subthreshold dose D. Sensitization
Patient suffering from angina pectoris was helping E. At any dose. E. Idiosyncrazy.
himself by sublingual usage of nitroglycerin. One
day he used the drug several times. The last use did Patient suffering from gastric peptic ulcer was Analgesic effect of methamizole(analgin) in the
not relieve his pain. What phenomenon does treated with M-cholinoblocker. What is the dental pain develops in 0.5 hour and lasts for 1.5-2
occur? mechanism of this drug action? hours. Tablets "Piraminal" in the same situation are
A. Antagonism A. Enzymatic faster and longer acting. What is the phenomenon
B. Tachyphylaxis* B. Receptor* responsible for difference in the action of these
C. Hypersensitivity C. Direct chemical drugs?
D. Cumulation D. Physicochemical A. Chemical antagonism
E. Binding to protein. E. Action on ion channels of membrane. B. Functional antagonism
C. Undirectional antagonism
Point out the kind of drug interaction that permits A patient with chronic cardiac insufficiency was D. Potentiation
to use one of them in lower dose. prescribed digoxin in average therapeutic dose. E. Summation*.
A. Antagonism After 2 weeks of treatment the symptoms of
In the elderly patient with symptoms of senile With long-term treatment by antipsychotic drugs at
atonic constipation long-term treatment with drug In a patient with traumatic brain injury there are therapeutic doses in patient, along with the
buckthorn has led to a weaking effect. What is this such symptoms: the obscured consciousness, restoration of the psychic function, some undesired
phenomenon? inpaired respiration, filiform pulse, absence of effects such as tension in the skeletal muscles,
A. Tolerance* reflexes. Which route of drug administration for stiffness of gait, and some autonomic disorders
B. Cumulation urgent aid is the most appropriate in this case? occurred. What the type of drug’s effects should
C. Sensibilization A. Rectal include these?
D. Desensitization B. Intravenous* A. Aftereffect
E. Tachyphylaxis. C. Subcutaneous B. Overdose
D. Oral C. The main therapeutic effect
During the barbituric anesthesia in a patient E. Inhaled. D. Intoxication
respiratory depression occurred. After intravenous E. Side effects*.
injection of 10 ml 0.5% solution of bemegride the In poisoning with mushrooms containing
patient's condition improved, the volume of muscarine atropine is used as an antidote. Which of After clinical and laboratory examination of a
pulmonary ventilation inceased. Point out the the following mechanisms, typical for atropine, patient with diabetes mellitus insulin was
phenomenon responsible for curative effect of leads to the neutralization of the poison mushroom appointed for treatment.What is the type of this
bemegride in this case - muscarine? therapy?
A. Direct synergisnm A. Antimetabolic A. Etiotropic
B. Undirectional antagonism B. Physical and chemical B. Replacement*
C. Direct antagonism* C. Antienzymatic C. Pathogenetic
D. Indirect antagonism D. Receptor* D. Symptomatic
E. Indirect synergism. E. Enzymatic. E. Functional.

A child of 2 years admitted to the infectious Prolonged use of certain drugs before pregnancy In order to reduce toothache a patient was
deseases hospital with a diagnosis of bacillary can disrupt fetal development. What is the name of recommended to take methamisol (analgin). What
dysentery. What a way of enteral administration of this action? the type of pharmacotherapy was used?
antimicrobial agent polymyxin is the most rational, A. Fetotoxic A. Functional
if child has uncontrollable vomiting? B. Embriotoxic B. Pathogenetic
A. Subcutaneous C. Teratogenic C. Causal
B. Intravenous D. Mutagenic* D. Substitution
C. Oral E. Carcinogenic. E. Symptomatic*.
D. Rectal*
E. Inhaled.
In 18 years old patient with shoulder phlegmon A. Acetylcholine A. Physostigmine
intramuscular injection of penicillin was made, B. Noradrenaline B. Aceclidine
after which he developed tachycardia, filiform C. Dopamine C. Cytiton
pulse, fall of blood pressure to 80/60 mm Hg. What D. Serotonin D. Carbacholine
type of pathological response was originated? E. Glycine E. Phosphacol
A. Potentiation Proserin
B. Anaphylaxis* A patient after disrupted cerebral circulation has M-cholinоblockers
C. Tolerance developed paralysis. Choose the anticholinesterase After a patient had taken a blocking agent, his heart
D. Antagonism drug to be prescribed in this case: rate (HR) increased. Pressing on the eyeballs didn’t
E. Tachyphylaxis. A. Proserin result in the expected reflectory decrease in heart
B. Cordiamin rate. What exactly was blocked by the drug in the
AGENTS AFFECTING C. Aceclidine pacemaker cells?
NEUROTRANSMISSION IN CHOLINERGIC D. Methacin A. M-cholinergic receptors
SYNAPSES (58) E. Hexamethonium (Benzohexonium) B. α-adrenergic receptors
C. β-adrenergic receptors
M-cholinоmimetics On the 2-3 day after stomach resection a patient is D. Ca2+-L-type channels
A man got poisoned with mushrooms. They still experiencing a failure of intestinal peristalsis. E. Fast Na+ channels
contain muscarine that stimulates muscarinic In order to stimulate the motility of gastrointestinal
cholinoreceptors. What symptom is typical for tract the following drug should be administered: Patient with complaints of dryness in the mouth,
poisoning with inedible mushrooms? A. Proserin photophobia and disturbances of vision was
A. Miosis B. Prazosin admitted to the reception-room. Skin is hyperemic
B. Mydriasis C. Cyclodolum and dry, pupils are dilated, tachycardia. Poisoning
C. Bronchi dilation D. Atropine sulphate with belladonna alkaloids was diagnosed on further
D. Heart rate rise E. Noradrenaline hydrotartrate examination. What medicine should be prescribed
E. Arterial pressure rise as antidote?
Physostigmine A. Diazepam
Acetylcholine In order to reduce salivation before a B. Prozerin (Neostigmine)*
A patient presented to a hospital with complaints stomatological procedure a dentist gave his patient C. Atropine
about quick fatigability and significant muscle 10 drops of 0,1% solution of atropine sulfate D. Pilocarpine
weakness. Examination revealed an autoimmune perorally. 30 minutes later the patient started E. Dipyroxim.
disease that causes functional disorder of receptors complaining of acute pain in the eyeballs, misty
in the neuromuscular synapses. This will result in vision, headache, palpitation. These symptoms Patient with pin-point pupils, sweating, salivation,
the disturbed activity of the following mediator: were eliminated by means of the following drug: abdominal cramps, disturbances in respiration was
admitted to the emergency room. Poisoning with anticholinesterase drug moderately improving patient did not restored spontaneous breathing.
what substance is the most likely occurred? mental development. Choose this drug. What kind of drug is able to restore breathing in
A. Non-narcotic analgesics A. Physostigmine this case?
B. Sedatives B. Neostigmine (Proserinum) A. Scopolamine
C. Narcotic analgesics C. Galanthamine* B. Aceclidine
D. М-cholinergic antagonists D. Pilocarpine C. Pilocarpine
E. Muscarine*. E. Lobeline. D. Neostigmine*
E. Izonitrozinum.
A known patient of myasthenia gravis came to you A patient with complaints of dryness in the mouth,
with his disease well controlled by neostigmine; photophobia and vision impairment was admitted A patient with signs of poisoning by
but he asked you to give him drug that needs more in the reception-room. Skin is hyperemic, dry, organophosphorus insecticides was delivered to the
rare usage. Choose the necessary drug. pupils, are dilated, tachycardia. Poisoning with hospital emergency department What medication
A. Physostigmine belladonna alkaloids was diagnosed on further should be applied as a first-aid?
B. Atropine examination. What medicine should be A. Atropine*
C. Ambenonium chloride* prescribed? B. Unitiol
D. Armin A. Armine C. Panangin
E. Pralidoxime. B. Dipiroxim D. Chlorpromazine
C. Pilocarpine E. Glucose.
A49-year old frequent business traveler presents to D. Diazepam
his physician requesting medication to help him E. Prozerin*. The patient was taken to the emergency department
with nausea and dizziness that he gets during with complaints of dryness in the mouth,
turbulent flights. A scolopamine patch is On the 2-3rd day after stomach resection intestinal photophobia and blurred vision. On examination
prescribed for his motion sickness. Which of the peristalsis wasn't restored. What is to be hyperemic dry skin, dilated pupils, tachycardia
following is the most likely side effects from this administered for stimulation of gastrointestinal were revealed, and poisoning with belladonna
patch? tract? alkaloids was diagnosed. Which of the drugs is
A. Bradycardia A. Atropine sulfate advisable to apply?
B. Diaphoresis B. Prasosin A. Neostigmine*
C. Diarrhea C. Acetylcholine B. Pilocarpine
D. Salivation D. Proserin(Neostigmine)* C. Dipiroxim
E. Urinary retention*. E. Cyclodole. D. Diazepam
E. Arminum.
In complex treatment of a child, suffering from After completion of abdominal surgery conducted
cerebral palsy, a doctor decided to include with the use of nondepolarizing myoreolaxant, the
The patient in the postoperative period to stimulate A patient who takes blocker of membrane though the speech is indistinct. BP is 110/70
intestinal motility and tone of the bladder has been cytoreceptors of efferent conductor synapses of mm Hg, heart rate is 110/min. Given symptoms
appointed agent of a group of anticholinesterase. autonomic nervous system complains about dry can indicate overdosage of the following drug:
Identify it among the following products: mouth. What receptors are blocked? A. Atropine
A. Propranolol (Anaprilin) A. Muscarinic cholinoreceptors B. Morphine
B. Mannitol B. Nicotinic cholinoreceptors C. Ephedrine
D. Aminazi
C. Reserpine C. H2 -receptors E. Caffeine
D. Dichlotiazid D. α-adrenoreceptors
E. Neostigmine*. E. β-adrenoreceptors A patient has been given atropine sulfate for rapid
relief of spastic colon symptoms. The use of this
Specify the M-cholinomimetic used for treatment A patient has a spasm of smooth muscles of drug is contraindicated during the followng
of glaucoma: bronchi. As the first aid it would be physiologically disease:
A. Neostigmine methylsulfate appropriate to inject the patient the antagonists of A. Glaucoma
B. Pilocarpine* the following receptors: B. Bronchial asthma
C. Adrenaline A. M-cholinoreceptors C. Bradycardia
D. Norepinephrine B. α -adrenoreceptors D. Hypotension
E. Phenylephrine (Mesatonum). C. N-cholinoreceptors E. Gastric ulcer
D. β-adrenoreceptors
Which of the following medicines is E. Adenosine receptors A patient was taken to a hospital with dizziness,
anticholinesterase agent: dry mouth, mydriatic pupils, accommodation
A. Clonidine In dental practice atropine sulfate is used to disorder, tachycardia, difficult urination, intestinal
B. Atropine decrease salivation. This drug belongs to the atony. These symptoms might have been caused by
C. Adrenaline following group: overdose of the following drug:
D. Acetylcholine A. Muscarinic antagonists A. Atropine sulfate
E. Neostigmine*. B. α-adrenergic agonists B. Furosemide
C. β-adrenoceptor antagonists C. Clonidine
Which group is pilocarpine? D. Ganglionic blockers D. Captopril
A. M-cholinomimetics* E. Muscle relaxants E. Prazosin
B. Anticholinesterase
C. Adrenomimetics Atropine sulfate A patient in grave condition has been delivered to
D. Adrenoblockers A patient complains of dizziness, thirst, difficult
the admission ward. Examination revealed pupil
E. Cholinoblockers. swallowing, and impaired vision of close objects.
mydriasis, no reaction to the light, considerable
Objectively: respiratory rate is increased, pupils
are dilated, general agitation, talkativeness, reddening and dryness of skin and mucous
membranes. What drug might have caused the C. Ganglionic synapses and cessation of anesthesia the independent
intoxication symptoms? D. Membrane conduction of excitement respiration wasn't restored. Which enzyme deficit
A. Atropine sulphate E. Proprioceptors prolongs the action of muscle relaxant?
B. Proserin A. Pseudocholinesterase
C. Adrenalin hydrochloride During an operation a patient got injection of B. Succinate dehydrogenase
D. Pilocarpine hydrochloride muscle relaxant dithylinum. Relaxation of skeletal C. Carbanhydrase
E. Dithylinum muscles and inhibition of respiration lasted two D. N-acetyltransferase
hours. This condition was caused by absence of the E. K-Na-adenosine triphosphatase
A patient was delivered to the admission ward with following enzyme in blood serum:
poisoning with an insecticide of anticholinesterase A. Butyrylcholin esterase A patient with dislocated jaw was given a short-
action. What drug able to block muscarinic B. Catalase acting muscle relaxant by a doctor. Name this
cholinoreceptors should be prescribed? C. Acetylcholinesterase drug:
A. Atropine sulfate D. Glucose 6-phosphatase A. Dithylinum (Suxamethonium chloride)
B. Pilocarpine hydrochloride E. Glutathione peroxidase B. Procaine
C. Dithylinum C. Cytitonum (Cytisine)
D. Benzohexonium The first-aid center has received a victim of a D. Papaverine hydrochloride
E. Mesatonum traffic accident diagnosed with closed displaced E. Pyridostigmine hydrobromide
racture of the middle third of the thigh. For
A patient has applied eye drops containing atropine repositioning of bone fragments the patient A patient with limb fracture should be prescribed a
which resulted in persistent mydriasis. Which received 10 ml of 2% dithylinum solutionin depolarizing muscle relaxant for brief surgical
muscle was blocked? travenously, which resulted in prolonged period of invasion. Name this drug:
A. Pupil-contracting apnoea and musclerelaxation.What enzyme is A. Dithylinum
B. Pupil-dilating deficient, resultingin such pharmacogenetic B. Azamethonium bromide (Pentaminum)
C. Ciliate enzymopathy? C. Cytisinum (Cytitonum)
D. Rectus A. Pseudocholinesterase D. Tubocurarin chloride
E. Oblique B. Uridine diphosphate glucuronyltransferase E. Atropine sulfate
C. Glucose 6-phosphate dehydrogenase
Dithylinum (Suxamethonium chloride) D. Methemoglobinreductase Hygronium
Curarelike substances (dithylinum) make it E. N-acetyltransferase During a surgery, the anesthesiologist used a
impossible for skeletal muscles to contract because ganglionic blocker for controlled hypotension.
they block: A patient had to go through an operation. Doctors What drug was given the patient in this case?
A. Neuromuscular synapses introduced him dithylinum (listenone) and A. Hygronium
B. Central synapses performed intubation. After the end of operation B. Benzohexonium
C. Pirilenum C. Neostigmine D. Hyascine
D. Pentaminum D. Blood transfusion* E. Isonitrosine.
E. Pachycarpinum E. Caffeine.
For testing refraction of eye atropine was instilled
Teenager with fracture of hand was presented to A 59 year old man develops excessive salivation into conjunctival sac. On completion of the
the emergency room. To perform reposition of and sweating, diarrhea, and bradycardia while procedure another cholinergic drug was used to
fractured bones it was necessary to relax skeletal being treated with neostigmine for myasthenia counteract mydriasis and cycloplegia, caused by
muscle. For this purpose myorelaxant causing gravis. Which of the following is the most atropine. Point this drug.
sustained depolarization of the post junctional appropriate therapy for these symptoms and signs? A. Pilocarpine*
membrane was administered. Which of the A. Atropine* B. Lobeline
following agents was used? B. Carbachol C. Hyoscine (Scopolamine)
A. Pancuronium C. Edrophonium D. Phenylephrine (Mesatonum)
B. Atracurium D. Epinephrine E. Pirenzepine.
C. D-tubocurarine E. Pralidoxime.
D. Succinylcholine* Administration of pirenzepine in patients with
E. Millectine. A patient with symptoms of phosphorganic gastric peptic ulcer is not accompanied by
poisoning was taken to emergency department. numerous side effects characteristic for atropine
Patient suffering from gastric peptic ulcer was Which of the following drugs mast be used as a and other M-cholinoblckers due to
treated with cholinotropic drug selectively drug of first aid? A. Inability to penetrate through blood brain
blocking M1 cholinoceptors of parietal gastric A. Unithyolum barrier
glands. Point out this drug? B. Potassium chloride B. Selective inhibition of M1 –cholinoceptors*
A. Atropine C. Atropine* C. Inhibition of all types of M –cholinoceptors
B. Plathyphylline D. Chlorpromazine (Aminazine) D. Inhibition of cholinesterase
C. Metacine E. Glucose. E. Significant protein binding.
D. Pirenzepine*
E. Hyoscine (scopolamine). After abdominal surgery performed under general It is known that administration of tubocurarine
anaesthesia with non- depolarizing myorelaxant during surgery can enhance the development of
A patient given succinylcholine for skeletal muscle spontaneous respiration was not restored. Which of postoperative paralytic ileus. What is the
relaxation during an operation, is not recovered for the following drugs is to be used as an antidote of mechanism of this side effect of tubocurarine?
the last 1 hour. What must be used to restore non-depolarizing myorelaxant? A. Inhibition of M –cholinoceptors
spontaneous respiration? A. Aceclidinum B. Inhibition of cholinesterase
A. Carbogen B. Pilocarpin C. Ganglion blocking activity*
B. Lobelin C. Neostigmine (Proserin)* D. Stimulation of M –cholinoceptors
E. Stimulation of both M – and N- respiration wasn't restored. Which enzyme deficit characteristic side effects such as blurred,
cholinoceptors. prolongs the action of muscle relaxant? constipation tachycardia do not appear. Point out
A. Pseudocholinesterase* the drug that patient uses.
A 50-year-old male farm worker has been brought B. K-Na-adenosine triphosphatase A. Platifillin
to the emergency room. He was found confused in C. N - acetyltransferase B. Atropine
the orchard and since then has remained D. Succinate dehydrogenase C. Metacin
unconscious. His heart rate is 45 and his blood E. Carbanhydrase. D. Ipratropium Bromide*
pressure is 80/40 mm Hg. He is sweating and E. Pirenzepine.
salivating profusely. Which of the following A patient with fracture of his lower jaw was
should be prescribed? admitted to the maxillofacial department. It was The drug selectively blocking M1 - cholinergic
A. Pentamine decided to fix his bones surgically under receptors was assigned to patient suffering from
B. Norepinephrine anaesthetic. After intravenous introduction of gastric peptic ulcer. What this drug is?
C. Physostigmine muscle relaxant there arose short fibrillar A. Pirenzepine*
D. Atropine* contractions of the patient's facial muscles. What B. Atropine
E. Proserine. muscle relaxant was applied? C. Platyphylline
A. Dithylinum* D. Methacinum
Introduction of a pharmaceutical substance to an B. Pipecuronium bromide E. Scopolamine.
experimental animal resulted in reduction of C. Diazepam
salivation, pupil mydriasis. Next intravenous D. Melictine Dentist in order to reduce salivation during the
introduction of acetylcholine didn't lead to any E. Tubocurarin chloride. filling of the tooth prescribed the drug to the
significant changes of heart rate. Name this patient. What the medicine was prescribed?
substance: If a man has an attack of bronchiospasm it is A. Adrenalin hydrochloride
A. Proserin necessary to reduce the effect of vagus on smooth B. Phenylephrine (Mezaton)
B. Atropine* muscles of bronchi. What membrane cytoreceptors C. Neostigmine
C. Propranolol should be blocked for this purpose? D. Pilocarpine hydrochloride
D. Adrenaline A. M-cholinoreceptors* E. Atropine sulfate*.
E. Salbutamol. B. beta-adrenoreceptors
C. alfa- and beta-adrenoreceptors Iintroduction of the drug in experimental animals
A patient had to go through an operation. Doctors D. alfa-adrenoreceptors decreases salivation, dilates pupils, and on the
introduced him dithylinum (listenone) and E. N-cholinoreceptors. subsequent intravenous injection of acetylcholine
performed intubation. After the end of operation heart rate did not change significantly. Specify the
and cessation of anesthesia the independent The patient with bronchial asthma uses inhaled name of the substance:
form M - holinoblocker. In this case, as a rule, A. Propranolol (Anaprilin)
B. Neostigmine methylsulfate C. Physostigmine Condition of a patient with diabetes mellitus
C. Adrenaline D. Isoprenaline sharply deteriorated after a regular injection of
D. Salbutamol E. Pyridostigmine bromide. insulin. The patient became anxious and broke out
E. Atropine*. in cold sweat; tremor of the extremities, general
In the patient with a forearm injury during the bone weakness, and dizziness appeared. What medicine
Dithylinum was injected to the patient before repositioning Dithylinum was used for muscle can remove these symptoms?
surgery and intubation was performed. After the relaxation. Full recovery of muscle tone and A. Adrenaline
operation, and termination of anesthesia function took more than an hour. What can be the B. Tolbutamide
spontaneous breathing was not restored. Deficiency cause of significant lengthening of the drug’s C. Caffeine
of which enzyme is responsible for prolongation of action? D. Noradrenaline
this muscle relaxant action? A. Genetic deficiency of monoamine oxidase E. Glibutid (Bufo min)
A. Carbonic anhydrase B. Genetic deficiency of
B. Pseudocholinesterase* butyrylcholinesterase* In a surgical department of a stomatological
C. Succinate dehydrogenase C. Inhibition of microsomal oxidation polyclinic a patient is being prepared for tooth
D. K-Na-ATPase D. Genetic deficiency of hydroxylase extraction. What drug should be added to the
E. N-acetyltransferase. E. Formation of the active metabolite. solution of a local anaesthetic in order to prolong
its action?
The preparation of the depolarizing neuromuscular AGENTS AFFECTING A. Adrenalin hydrochloride
blocking agents should be assign to the patient with NEUROTRANSMISSION IN ADRENERGIC B. Noradrenaline hydrotartrate
a broken limb for short-lasting surgical procedure. SYNAPSES (60) C. Isadrine
Choose the drug. D. Salbutamol
A. Atropine sulfate Adrenalin hydrochloride (Epinephrine E. Octadine
B. Pentamin hydrochloride)
C. Dithylinum* During introduction of local anesthesia Norepinephrine
D. Cytitonum a patient has gone into anaphylactic shock. Monoamine oxidase inhibitors are widely used in
E. Tubocurarine chloride. What drug must be administered to the patient? clinics as psychopharmacological drugs. They
A. Epinephrine hydrochloride change the level of the following neurotransmitter
In a patient with myasthenia after the treatment B. Diazepam in the synapses:
with neostigmine nausea, diarrhea, and C. Atropine sulfate A. Norepinephrine
hypersalivation appeared. Point out the drug to D. Propranolol B. Acetylcholine
eliminate these side effects. E. Nitroglycerin C. ATP
A. Atropine Sulfate* D. Substance P
B. Phenylephrine (Mezaton) E. L-glutamate
During a visit to a dentist a patient has developed A doctor has prescribed clonidine for rapid relief of
Ephedrine hydrochloride collapse. What drug can be applied to manage this hypertensic crisis. What group does this drug
A patient with arterial hypertension has developed situation? belong to according to its mechanism of action?
a bronchial asthma attack. Which of the following A. Mesaton A. Central α2-adrenoceptor agonist
bronchodilators may provoke a hypertensive crisis? B. Strophanthine B. α-adrenergic blocking agent
A. Ephedrine hydrochloride C. Propanolol C. Nonselective α-adrenoceptor agonist
B. Salbutamol D. Nitroglycerine D. Selective α1-adrenoceptor agonist
C. Aminophylline E. Seduxen E. Selective β1-adrenergic blocking agent
D. Berotec
E. Isadrine Due to overdosage of a vasodilator a 58-year-old A patient was administered clonidine to be taken
patient has developed acute vascular insufficiency. parenterally in case of abrupt rise of arterial
Methyldopa What drug would be advisable for termination of pressure. What is its mechanism of action?
A 58-year-old patient with essential hypertension this pathological condition? A. Stimulation of central α 2-
complains about general health deterioration. She A. Mesaton (Phenylephrine) adrenoreceptors
has been administered methyldopa. Specify the B. Euphyllin (Aminophylline) B. Block of nicotinic cholinoreceptors of
mechanism of this drug action: C. Dopamine ganglia
A. Inhibtion of noradrenaline synthesis D. Cordiamin C. Block of α1- and α2-adrenoreceptors
B. Increase in acetylcholine synthesis E. Aethimizolum (Methylamide) D. Block of α1-adrenoreceptors
C. Blockade of α-adrenergic receptors E. Stimulation of central imidazole-1 receptors
D. Blockade of β-adrenergic receptors Clophelin (Clonidine)
E. Inhibition of angiotensin-converting factor A 45-year-old patient with essential hypertension, Dobutamine
who has been taking an antihypertensive drug for 4 A patient with acute heart failure refractory to
Phenylephrine (Mesaton) days, notes that his blood pressure is normalized; cardiac glycosides was given an injection of
A patient in a collaptoid state has been given an however the patient complains of sleepiness and dobutamine. What is the mechanism of action of
injection of mesatonum for the correction of blood sluggishness. What drug is the patient taking? this drug?
pressure. What is the mechanism of this drug A. Clophelin (Clonidine) A. Stimulation of β1-adrenergic receptors
action? B. Prazosin B. Complexation with membrane
A. It stimulates α-adrenergic receptors C. Captopril phospholipids
B. It stimulates β-adrenergic receptors D. Enalapril C. Inhibition of K+, Na+- ATPase
C. It blocks α-adrenergic receptors E. Apressin (Hydralazine) D. Inhibition of phosphodiesterase activity
D. It blocks β-adrenergic receptors E. Increase of n.vagus tonus
E. It stimulates α- and β-adrenergic receptors
Isadrin
A patient with bronchial asthma has been While being at the dentist’s a patient had an attack salbutamol. This drug relates to the following
administered inhalations of 0,5% isadrin solution. of bronchial asthma. The dentist applied a β2- group of therapeutic agents:
This helped to relieve bronchiospasms but the adrenomimetic drug in form of inhalations. What A. β2-adrenomimetics
patient started complaining of heart pain and drug was applied? B. α-adrenomimetics
palpitation. What is the cause of these A. Salbutamol C. β1-2-adrenomimetics
presentations? B. Aminophylline D. Sympatholytics
A. β1-adrenoreceptor stimulation C. Adrenaline hydrochloride E. α-β-adrenomimetics
B. β2-adrenoreceptor stimulation D. Atropine sulfate
C. α-adrenoreceptor stimulation E. Ephedrine hydrochloride A patient with bronchial asthma is administered
D. M-cholinoreceptor activation inhalation of 0,5 percent isadrine solution.
E. Inhibition of acetylcholine synthesis A patient with bronchial asthma has developed a Bronchospasm was relieved but the patient began
bronchial spasm during the visit to a dentist. Name complaining of pain in the heart region and
Salbutamol the drug necessary to arrest the spasm: palpitation. It is connected with the stimulation of :
A 38-year-old woman developed an attack of A. Salbutamol A. alpha one – adrenoreceptros
bronchial asthma. What bronchial spasmolytic for B. Naphthizin B. Beta two – adrenoreceptros
emergency medical aid is a beta-2-adrenergic C. Mesaton (Phenylephrine) C. Beta one – adrenoreceptros*
agonist? D. Anaprilin (Propranolol) D. Acetylchloline synthesis
A. Salbutamol E. Bisoprolol E. M-chlolinoceptors.
B. Adrenaline
C. Ipratropium bromide A 42-year-old woman has been administered Spasm of smooth muscle of bronchi developed in
D. Platyphyllin propranolol for the ischemic heart disease. Yet she the patient. Usage of activators of what membrane
E. Atropine has been found to have a disease in case of which cytoreceptors is physiologically valid to decrease
the use of propranolol is contra-indicated. What attack?
A 60-year-old patient with a history of bronchial disease is it? A. alpha-аdrenoreceptors
asthma has had several attacks during the day. A. Bronchial asthma B. beta-adrenoreceptors*
What is the optimal drug to be used for their B. Cholecystitis C. alpha-and- beta-аdrenoreceptors
prevention? C. Arterial hypertension D. N-cholinoreceptors
A. Salbutamol D. Duodenal ulcer E. М-cholinoreceptors.
B. Isadrinum E. Myasthenia
C. Adrenaline hydrochloride In the treatment of patient suffering from
D. Dobutamine A patient had an attack of bronchial asthma in the congestive heart failure resistant to digitalis
E. Methacinum dentist’s office. The attack was arrested by adrenomimetic was used as inotropic agent. Point
out which of the following drugs was administered.
A. Phenylephrine C. stimulation of beta1– adrenoceptros* intravenously drug that promptly improved the
B. Partusisten D. stimulation of beta2– adrenoceptros patient's condition. Which of the following drugs
C. Dobutamine* E. inhibition of M-chlolinoceptors. can be used for first aid in severe allergic
D. Xylometazoline reactions?
E. Naphazoline. A patient with moderate hypertension was treated A. Ipratropium bromide
with the drug that realizes its effect by decrease in B. Labetalol
63 years patient with symptoms of collapse was noradrenaline release from sympathetic nerve C. Fenoterol
admitted into emergency department. Doctor has endings. While treatment patient experienced D. Adrenaline*
chosen noradrenaline to overcome collapse. What drowsiness. What the drug was used? E. Dobutamine.
mechanism is responsible for therapeutic effect of A. Prazosin
this drug? B. Reserpin* A patient with heart failure, resistant to cardiac
A. Activation of alfa1 receptors* C. Drotaverine glycosides was admitted to hospital. A doctor
B. Activation of beta adrenoceptors D. Atenolol appointed adrenomimetic as inotropic (cardiotonic)
C. Activation of dopaminergic receptors E. Dichlorothiazide. drug. Which of these adrenomimetic has inotropic
D. Blockade of M –cholinoceptors (cardiotonic) action?
E. Activation of serotonin receptors.. Surgical treatment of benign tumor of prostate in A. Dobutamine*
67- year patient was postponed instead B. Phenylephrine (Mezaton)
To perform fundoscopy ophthalmologist instilled pharmacotherapy with adrenergic drag was started. C. Naphazoline
in the eye an agent capable of causing mydriasis Point out which of the drugs was most probably D. Fenoterol
without cycloplegia. Point out this agent. used. E. Halazolin.
A. Phenylephrine (Mesatonum)* A. Adrenaline
B. Noradrenaline B. Ephedrine The patient went to the doctor for medical
C. Atropine C. Prazosine* examination. In the queue while waiting an attack
D. Pilocarpine D. Propranolol (Anaprilinum) of asthma originated. Which group of drugs is
E. Isoprenaline (Isadrinum). E. Atenolol . advisable to use to provide first aid?
A. N - cholinomimetics
A patient with bronchial asthma addresses his At the doctor's the dentist- surgeon injected 1% - B. Sympathomimetics
doctor with complaints about unpleasant procaine for conduction anesthesia. It caused the C. Beta-adrenomimetics*
palpitations that occur after usage of inhalation change in patient’s condition: the skin was covered D. Beta-adrenoblockers
form of isoprenaline (isadrinum).What is the cause with sticky and cold sweat, the pulse became E. M - cholinomimetics.
of this side effect? almost impalpable, breathing became rare and
A. stimulation of alfa1– adrenoceptros weak. Realizing that anaphylactic shock is In anesthetic practice adrenaline is usually added to
B. stimulation of alfa2– adrenoceptros originated, a doctor immediately introduced the local anesthetics to prolong analgesia and
reduce side effects. What the effect of adrenaline that occurred in half an hour, ephedrine did not has been found to have a concomitant condition
contributes to this? cause any effect.What is the name of this that renders propranolol to be contraindicated.
A. Reducing the sensitivity of pain receptors phenomenon? What disease is it?
B. Constriction of blood vessels* A. Addiction A. Bronchial asthma
C. Increase in peripheral resistance B. Biotransformation B. Arterial hypertension
D. Stimulation of the antinociceptive system C. Elimination C. Cholecystitis
E. CNS depression. D. Adsorption D. Duodenal ulcer
E. Tachyphylaxis*. E. Myasthenia
In the course of therapy of a patient a number of
complications such as mydriasis, tachycardia, and In a patient during a visit to the dentist an acute A patient with essential hypertension was admitted
increased blood pressure have occurred. Which of hypotension occurred. Which of the drugs that to the cardiological department. In order to lower
the listed drugs can cause such side effects? stimulate adrenergic structures should be used to arterial pressure a doctor prescribed a drug that
A. Propranolol (Anaprilin) normalize blood pressure? blocks β1 and β2-adrenoreceptors. What drug is it?
B. Adrenaline* A. Naphazoline A. Propranolol
C. Phentolamine B. Xylometazoline B. Proserin
D. Isoprenaline C. Phenylephrine (Mezaton)* C. Celecoxib
E. Norepinephrine. D. Ergotamine D. Prednisolone
E. Doxozasin. E. Indometacin
A pregnant woman with a danger of abortion is
admitted to obstetric department. Which of these Propranolol The patient with a history of asthma have
drugs should be applied to save the pregnancy? A patient suffering from ciliary arrhythmia with developed atrial fibrillation, that requires agent
A. Salbutamol* anamnesis of bronchial asthma should be appointment. Which of the listed antiarrhythmic
B. Oxytocin prescribed an antiarrhythmic drug. What drug is contraindicated in this case?
C. Neostigmine methylsulfate antiarrhythmic drug is CONTRAINDICATED in A. Nifedipine
D. Dinoproston this case? B. Verapamil
E. Quinine. A. Anaprilin (Propranolol) C. Propranolol (Anaprilin)*
B. Ajmaline D. Procainainamide
An ambulance was called to women 28 years old in C. Verapamil E. Ajmaline.
connection with an attack of asthma. After D. Nifedipine
intramuscular injection of ephedrine her condition E. Novocainamide (Procainamide) A patient with fibrillation, who has had bronchial
was improved. However, after 49 minutes the asthma in his anamnesis, should be administered
attack was repeated, ephedrine injected once more A 42-year-old woman has been administered antiarrhythmic drug. Which preparation from the
acted weaker, in the third attack of bronchospasm propranolol for the ischemic heart disease. Yet she suggested group is contraindicated for this patient?
A. Ajimaline D. D Stimulation of central alpha2 E. Propranolol.
B. Verapamil adrenoceptors
C. Novocainamide E. Blockade of central M-cholinoceptors. A 43 year-old male patient is suffering from
D. Nifedipine hypertension. His blood pressure was successfully
E. Propranolol (Anapriline )*. Patient with initial form of hypertension complains controlled by monotherapy with adrenoblocker.
of pain in heart and tachycardia. Which of the With time treatment was complicated with
Doctor prescribed selective alfa1-adrenoceptor following drugs has to be used in the treatment of diarrhoea and impotence. The patient addressed his
blocker for treatment of hypertensive patient. this patient doctor and the drug was changed for another
Which of the following drugs belongs to this A. Phentolamine adrenoblocker. Which of adrenoblockers can cause
group? B. Dihydroergotamine above listed side effects?
A. Metoprolol C. Propranolol (Anaprilin)* A. Metoprolol
B. Phentolamine D. Prazosin B. Salbutamol
C. Cordarone E. Guanethidine(Octadinum). C. Propranolol
D. Tropaphan D. Dobutamine
E. Doxazosine*. Patient having in the past episodes of bronchial E. Prazosin*.
asthma addressed doctor with angina pectoris.
Patient with angina pectoris was treated with drug Doctor decided to treat patient with B- Doctor prescribed selective alfa1-adrenoceptor
decreasing oxygen demands of myocardium. While adrenoblocker. Which of B-adrenoblocker is not blocker for treatment of hypertensive patient.
treatment, patient starts to complain of extremities contraindicated in this case? Which of the following drugs belongs to this
coldness especially fingers. Which of the drug was A. Anapriline(Propranolol) group?
used? B. Atenolol* A. Metoprolol
A. Atenolol C. Oxprenolol B. Phentolamine
B. Propanol (anaprilin)* D. Pindolol C. Cordarone
C. Reserpine E. Nadolol. D. Tropaphan
D. Prazosin E. Prazosin*.
E. Labetalol. For treatment of hypertensive patient doctor
prescribed antiadrenergic drug, first administration A 62-year-old patient, prone to
Patient with Raynaud disease was treated with of which in therapeutic dose caused “first dose bronchoconstriction, addressed his doctor with
phentolamine. What is the mechanism of action of syncope”. Which drug was most probably used? complaints of tachyarrhythmia troubling him
this drug? A. Metoprolol several times a day. The doctor prescribed
A. Blockade of B-adrenoceptors B. Atenolol anaprilin (propranolol). Administration of drug
B. Blockade of alpha adrenoceptors* C. Prazosin* provoked severe attack of bronchospasm. What is
C. Decrease in neurotransmitter amount D. Reserpin the mechanism of this side effect?
A. Stimulation of M1- cholinoceptors C. Phentolamine C. Furosemide
B. Inhibition of alfa-adrenoceptors D. Propranolol (Anaprilin) D. Oxytocin
C. Inhibition of beta2 - adrenoceptors* E. Reserpine. E. Atropine sulfate.
D. Increase in n. vagus tone
E. Stimulation of leukotriene release. The patient in case of angina chest pain regularly Note the non-selective beta-blocker.
takes nitroglycerin, however, recently angina A. Mezatonum
A young woman with 6 months pregnancy visited pectoris complicated by extrasystoles and episodes B. Atenolol
her doctor complaining of periodical slight of bronchospasm. Which of beta - blockers better C. Adrenaline
contractions of uterus. To prevent premature labour to appoint for such a patient? D. Salbutamol
doctor prescribed partusisten (fenoterol). What is A. Timolol E. Propranolol (Anaprilin)*.
the mechanism of action of this drug? B. Propranolol (Anaprilin)
A. Stimulation of beta2 - adrenoceptors* C. Oxprenolol The patient with hypertension was appointed
B. Inhibition of beta2 - adrenoceptors D. Atenolol* cardioselective beta-blocker. Which drug is
C. Stimulation of alfa1 - adrenoceptors E. Nadolol. assigned to the patient?
D. Inhibition of alfa1 - adrenoceptors A. Reserpine
E. Stimulation of M-cholinoceptors. Patients with essential hypertension (stage II) used B. Propranolol (Anaprilin)
one of antihypertensive drugs. After a while blood C. Metoprolol*
The patient with the initial form of hypertension pressure decreased, but the patient began to D. Atropine
complains of pain in the heart and tachycardia. complain of fatigue, sleepiness, apathy. Later on, a E. Adrenaline.
Which of the following drugs should be used in the pain in the stomach occurred, and gastric peptic
treatment of this patient? ulcer was diagnosed. What the drug was used? To patient with a tachyarrhythmia doctor
A. Prazosin A. Reserpine* prescribed a beta-blocker. Enter the drug.
B. Phentolamine B. Verapamil A. Adrenaline
C. Dihydroergotamine C. Captopril B. Propranolol (Anaprilin)*
D. Propranolol (Anaprilin)* D. Dibazol C. Atropine
E. Reserpine. E. Furosemide. D. Isoprenaline
E. Salbutamol.
A doctor prescribed the drug to the patient with A patient with ischemic heart disease was admitted
hypertension. The drug is acting by selective block to cardiology department. To prevent attacks of The patient suffering from angina takes a selective
of alpha 1 - adrenergic receptors. What the drug angina pains drug of beta-1adrenoblockers was beta-adrenoblocker. Enter the drug.
was prescribed? assigned. Name this drug: A. Metoprolol*
A. Metoprolol A. Metoprolol* B. Prazosin
B. Doxazosin* B. Morphine hydrochloride C. Atropine
D. Pilocarpine B. Procainamide hydrochloride A. Quick absorption of drug
E. Propranolol (Anaprilin). C. Trimecaine B. Quick metabolism of anesthetic
D. Anesthezin C. Alkaline medium
E. Tetracaine D. Acidic pH of tissues*
LOCAL AND GENERAL ANESTHETICS (31) E. Neutral medium.
A patient has developed paroxysmal
Novocaine ventricular tachycardia against the background of Surgeon is going to perform the operation under
Dentists commonly practice local anaesthetization cardiac infarction.What antiarrhythmic drug should local anaesthesia. The probable duration of
by applying novocaine solution with 0,1% be chosen to avoid lowering cardiac output? operation is more than 2 hours. In past patient was
adrenalin solution. The added adrenalin induces: A. Lidocaine hydrochloride complaining for ventricular extrasystole. Which
A. Local vasoconstriction B. Procainamide anaesthetic is reasonable to chose for local
B. Local vasodilatation C. Verapamil anaesthesia?
C. Arterial pressure drop D. Propranolol A. Tetracaine(decaine)
D. Decrease in vascular resistance E. Potassium chloride B. Procaine(novocaine)
E. Arterial pressure rise C. Benzocaine(anaesthesine)
To perform conduction anesthesia a patient had D. Trimecaine*
Novocaine acts as an anesthetic by making nerve been administered a drug used in dental surgery. E. Cocaine.
fibers unable to conduct stimulation. What The patient developed the symptoms of poisoning:
mechanism of action regarding membrane’s central nervous system excitation followed by Dentist revealed that his patient is suffering from
permeability to ions does this drug have? paralysis and acute cardiovascular insufficiency hypersensitivity to procaine. Which of the
A. Sodium ion-selective channels blockade (collapse). Additionally there were allergic following agents may be used for local
B. Potassium ion-selective channels blockade reactions (itching, swelling, erythema). Name this anaesthesia?
C. Calcium ion-selective channels blockade drug: A. Benzocaine(anaesthesine)
D. Sodium-potassium pump blockade A. Lidocaine B. Piromecaine
E. Sodium-proton pump blockade B. Suxamethonium chloride C. Dicaine(tetracaine)
C. Thiopental sodium D. Lidocaine*
Lidocaine D. Tubocurarin chloride E. Cocaine.
Before the infiltration anaesthesia a patient had E. Pipecuronium bromide
been tested for sensitivity to novocaine. The Which of the listed drugs is usually added to local
reaction turned out to be positive. Which of the To lance abscess doctor performed local anesthesia anaesthetic to prolong its action and to decrease
below listed drugs can be used for anaesthetization with therapeutic dose of anesthetic, but it was not toxicity?
in this case? sufficient. What was the cause of decrease in A. Noradrenaline
A. Lidocaine anesthetic effect? B. Ephedrine
C. Adrenaline* C. Neostigmine E. Bismuth subnitrate.
D. Reserpine D. Magnesium sulfate (enterally)
E. Atropine. E. Camphor spirit. Athlete has a sharp muscular pain in the back after
training. What the irritating drug is helpful in this
A 25-year-old woman with red and itchy Bed sores appeared in patient on strict bed case?
eczematoid dermatitis visits your office, She had a regiment. What the irritating drug was prescribed A. The turpentine oil*
dental procedure one day earlier with by physician to improve blood circulation in the B. A solution of menthol
administration of a local anesthetic. There were no area of bed sores C. Ammonia
other findings, although she indicated that she had A. Camphor spirit* D. Validol
a history of allergic reactions. Which of the B. Vishnevsky’s ointment E. Chloroform.
following drugs is most likely involved? C. Wilkinson's ointment
A. Procaine* D. A solution of potassium permanganate Gastric lavage with a solution of tannin was
B. Etidocaine E. Talc. performed to the patient with alkaloid poisoning.
C. Bupivacaine What the concentration of tannin solution should
D. Cocaine A child with acute poisoning with datura seeds be used for this purpose?
E. Lidocaine. doctor prescribed oral usage of activated charcoal. A. 0.5%*
Specify the mechanism of its action. B. 0.1%
For the patient with gastroenteritis doctor A. Adsorptions alkaloids on the surface* C. 0,01
prescribed preparation of tannin derivative with B. Insoluble salts formation D. 0,05
anti-inflammatory effect on the basis of the C. Block of sensory receptors of the stomach E. 0,1.
astringent mechanism. Which of the following D. Precipitation of proteins on the surface of
drugs was prescribed? gastric mucous membrane The patient suffers from hemorrhoids. For pain
A. Bismuth subnitrat E. Formation of protective layer of the colloid relief doctor prescribed rectal suppositories with
B. Tannin on the mucous membranes. local anesthetic. Point out anesthetic used in this
C. Xeroform dosage form.
D. Tanalbin* The drug from the group covering agents with A. Cocaine
E. De-nol. antacid effect was prescribed to patients with B. Tetracaine
gastric ulcer. Which of the following medications C. Novocaine
Patient 75 years old suffers from chronic was prescribed? D. Trimekaine
constipation. What the drug of a group of A. Tannin E. Benzocaine (Anestesin)*.
emollients can be assigned to this patient? B. Decoction of oak bark
A. Senna leaf* C. Almagel* Doctor applied paste with benzocaine to reduce the
B. Phenolphtalein D. Infusion of sage leaves itching of the forearm skin. Due to which of the
following properties of benzocaine it is used in soft
dosage form? The anaesthetic used along with halothane to give To make the treatment of burn surface painless
A. Good absorption excellent analgesia and rapid induction is intravenous general anesthetic for short-term
B. Poor solubility* A. Nitrous oxide* anesthesia was injected to patient. Anesthesia
C. The high degree of distribution B. Thiopental developed in 1 minute during anesthesia increase
D. The good solubility C. Desflurane in blood pressure, tachycardia, increase in tone of
E. Inadequate absorption. D. Propofol skeletal muscles have occurred. On the recovery
E. Choroform. from anesthesia patient experienced disorientation
The patient will have surgery on the limbs of more and visual hallucinations. What is the drug entered
than 2 hours duration, which anesthetist plans to To anaesthetize the manipulation related to burn the patient?
hold under local anesthesia. Which of the surface treatment, a patient was intravenously A. Ether for anesthesia
following drugs should be selected for long-term injected a medication for short-acting narcosis. 1 B. Sombrevin
conduction anesthesia? minute later the patient being under anaesthesia C. Ketamine*
A. Novocaine had increased blood pressure, tachycardia, D. Thiopental sodium
B. Cocaine increased tone of sceletal muscles reflexes E. Nitrous oxide.
C. Tetracaine remained. After awakening the patient had
D. Benzocaine desorientation and visual hallucinations. What To do a biopsy of breast tumors in 35 years old
E. Lidocaine*. medication was the patient injected? woman analgesia is required. What type anesthesia
A. Nitrous oxide should be used?
To anaesthetize the manipulation connected with B. Thiopental sodium A. Potentiated anesthesia with muscle
burn surface treatment, a patient was intravenously C. Ketamine* relaxants
injected a medication for short-acting narcosis.1 D. Diethyl ether B. Combined mixed anesthesia
minute later the patient being under anesthesia had E. Sombrevin. C. Neurolept-analgesia*
elevated blood pressure, tachycardia, increased D. Basic anesthesia (Na-hydroxybutyrate)
tone of skeletal muscles; the reflex were reversed. Dental surgeon made tooth extraction in a patient E. Inhalation anesthesia.
After awakening the patient had disorientation and under general anesthesia. Which of the following
the visual hallucination. What medicine was medications can be used in this case for short-term A 28 years old woman addressed the dentist about
injected to the patient? noninhalation general anesthesia? dental prosthetics. Because she is allergic to local
A. Sombrevin A. Ketamine* anesthetics, dental treatment was necessary to carry
B. Thiopental sodium B. Predion out under general anesthesia. Which general
C. Nitrous oxide C. Thiopental sodium anesthetic drug should be used for this purpose,
D. Ketamine* D. Halothane taking into consideration short duration of
E. Diethyl ether. E. Nitrogen oxide. manipulation?
A. Sodium hydroxybutyrate Identify the drug that has sedative, hypnotic, Due to severe pain syndrome a patient has been
B. Ketamine* anesthetic, anti-hypoxic, myorelaxant actions and it prescribed a narcotic analgesic. Specify the
C. Thiopental is used as general anesthetic and hypnotic. prescribed drug:
D. Halothane A. Sodium hydroxybutyrate* A. Morphine
E. Hexobarbital (Hexenalum). B. Hexobarbital B. Analgin (Metamizole)
C. Chloral hydrate C. Nimesulid
To anesthetize the procedure of taking a biopsy in D. Diazepam D. Dimexid
patient a drug that causes a fast anesthesia lasting E. Thiopental sodium. E. Indometacin
about 5 minutes was injected intravenously. During
anesthesia there were involuntary movements, The patient takes disulfiram (teturam). What is the Promedol
slight decrease in blood pressure, short-term indication for use of this drug? A patient with urolithiasis has developed
respiratory arrest, which was quickly restored. A. Psychosis severe pain attacks. For pain shock prevention
What kind of drug was used? B. Preparation for surgery he was administered an antispasmodic
A. Halothane C. Fear and anxiety narcotic analgesic along with atropine.
B. Ketamine D. Neurosis Name this drug:
C. Predion E. Alcoholism*. A. Promedol
D. Propanidid* B. Nalorphine
E. Sodium hydroxybutyrate. OPIOID AND NON-OPIOID ANALGESICS C. Tramadol
(35) D. Ethylmorphine hydrochloride
Generalized tonic-clonic seizures with loss of E. Morphine hydrochloride
consciousness were periodically occurring in a Morphine hydrochloride
patient after an injury; these seizures were After parenteral introduction of a medication a A patient with a severe maxillofacial trauma has
followed by a general depression of central patient fell into a coma. He had Cheyne-Stokes been delivered to the emergency department. What
nervous system. What the drug should be assign to respiration, apparently miotic pupils. The patellar drug should be given this patient to relieve pain
prevent seizures? reflex was preserved. What medication might have shock?
A. Trihexyphenidyl (Cyclodolum) caused the intoxication? A. Promedol
B. Midantan A. Morphine hydrochloride B. Sydnocarb
C. Levodopa B. Aminazine C. Ibuprofen
D. Disilfiram (Teturam) C. Diazepam D. Pantogam
E. Phenobarbital*. D. Analgine E. Mydocalm
E. Phenobarbital
A patient has been diagnosed with transmural it would be rational to combine fentanyl with the Very painful uterine contractions have been
myocardial infarction. What drug should be given following medicine: occurring in a pregnant woman at normal term. To
in order to prevent cardiogenic shock? A. Droperidol relieve the woman’s condition during labour, a
A. Promedol B. Cholosasum doctor decided to use opioid (narcotic) analgesic.
B. Reserpin C. Salbutamol Which of the listed drugs can be used in this case?
C. Octadine D. Pilocarpine A. Trimeperidine (Promedolum)*
D. Phentolamine E. Fraxiparine B. Morphine
E. Analgin C. Metamizole (Analginum)
A patient with myocardium infarction was D. Papaverine
A 4-year-old child was admitted to an admitted to the cardiological department. In order E. Codeine.
orthopaedic department with displaced shin to relieve his pain it was decided to potentiate
fracture. Bone fragments reposition requires action of fentanyl by a certain neuroleptic. What is A 60-year-old male is brought to emergency room.
analgesia. What drug should be chosen? the most suitable neuroleptic for He is comatose and his pupils are constricted.
A. Promedol neuroleptanalgesia? Physician suspects opium overdose. What is the
B. Analgin A. Droperidol best drug to be administered?
C. Morphine hydrochloride B. Aminazine A. Flumazanil
D. Panadol C. Triftazine B. Calcium carbonate
E. - D. Haloperidol C. Sodium bicarbonate
E. Sulpiride D. Naloxone*
A patient with myocardium infarction was E. Atropine.
delivered to the resuscitation department. What Naloxone
drug should be injected to the patient for A patient with a malignant neoplasm on the upper A young man was taken to emergency department
prophylaxis of pain shock? jaw had been administered morphine hydrochloride with numerous traumas of chest and head.
A. Promedol for analgesia. The injection induced respiratory Anesthesiologist proposed to inject morphine to
B. Analgin depression, pupil constriction, cyanosis, relieve patient’s condition, but traumatologist
C. Paracetamol hypothermia, loss of consciousness. What antidote rejected proposition of his colleague. Why
D. Celecoxib must be used? morphine is contraindicated in this case?
E. Naloxone A. Naloxone A. It increases intracranial pressure*
B. Atropine sulfate B. It stimulates vagal center
Droperidol C. Droperidol C. It decreases intraocular pressure
A patient was admitted to a surgical department for D. Promedol D. It causes miosis
an operation. He has to undergo E. Adrenalin hydrochloride E. It depresses center of cough reflex.
neuroleptanalgesia. To achieve neuroleptanalgesia
It is known that morphine can cause a shift of A patient with acute morphine poisoning was The patient with inoperative lung cancer
blood from pulmonary to systemic circuit. It results delivered to a hospital. What specific narcotic metastasizing in the spine was suffering from
in lowering of resistance in pulmonary vascular antagonist should be chosen in this case? severe pain. To relieve patient's condition analgesic
bed. In which case in may be used? A. Unithiol is required. Which drug should be administered?
A. Bronchitis B. Paracetamol A. Analgine
B. Pneumonia C. Digoxin B. Codeine
C. Pulmonary oedema* D. Methacin C. Paracetamol
D. Bronchiectatic disease E. Naloxone*. D. Morphine*
E. Emphysema. E. Papaverine.
A young man with an addiction to opioid drugs
Patient in unconscious state was admitted to the denies drug dependence. Introduction of one of On arrival of ambulance a 25 years old patient was
emergency room. Skin is cold, pupils are pinpoint these drugs will help to prove drug dependence. unconscious and 15 vials of morphine solution
like, breathing is heavy, with cycles of the Cheyne- Point out this drug. were found near him patient had periodic
Stokes type, blood pressure is decreased, urinary A. Morphine breathing, slowed heart rate, suppressed reflexes
bladder is overloaded. Poisoning with what B. Codeine except for the knee, and narrowed pupils he was
substance is the most likely? C. Trimeperidine (Promedol) taken to the emergency department. What measure
A. Neosigmin (Proserinum) D. Fentanyl should be applied in the first place?
B. Sedatives E. Pentazocine*. A. Gastric lavage*
C. M-cholinergic antagonists B. Diuretic
D. Nor-narcotic analgesics A 30 years old patient was delivered to emergency C. Antiarrhythmic drug
E. Narcotic analgesics*. department with a fractured thigh bone after a car D. Enema
accident. The patient had dramatically reduced E. Induction of vomiting.
Examination of a patient revealed extremely blood pressure to 70/40 mm Hg, weak pulse,
myotic pupils, sleepiness, infrequent Chain-Stoke's enhanced pain response to touch to damaged area. A child of 4 years is hospitalized to the orthopedic
respiration. urinary retention, slowing-down of What should be used for prevention of traumatic department with a fractured ankle with
heart rate, enhancement of spinal reflexes. What shock in patient? displacement. The reposition of bone fragments
substance caused the poisoning? A. Metamizol (Analgin) required analgesia. What medication should be
A. Atropine B. Papaverine used?
B. Phosphacole C. Morphine* A. Analgin
C. Caffeine D. Paracetamol B. Promedol*
D. Morphine* E. Pentazocine. C. Morphine hydrochloride
E. Barbital. D. Panadol
E. Omnopon.
inflammatory agent, selectively inhibiting COX–2. D. Antioxidative effect
Choose a specific antagonist of narcotic analgesics. Point out this drug. E. Antimicrobial effect.
A. Caffeine-sodium benzoate A. Paracetamole
B. Ethimizol B. Diclophenac-sodium The usage of acetylsalicylic acid resulted in
C. Bemegride C. Celecoxib* occurrence of epigastric pain in patient. On
D. Strychnine D. Metamisole (Analginum) examination of the patient gastric ulcer was
E. Naloxone*. E. Indomethacine. revealed. What the mechanism is responsible for
ulcerogenity of this drug?
A patient with symptoms of renal colic is admitted In complex treatment of a patient with angina A. Stimulation of pepsin secretion
to the urology department. Which of these drugs pectoris non- narcotic analgesic exerting B. Inhibition of prostaglandin synthesis*
should be advised to provide him first aid? antiplatelet activity was included. Point out this C. Increase in bile production and its reflux
A. Morphine agent. D. Spasm of vessels
B. Trimeperidine (Promedol)* A. Meperidine E. Spasm of gastric smooth muscles.
C. Pentazocine B. Morphine
D. Fentanyl C. Tramadol Doctor visited a 5-year-old child with fever and
E. Tramadol. D. Metamizole complaints of headache. Doctor suspected the
E. Aspirin*. onset of viral infection. To relieve patient’s
A patient with dislocation of shoulder was condition doctor prescribed non-opioid analgesic,
admitted to traumatologic department. The Due to inhibition of COX aspirin exerts all the which additionally exerts antipyretic effect but
reduction of dislocation should be performed under following effects except lacks anti-inflammatory action. Point out the
neuroleptanalgesia. Chose the opioid analgesic A. Anti-inflammatory prescribed drug
used for this purpose. B. Antiplatelet A. Metamizole (Analginum)
A. Naloxone C. Increase in warfarin efficacy* B. Celecoxib
B. Cordiamin D. Antipyretic C. Aspirin
C. Morphine E. Analgesic. D. Acetaminophen (Paracetamol)*
D. Ethimizol E. Diclophenac-sodium.
E. Fentanyl*. A patient with toothache was relieving his pain
with help of metamizole (analginum). Point out After the usage of acetylsalicylic acid a patient
A 67 year-old male patient suffering from arthritis another useful effect of this drug that contributes to developed epigastric pain because of exacerbation
was treating himself with acetyl salicylic acid the improvement of patient’s condition of his ulcer. What is the mechanism of this drug
(aspirin). In 2 weeks patient addressed doctor A. Sedative effect ulcerogenity?
complaining of pain in epigastric area. On B. Anti-inflammatory effect* A. Stimulation of pepsin secretion
examination of the patient doctor prescribed anti- C. AntIplatelet effect B. Antiprostagladinic effect*
C. Cholagogic effect Haloperidol
D. Spasm of vessels NEUROLEPTICS, TRANQUILLIZERS, AND A 49-year-old patient has psychotic manifestations
E. Immunodepressive effect. SEDATIVE DRUGS (31) in form of psychomotor agitation, auditory and
visual hallucinations. What drug is indicated in the
Point out non- narcotic analgesic, which exerts Aminazine described case?
antiplatelet activity. A schizophrenic patient has been prescribed A. Haloperidol
A. Meperidine aminazine. What pharmacodynamic action of this B. Sodium bromide
B. Morphine drug justifies its prescription in this case? C. Diazepam
C. Tramadol A. Antipsychotic D. Valerian tincture
D. Metamizole B. Antiemetic E. Chlordiazepoxide
E. Aspirin*. C. Hypothermic
D. Muscle relaxant Tranquilizers
A 2-year-old child is often catching a cold with E. Hypotensive To perform a scheduled surgery on the upper jaw, a
fever. Which antipyretic drug is safer at this age surgeon decided to apply ataralgesia. What
than acetylsalicylic acid? An alcoholic has alcoholic psychosis medications are used for this manipulation?
A. Analgin with evident psychomotor agitation. What A. Tranquilizers
B. Indomethacin neuroleptic drug should be administered for B. General anesthetics
C. Paracetamol* emergency aid? C. Narcotic analgesics
D. Phenylbutazone A. Aminazine D. Non-narcotic analgesics
E. Phenylbutasone. B. Diazepam E. Sedatives
C. Sodium bromide
A 7 year child have caught cold, he was shivering, D. Reserpine Diazepam
fever up to 39,2 ° C, his respiratory rate - 45 per E. Halothane With the purpose of analgesia, a narcotic analgesic
minute, pulse - 105 beats per minute. The doctor has been used with a benzodiazepine drug. What
diagnosed an acute viral infection, and appointed a A schizophrenic patient has been prescribed drug has been used to potentiate analgesia?
baby aspirin. By means of what mechanism the aminazine. What pharmacodynamic action of this A. Diazepam
drug realized its antipyretic effect in this case? drug justifies its prescription in this case? B. Chlorprothixene
A. The overall calming effect A. Antipsychotic C. Triftazin
B. Slowing down heartbeat B. Hypotensive D. Carbamazepine
C. Increase in heat output* C. Antiemetic E. Imizinum
D. Slowing of the respiratory rhythm D. Hypothermic
E. Reduction of heat production. E. Muscle relaxant
A dental patient was prescribed a psychosedative A. Interaction with benzodiazepine treatment with chlorpromazine (aminazinum). She
for his fear of pain. What drug would be the most receptors was troubled with tremor and disturbances of
effective in this case? B. β-adrenoreceptor block movements. What is the mechanism of this side
A. Diazepam C. M-cholinoreceptor activation effect?
B. Aminazine D. Dopamine receptor block A. Activation of hyppocampus
C. Lithium carbonate E. α-adrenoreceptor block B. Inhibition of reticular formation (alfa1 -
D. Valerian tincture adrenoceptors)
E. Sodium bromide Bromine salts C. Inhibition of neostriatum (D2 receptors)*
For two weeks a woman has been taking the D. Inhibition of hypothalamus
A 45-year-old female patient has neurosis with mixture for neurasthenia, which was prescribed by E. Inhibition of hyppocampus.
irritability, insomnia, amotivational anxiety. What a neurologist. Her general state slightly improved
tranquilizer will be able to eliminate all symptoms but shortly she started complainin go frhinitis, A 50 years old patient with chronic alcoholism
of the disease? conjunctivitis, skinrashes, fatigue, and memory became aggressive. To abolish aggression,
A. Diazepam impairment. What group of drugs can have such a chlorpromazine was administered intramuscularly.
B. Paracetamol side effect? The patient’s attempt to rise soon after an injection
C. Piracetam A. Bromine salts resulted in loss of consciousness. What is the
D. Caffeine-sodium benzoate B. Valerian preparations probable cause of such complication?
E. Levodopa C. Motherwort preparations A. Blockade of reticular formation
D. Adaptogens B. Disturbance of coronary blood circulation
A male patient waiting for tooth extraction has E. Hop preparations C. Blockade of alfa- adrenoceptors*
developed a strong sense of anxiety. Which drug D. Inhibition of hypothamus
should be given to him in order to relieve him of A patient was treated for psychosis for 2 weeks. E. Blockade of M- cholinoceptors.
this discomfort? The psychic patient's condition improved but
A. Diazepam rigidity, tremor, and hypokinesia developed. Which After emotional stress, patient was permanently in
B. Aminazine of the drugs can cause such complications? a condition of nervous tension and had poor sleep.
C. Analgin A. Diazepam Doctor prescribed him diazepam. Which of the
D. Aethimizolum B. Chlorpromazine (Aminazine)* listed effects of this drug is responsible for its
E. Carbamazepine C. Amytriptiline clinical use?
D. Piracetam A. Hypotensive effect
Before a tooth extraction a 48-year-old female E. Chlordiazepoxide. B. Analgesic effect
patient received an injection of diazepam. C. Anticonvulsant
Anxiolytic effect of this drug can be explained by: A 60 years old woman addressed her doctor D. Tranquilizing*
complaining of side effects which appeared while E. Anti psychotic.
A patient who has been treated in a neural clinic C. Reserpine
Point out the neuroleptic that is most commonly and has been taking a sedative for a long time got D. Tincture of valerian
used in neuroleptanalgesia. the following complication: cough, rhinitis, E. Amitriptyline.
A. Chlorpromazine epiphora. What drug caused these disturbances?
B. Droperidol* A. Sodium bromide* The patient with schizophrenia was appointed
C. Diazepam B. Valerian chlorpromazine. What the pharmacological effect
D. Chlorprotixene C. Phenazepam is the basis for assigning it to the patient?
E. Sulpiride. D. Reserpine A. Hypothermic
E. Diazepam. B. Hypotensive
A patient on a particular psychotropic medication C. Antipsychotic*
complains of severe dizziness. His blood pressure Select the most appropriate drug for the systematic D. Antiemetic
in a supine position is 115/80 mm hg on standing it treatment of a patient with severe productive E. Myorelaxant.
drops to 82/50 mm hg. Which of the following psychotic symptoms (delirium and hallucinations).
drugs is most likely responsible for these A. Trifluoperazine (Triftazin)* A patient addressed a doctor complaining of
symptoms? B. Clozapine anxiety, fear, uneasiness, inner tension. Which
A. Carbamazepine C. Medazepam drug should be assigned?
B. Chlorprozamine* D. Amitriptyline A. Tincture of valerian
C. Chlordiazepoxide E. Buspirone. B. Pavlov Medicine
D. Cortisone C. Diazepam*
E. Ibuprofen. Neuroleptics have the properties of eliviating D. Tincture of leonuri
delusions, hallucinations, reducing of aggression E. Etaminal sodium.
A patient has been taking a mixture prescribed by and psychomotor agitation. This action is called:
neuropathologist for neurasthenia for two weeks. A. Antipsychotic* On examination of the patient, unstable blood
The patient feels better but has developed coryza, B. Hypodynamic pressure, irritability, emotional imbalance, poor
conjunctivitis, rash, inertia, decrease of memory. C. Antihysteric sleep were revealed, and doctor has prescribed
She is diagnosed with bromizm. What should be D. Anxiolytic chlordiazepoxide (elenium). Which of the
prescribed to decrease the E. Antineurotic. following statements is ground for prescription
A. Glucose solution 5% elenium in this case?
B. Polyglucin A 40 years patient delivered to psychiatric clinic in A. Psychosis
C. Asparcam a state of agitation, aggression, delusions. What B. Cardiopathy
D. Natrium chloride* medication should be administered? C. Hypertention
E. Kalium chloride. A. Sodium bromide D. Coronary artery disease
B. Chlorpromazine* E. Neurosis*.
ineffectiveness of drugs previously designated
The presence of which syndrome is the basis for gave the grounds for doctor to use Sodium valproate
the appointment of diazepam? neuroleptanalgesia. Which antipsychotic agent is A patient with epilepsy and depressive reaction has
A. Lingering paralysis used for this type of anesthesia? been administered a drug that reduced epilepsy
B. Inadequate perceptions (hallucinations) A. Chlorpromazine manifestations and improved the patient’s psychic
C. Euphoric state of pathological origin B. Triftazinum condition.
D. Anxiety* C. Metaperazine A. Sodium valproate
E. Pathological thought disorder. D. Droperidol* B. Ethosuxemide
E. Thioridazine. C. Amitriptyline
Patient 38 years old suffering from schizophrenia D. Phenytoin
for a long time is on the psychotropic drugs. He A patient with schizophrenia is taking typical E. Phenobarbital
addressed the doctor with complaints of neuroleptic. Which of the effects is the basis for its
disturbances of motor coordination, hand tremor, application in schizophrenia? A 56-year-old woman is registrated with a
drowsiness. Which group of drugs can cause such a A. Antipsychotic* psychoneurologic dispensary due to her suffering
condition? B. Antiemetic from epilepsy, specifically, minor attacks (pti
A. Antipsychotics* C. Hypothermic mal).What drug is the most efficient in this case?
B. Antidepressants D. Myorelaxant A. Sodium valproate
C. Analgesics E. Hypotensive. B. Trihexyphenidyl
D. Adaptogens C. Phenytoin
E. Psychomotor stimulants. A patient addressed the doctor with complaints of D. Levodopa
poor health, depression, skin rashes. An interview E. Phenobarbital
A single parenteral administration of the drug in revealed that the patient within three weeks was
the patient with schizophrenia caused severe taking sedative medicine, prescribed by a A 56-year-old female patient is regi-strated in a
hypotension. To which of these groups does drug neurologist. With the admission of which drugs psychoneurologic dispensary due to her suffering
belong? complaints of patient were associated? from epilepsy, specifically, minor attacks (pti mal).
A. Nootropics A. Bromides* What drug is most efficient in this case?
B. Antipsychotics* B. Lithium A. Sodium valproate
C. Tranquilizers C. Tranquilizers B. Phenobarbital
D. Antidepressants D. Antipsychotics C. Trihexyphenidyl
E. CNS stimulants. E. Preparations of valerianae. D. Levodopa
E. Phenytoin
A patient with acute myocardial infarction HYPNOTIC, ANTIEPILEPTIC, AND
experienced persistent chest pain. The ANTIPARKINSONISM DRUGS (22) Nitrazepam
A patient who suffers from insomnia caused by Levodopa usage of phenobarbital as a hypnotic. What the
emotional disorder was prescribed a hypnotic drug A patient suffering from parkinsonism has been drug should be used instead of phenobarbital?
with tranquillizing effect. What hypnotic was prescribed levodopa, which resulted in rapid A. Sodium hydroxybutyrate
prescribed? improvement of the patient’s condition. What B. Bromizoval
A. Nitrazepam mechanism of action is characteristic of this drug? C. Chloral hydrate
B. Phenobarbital A. Stimulation of dopamine synthesis D. Nitrazepam*
C. Chloral hydrate B. Block of muscarinic receptors E. Noxiron.
D. Sodium ethaminal C. Stimulation of dopamine receptors
E. Bromisoval D. Anticholinesterase action Due to emotional stress a man permanently is in a
E. Stimulation of muscarinic receptors state of nervous tension and has poor sleep. The
Gamma-amino butyric acid doctor gave him a diazepam. Which of the
A patient presents with dysfunction of cerebral A 5 years old child from time to time have seizures following effects of diazepam is the base for its
cortex accompanied by epileptic seizures. He has with loss of consciousness. The doctor diagnosed clinical application in this case?
been administered a biogenic amine synthetized epilepsy (large seizures). Which drug is advisable A. Hypotensive
from glutamate and responsible for central to appoint a patient? B. Anxiolytic (Tranquilizing)*
inhibition. What substance is it? A. Phenobarbital* C. Analgesic
A. Gamma-amino butyric acid B. Benactizin(Amizulum) D. Anticonvulsant
B. Serotonin C. Trihexyphenidyl (Cyclodolum) E. Antidepressant.
C. Dopamine D. Ethosuximide
D. Acetylcholine E. Levodopa. A patient suffering from insomnia addressed his
E. Histamine doctor. The main problem was in the difficulties
A man aged 68 with trembling hands and with sleep onset. Doctor prescribed zolpidem.
A patient presents with dysfunction of the cerebral incoordination diagnosed with Parkinson's disease. Which of the following characteristics is true for
cortex accompanied by epileptic seizures. He has Which drug is advisable to appoint a patient? this agent?
been administered a biogenic amine synthetized A. Phenytoin A. It belongs to benzodiazepine derivatives
from glutamate and responsible for central B. Finlepsin B. It belongs to barbiturates
inhibition. What substance is it? C. Ethosuximide C. It has the ability to cause quick onset of
A. γ-aminobutyric acid D. Phenobarbital sleep*
B. Serotonin E. Trihexyphenidyl (Cyclodolum)*. D. It’s poorly absorbed from GIT
C. Dopamine E. It’s poorly bind to protein.
D. Acetylcholine 40 years woman addressed doctor with complaints
E. Histamine of psychic disturbances appeared with long-term
A 46- year-old patient with a history of epilepsy main causes of Parkinson’s disease. Point out the B. Carbamazepine
(grand mal) developed status epilepticus. Choose drug used for replacement therapy of this disease C. Valproic acid
the drug to arrest convulsions. A. Atropine D. Ethosuximide
A. Sodium valproate B. Dopamine E. Phenytoin*.
B. Diazepam* C. Levodopa*
C. Levodopa D. Carbidopa A woman, which is on phenytoin, wants to
D. Chlorpromazine E. Amantadine. conceive. What advice you can give to her about
E. Droperidol. drug?
The patient was taken the mixture prescribed by A. Increase the dose of phenytoin
An antiepileptic drug, which prevents metabolism neuropathologist for neurasthenia for 2 weeks. B. Change to valproic acid
of GABA, was included in complex therapy of Patient felt better but developed coryza, C. Addition of valproic acid
patient with epilepsy (grand mal). Point out this conjunctivitis, rash, inertia, decrease of memory. D. Change to phenobarbitone*
drug Bromizm was diagnosed. What should be E. Decrease the dose of phenobarbitone.
A. Phenytoin prescribed to decrease symptoms?
B. Phenobarbitone A. Glucose solution 5% An old epileptic patient on phenytoin is having
C. Carbamazepine B. Calcium chloride status eplilepticus. What will you give to this
D. Ethosuximide C. Natrium chloride* patient in emergency?
E. Sodium valproate*. D. Polyglucin A. Phenytoin
E. Asparcam. B. Carbamazepine
Patient with phenobarbital poisoning was presented C. Diazepam*
to emergency department. To promote kidney The antiepileptic drug which causes nystagmus, D. Valproic acid
excretion of poison sodium bicarbonate was ataxia and gum hypertrophy is E. Primidone.
administered intravenously. What mechanism is A. Phenytoin*
responsible for an increase in barbiturate B. Phenobarbitone Disulfiram is widely used in medical practice to
excretion? C. Carbamazepine prevent alcocholism. It inhibits aldehyde
A. Increase in polarity of barbiturate* D. Ethosuximide dehydrogenase. Increased level of what metabolite
B. Increase in fat solubility of barbiturate E. Valproic acid. causes aversion to alcochol?
C. Increase in barbiturate reabsorption A. Propionic aldehyde
D. Increase in rate of glomerular filtration A 16 years boy is a known as patient suffering B. Methanol
E. Decrease in protein binding. from epilepsy. Following several years of a drug C. Ethanol
therapy, gingival hyperplasia developed. For which D. Acetaldehyde*
Deficiency in dopamine production by neurons of drug this side effect is characteristic? E. Malonyc aldehyde.
strio-pallidal system is considered to be one of the A. Alprazolam
PSYCHOMOTOR STIMULANTS, compounds in synapses can be increased by means B. Amytriptyline*
ANTIDEPRESSANTS, NOOTROPIC DRUGS, of antidepressants that inhibit the activity of the C. Sydnocarbum
AND ANALEPTICS (38) following enzyme: D. Ethymisolum
A. Monoamine oxidase E. Caffeine.
Ammonia spirit solution B. Diamine oxidase
A man who has been staying in a stuffy room for a C. L-amino acid oxidase A patient had a stroke. Which of the listed drugs is
long time lost consciousness. He regained D. D-amino acid oxidase necessary to include in the complex therapy in
consciousness after inhalation of ammonia spirit E. Phenylalanine-4-monooxigenase order to improve mental performance?
vapour. This substance’s effect is connected with A. Caffeine
direct influence upon the following structures: A patient with poisoning caused by carbon B. Piracetam*
A. Receptors of upper airways monoxide was administered directly acting C. Diazepam
B. Vasculomotor centre analeptic drug. What medicine was used? D. Phenazepam
C. Respiratory centre A. Codeine E. Amitryptiline.
D. Resistive vessels B. Ephedrine
E. Capacitive vessels C. Caffeine* What is the main mechanism by which
D. Atropine sulfate amitriptyline increases amount of catecholamines
Monoamine oxidase inhibitors E. Lobeline. in CNS synapses?
A 45-year-old woman has addressed a doctor with A. Increase in catecholamines release from
complaints of rapid mood swings, tearfulness, A premature newborn was apnoic. Directly acting presynaptic membrane
apathy. Antidepressants prescribed for her analeptic drug was given to restore breathing. What B. Increase in catecholamines synthesis in
treatment are monoamine oxidase inhibitors. These medicine was most probably administered to the presynaptic membrane
drugs have a certain effect on catecholamines, patient? C. Prevention of catecholamines degradation
which results in their medicinal action. Name this A. Ethymisolum* in the synapse
effect: B. Phenylephrine (Mesatonum) D. Inhibition of neuronal re uptake of
A. Increased concentration C. Adrenaline hydrochloride catecholamines*
B. Neutralization activation D. Atropine sulfate E. Inhibition of MAO.
C. Inhibition of back transfer E. Lobeline hydrochloride.
D. Increased deaminization Analeptical directly acting drug was given to the
E. Decreased concentration A psychiatrist was invited to a patient who made patient for restoration of breathing after poisoning
an attempt to commit suicide. Doctor diagnosed with carbon monoxide. What medicine was
Depressions and emotional disorders result from endogenous depression. Choose the most suitable prescribed to the patient?
noradrenaline, serotonin and other biogenic amines drug for treatment of this patient. A. Pentamin
deficiency in brain. Concentration of these A. Nootropil (piracetam) B. Mesaton
C. Caffeine* C. Tolerance Doctor recommended taking caffeine the patient
D. Atropine sulphate D. Summation with pathological sleepiness.Which of the
E. Lobeline hydrochloride. E. Potentation*. following mechanisms of action provides its
therapeutic effect in this disease?
Which of the following characteristics of Pharmacological effects of antidepressants are A. The concentration and increased inhibition
amphetamines is most likely to be responsible for connected with inhibitionof an enzyme catalyzing in the cerebral cortex
increasing blood pressure? biogenic amines noradrenaline and serotonine in B. Increased synthesis of dopamine
A. Indirect release of endogenous the mitochondrions of cerebral neurons. What C. Enhancement the processes of excitation in
catecholamines* enzyme participates in this process? the brain*
B. Inhibition of catecholamines metabolism A. Lyase D. Inhibition of the midbrain reticular
C. Metabolism to false neuro-chemical B. Monoamine oxidase* formation
transmitters C. Transaminase E. The weakening of the excitation in the
D. Potent alpha1 adrenergic transmitters D. Peptidase brain.
E. Potent beta-2 adrenergic agonist. E. Decarboxylase.
Point out preferable analeptic in acute barbiturate
The CNS stimulation produced by A psychiatrist was called for the woman who tried poisoning
methylxanthines, such as caffeine, is most likely to commit suicide on examination the state of A. Cordiaminum
due to the antagonism of one of the following endogenous depression was diagnosed. Which drug B. Caffeine
recertors: is most efficient for treatment of this patient? C. Camphor
A. Glycine receptors A. Piracetam (Nootropil) D. Bemegride*
B. Adenosine receptors* B. Amitriptyline* E. Aethimizolum.
C. Glutamate receptors C. Sydnokarb
D. GABA receptors D. Aethimizolum The patient had a stroke. Which of the following
E. Cholinergic muscarinic receptors. E. Caffeine. drugs should be included in the combined therapy
to improve the metabolism of the brain?
A patient who has been treated with diazepam on Which agent will you choose for increase in mental A. Piracetam*
account of neurosis complains of toothache. Doctor performance and decrease of tiredness? B. Caffeine
administered him an analgetic, but its dose was A. Buspirone C. Diazepam
lower then average therapeutic dose. What B. Nialamide D. Phenazepam
phenomenon did the doctor take into account while C. Sidnokarb E. Amitriptyline.
prescribing the patient an underdose? D. Imipramine
A. Drug dependence E. Caffeine*.
B. Cumulation
Which the drug from the group of psychomotor D. Caffeine-sodium benzoate* A. After 9-15 days*
stimulants can cause the development of drug E. Ketotifen. B. After 30-60 minutes
addiction? C. 1-3 days
A. Amphetamin (Phenaminum)* A patient admitted to the neurological department D. After 30-45 days
B. Meridil complains of memory loss and cognitive disability E. After 3-6 hours.
C. Sidnokarb after trauma of the head. What kind of medication
D. Bemithylum is needed to improve the metabolism of the brain? Doctor recommended taking caffeine the patient
E. Caffeine. A. Caffeine with pathological sleepiness. Which of the
B. Metamizole (Analginum) following mechanisms of action provides its
Which of the following effects of piracetam is used C. Piracetam (nootropil)* therapeutic effect in this disease?
for chronic vascular disorders of the brain? D. Sidnokarb A. Enhancement the processes of excitation in
A. Anticonvulsant activity E. Meridil. the brain*
B. Improvement of mental performance* B. Increased synthesis of dopamine
C. Antiwithdrawal effect A psychiatrist was called for the woman who tried C. The concentration and increased inhibition
D. Improvement of physical activity to commit suicide on examination the state of in the cerebral cortex
E. Antipsychotic effect. endogenous depression was diagnosed. Which drug D. Inhibition of the midbrain reticular
is most efficient for treatment of this patient? formation
The patient complains of fatigue, general A. Piracetam (Nootropil) E. The weakening of the excitation in the
weakness, decrease in visual acuty, low blood B. Amitriptyline* brain.
pressure. Which drug is helpful in this case? C. Sydnokarbum
A. Liquid extract of Eleutherococcus* D. Aethimizolum Point out preferable analeptic in acute barbiturate
B. Phenibut E. Caffeine. poisoning
C. Piracetam A. Caffeine
D. Dry extract of Valeriana Which agent will you choose for increase in mental B. Bemegride*
E. Amitriptyline. performance and decrease of tiredness? C. Camphor
A. Nialamide D. Cordiaminum
The primary goal of medical aid in vascular B. Diazepam E. Aethimizolum.
collapse is an increase in blood pressure. Which of C. Imipramine
the following medications can be used for this D. Buspirone The patient had a stroke. Which of the following
purpose? E. Caffeine*. drugs should be included in the combined therapy
A. Aethimizolum to improve the metabolism of the brain?
B. Diphenhydramine When should you expect for a specific therapeutic A. Caffeine
C. Bemegride effect of pantocrinum in its systematic application? B. Diazepam
C. Piracetam* Among the antidepressant drugs there are those A. It causes tachycardia, increases in oxygen
D. Phenazepam that inhibit neuronal norepinephrine reuptake. demand of the heart*
E. Amitriptyline. Point out such drug. B. It slows impulse conduction in the heart
A. Fluoxetine C. It weakens the force of heart contractions
The primary goal of medical aid in vascular B. Nialamide D. It causes a narrowing of the coronary
collapse is an increase in blood pressure. Which of C. Chlorprothixene vessels
the following medications can be used for this D. Chlorpromazine E. It decreases automaticity of the heart.
purpose? E. Maprotilime*.
A. Diphenhydramine What is the mechanism of action of antidepressant
B. Bemegride Mark pharmacological effect, limiting the use of pirlindol (pirazidol)?
C. Caffeine-sodium benzoate* strychnine as analeptic. A. The reversible inhibition of MAO*
D. Aethimizolum A. Stiffness in skeletal muscle, convulsions* B. Irreversible inhibition of MAO
E. Ketotifen. B. Hypercatharsis C. Selective inhibition of neuronal capture of
C. Increased metabolism catecholamines
A patient admitted to the neurological department D. Strengthening of sight and hearing D. Non-selective inhibition of neuronal capture
complains of memory loss and cognitive disability E. Impact on hematopoiesis. catecholamines
after trauma of the head. What kind of medication E. Inhibitor of vesicular capture
is needed to improve the metabolism of the brain? What drug from the group of analeptics is catecholamines.
A. Piracetam (nootropil)* preferable to use in newborn asphyxia?
B. Metamizole (Analginum) A. Caffeine A patient with symptoms of mental depression in
C. Caffeine B. Aethimizolum* the drug treatment did not keep the doctor
D. Sydnokarbum C. Camphor recommended a diet that caused the rise in blood
E. Meridil. D. Cordiaminum pressure. Which group of antidepressants requires
E. Corazolum. compliance in this condition?
Which of the following tonics is derived from A. Tricyclic antidepressant
animal products? A 19 years old boy addressed doctor complaining B. Nonselective MAO inhibitors*
A. Tincture ginseng of discomfort in the heart area. Doctor asked him C. Selective inhibitors of catecholamines
B. Pantocrine* about smoking and other bad habits. Patient denied reuptake
C. Saparalum all of mentioned bad habits, and sad he likes coffee D. Lithium preparations
D. Ecdistenum and uses up to 8 glasses of strong drink a day. Why E. Nonselective inhibitors of neuronal
E. Securininum. coffee containing caffeine can worsen heart reuptake of catecholamines.
function?
HORMONAL DRUGS, THEIR B. Humulin (Human insulin) D. Somatotropin
ANTAGONISTS AND OTHER RELATED C. Parathyreoidinum E. Noradrenaline
AGENTS (39) D. Thyrocalcitonin
E. Prednisolone A patient has been suffering from diabetes mellitus
Oxytocin for 5 years. As a result of not keeping to a diet the
A 26-year-old woman at 40 weeks’ gestation was A patient with autoimmune thyroiditis has been patient passed into a comatose state. Emergency
admitted to the maternity ward. Examination prescribed a peptide hormonal agent. Specify this doctor injected him glucose. The patient’s state got
revealed that the cervix was open, but uterine agent: better. What is the most probable type of coma in
contractions were absent. The doctor gave her a A. L-thyroxin this case?
hormonal drug to induce labor. Specify this drug: B. Trimethoprim A. Hypoglycemic
A. Oxytocin C. Triamcinolone B. Acidotic
B. Hydrocortisone D. Triquilar C. Hyperglycemic
C. Estrone E. Tamoxifen D. Hepatic
D. Testosterone E. Hypothyreoid
E. ACTH Calcitonin
Parodontitis is treated with calcium preparations A patient suffering from non-insulin-dependent
Parathyroid hormone and a hormone that stimulates tooth mineralization diabetes mellitus was prescribed glibenclamid
Following thyroid surgery, a 47-year-old female and inhibits tissue resorption. What hormone is it? internally. What is the mechanism of its
patient had fibrillary twitching of muscles in the A. Calcitonin hypoglycemic action?
arms, legs and face. These disorders can be treated B. Parathormone A. It stimulates generation of endogenous
by the introduction of the following hormone: C. Adrenalin insulin by beta cells
A. Parathyroid hormone D. Aldosterone B. It inhibits gluconeogenesis in liver
B. Triiodothyronine E. Thyroxine C. It intensifies utilization of glucose by
C. Thyrotropin peripheral tissues
D. Thyroxine Insulin D. It inhibits glucose absorption in the bowels
E. Thyroid-stimulating hormone A nurse accidentally injected a nearly double dose E. It inhibits alpha glucosidase and
of insulin to a patient with diabetes mellitus. The polysaccharide breakdown
L-thyroxin patient lapsed into a hypoglycemic coma. What
Prolonged treatment of hypothyroidism drug should be injected in order to help him out of A patient is in the state of hypoglycemic coma.
has caused general dystrophy, dental caries, coma? What hormone can cause this condition if
tachycardia, tremor of extremities. What A. Glucose overdosed?
drug is the cause of these side effects? B. Lidase A. Insulin
A. L-thyroxin C. Insulin B. Progesterone
C. Cortisol A patient has a systemic inflammatory lesion of B. Aldosterone*
D. Somatotropin connective tissue. Which antiinflammatory drug C. Adrenalin
E. Corticotropin will reduce all the inflammatory phases? D. Vasopressin
A. Prednisolone E. Auricular sodiumuretic factor.
Glibenclamide B. Contrycal
A 56-year-old man complains of thirst and frequent C. Phenylbutazone A patient with infectious mononucleosis had been
urination. The endocrinologist diagnosed this D. Indomethacin taking glucocorticoids for two weeks. He was
patient with diabetes melli- tus and prescribed him E. Diclofenac sodium brought into remission, but he fell ill with acute
glibenclamide. What mechanism of action does attack of chronic tonsillitis. What action of
this drug have? A patient with periodontitis has been administered glucocorticoids caused this complication?
A. Stimulation of β-cells of islets of a glucocorticoid drug in form of an ointment. A. Antiallergic
Langerhans Specify this ointment: B. Immunosuppressive*
B. Facilitates glucose uptake by the tissues A. Prednisolone C. Anti-inflammatory
C. Facilitates glucose transport through cell B. Tetracycline D. Antishock
membranes C. Decamine E. Antitoxic.
D. Suppression of α-cells of islets of D. Ampicillin
Langerhans E. Erythromycin A patient had been taking glucocorticoids for a
E. Inhibits glucose absorption in the intestine long time. When the preparation was withdrawn he
A patient with rheumatoid arthritis has been given developed the symptoms of disease aggravation,
A patient with diabetes mellitus complicated by hydrocortisone for a long time. He has developed decreased blood pressure and weakness. What is
angiopathy has been recommended a drug which is hyperglycemia, polyuria, glycosuria, thirst. These the reason of this condition?
a sulphonyl urease derivate of the second complications of treatment result from the A. Appearance of adrenal insufficiency*
generation. It improves microcirculation and is activation of the following process: B. Hyperproduction of ACTH
known for its relatively good tolerance. What drug A. Gluconeogenesis C. Cumulation
is it? B. Glycogenolysis D. Sensibilization
A. Glibenclamide C. Glycogenesis E. Habituation.
B. Glibutidum D. Glycolysis
C. Insulin E. Lipolysis Examination of a 60 y.o. patient revealed
D. Acarbose hyperglycemia and glucosuria. A doctor
E. Adrenalin A person has reduced diuresis, hypernatremia, administered him a medication for internal use.
hypokalemia. Hypersecretion of what hormone can What medication is it?
Prednisolone cause such changes? A. Oxytocin
A. Parathormone B. Corglycon
C. Furosemide E. Hydrochlorothiazide. A. Levodopa
D. Pancreatine B. L-thyroxine*
E. Glibenclamid*. Examination of a 70 year old patient revealed non C. L-asparaginase
insulin-dependent diabetes. What drug should be D. L-carnitine
A patient ill with collagenosis has been taking administered? E. Parathyroid hormone.
prednisolone for a long time. Hypokaliemia A. Glibenclamid*
development caused spastic pain of skeletal B. Parathyroidin Choose the drug that is a hormone produced by
muscles. What medication should be used in order C. Insulin alpha-cells of Langerhan’s islets of the pancreas.
to correct potassium exchange? D. Mercazolilum A. Insulin
A. Dithylinum E. Cortisone. B. Glucagon*
B. Panangin* C. Somatostatin
C. Thyrocalcitonin Examination of a 60 year old patient revealed D. Natriuretic hormone
D. Diazepam hyperglycemia and glucosuria. A doctor E. Testosterone.
E. Noshpa. administered him a medication forinternal use.
What medication is it? Replacement hormonal treatment was assigned for
In course of histidine catabolism a biogenic amin is A. Pancreatine patient with hypothyrosis. Which drug should be
formed that has powerful vasodilatating effect. B. Furosemide selected for treatment?
Name it: C. Oxytocin A. Merkazolil (Thiamazole)
A. Histamine* D. Glibenclamid* B. Parathyreoidinum (Parathyreocrinum)
B. Dopamine E. Corglycon. C. Dihydrotachysterol
C. Dioxyphenylalanine D. Potassium perchlorate (Kalii perchloridi)
D. Noradrenalin Doctor prescribed preparation of posterior pituitary E. Thyroidin*.
E. Serotonin. for the woman with the uterine inertia. Choose the
preparation: Patients aged 73 years has slovly healing fracture
Continious taking of a drug can result in A. Dihydroergotamine of the femoral neck. What of hormones with
osteoporosis. erosion of stomach mucous B. Vasopressin anabolic effect can be assigned to him?
membrane, hypokaliemia, retention of sodium and C. Oxytocin* A. Parathyreoidin (Parathormon,
water, reduced content of corticotrophin in blood. D. Adrenocorticotropin Parathyreocrinum)
Name this drug: E. Dinoprost. B. Prednisolone
A. Digoxin C. Insulin
B. Indometacin The levorotatory isomer of thyroid hormone was D. Retabolil (nandrolone)*
C. Reserpine assigned to the patient with diagnosed myxedema. E. Thyroxine.
D. Prednisolone* What drug was assigned?
Blood pressure and blood glucose level were A patient ill with neurodermatitis has been taking
increased in female patients with low blood Dermatologist for topical treatment of eczema prednisolone for a long time. Examination revealed
pressure after parenteral administration of the selected ointment containing fluorinated high level of sugar in his blood. This complication
hormone. Which hormone was injected? glucocorticoid, which provides a more expressed is caused by the drug influence
A. Folliculin (Estrone) antiinflammatory effect. Which of the following A. Glycogenogenesis activation
B. Adrenaline* dermatological ointments was applied? B. Gluconeogenesis activation*
C. Glucagon A. Hydrocortisone ointment C. Intensification of glucose absorption in the
D. Progesterone B. Prednizolone ointment bowels
E. Insulin. C. Ointment "Sinaflan"* D. Activation of insulin decomposition
D. Diclophenac ointment E. Inhibition of glycogen synthesis.
E. Heparin ointment .
The patient developed a hypersensitivity to Continious taking of a drug can result in
common cold after long-term use of one of the Patient was on glucocorticoids for a long time, osteoporosis, erosion of stomach mucous
therapeutic agents. Which of these drugs could discontinuation of usage caused exacerbation of the membrane, hypokaliemia, retention of sodium and
reduce immunity? illness, decreased BP, weakness. How can you water, reduced content of corticotropin in blood.
A. Sustak forte (Nitroglycerin) explain it? Name this drug:
B. Prednisone* A. Insufficiency of adrenal glands* A. Digoxin
C. Reserpine B. Adaptation to the medicine B. Hydrochlorothiazide
D. Tincture Ginsengi C. Sensitization C. Prednisolone*
E. Clonidine. D. Hyperproduction of ACTH D. Indometacin
E. Cumulation. E. Reserpine .
16 years old young man was suffering from
diabetes mellitus for 10 years. He heard about the Testosterone and it's analogs increase the mass of A patient suffers from diabetes melitus. After the
replacement of insulin by glibenclamide tablets. skeletal muscles that allows to use them for regular insulin injection his condition grew worse:
However, the doctor refused to change insulin for treatment of dystrophy. Due to interaction of the there appeared anxiety, cold sweat, tremor of
glibenclamide tablets. Why glibenclamide is not hormone with what cell substance is this action limbs, general weakness, dizziness. What
assigned in this case? caused? preparation can eliminate these symptoms?
A. Rapidly degraded in the liver. A. Proteins- activators of transcription A. Adrenaline hydrochloride*
B. Deplete the function of beta cells of the B. Membrane receptors B. Butamide
pancreas.* C. Ribosomes C. Caffeine
C. Causes hyperproduction of hydrocortisone. D. Chromatin D. Noradrenaline
D. Increased allocation of adrenaline. E. Nuclear receptors*. E. Glibutide .
E. Stimulates the alpha cells of the pancreas..
Examination of a 60 y.o. patient revealed Diclofenac sodium A. Ulcerogenicity
hyperglycemia and glucosuri A doctor A patient has the pronounced pain syndrome B. Teratogenicity
administered him a medication for internal use. induced by neuralgia. What drug from the group of C. Ototoxicity
What medication is it? nonsteroidal antiinflammatory drugs will reduce D. Carcinogenicity
A. Corglycon pain sensitivity? E. Drug dependence
B. Furosemide A. Diclofenac sodium
C. Oxytocin B. Codeine phosphate On the 4th day of treatment with diclofenac sodium
D. Pancreatine C. Ketamine hydrochloride a 55-year-old patient has developed gastric
E. Glibenclamid*. D. Lidocaine hydrochloride hemorrhage due to an ulcer appearing on the
E. Droperidol gastric mucosa. Ulcerogenic action of this drug is
An elderly female patient suffers from the type 2 caused by decreased secretion of:
diabetes mellitus accompanied by obesity, A dentist prescribed the patient with maxillofacial A. Prostaglandin E2
A. Insulin arthritis diclofenac sodium. What is the mechanism B. Prostaglandin E1
B. Glibenclamid* of action of this drug? C. Thromboxane
C. Retabolil A. Catalase inhibition D. Cyclic endoperoxides
D. Lovastatin B. Opiate receptors activation E. Leukotriene
E. Amlodipine . C. Opiate receptors block
D. Phosphodiesterase activation Celecoxib
ANTI-INFLAMMATORY, ANTI-ALLERGIC A patient with arthritis and varicose veins has been
AND IMMUNOTROPIC DRUGS (48) A patient consulted a dentist about the taking a nonsteroidal antiinflammatory drug for a
temporomandibular joint arthritis. The dentist long time, which caused the thrombosis of
A 42-year-old female patient consulted a doctor administered an ointment containing diclofenac cutaneous veins. Which of the listed drugs might
about pain in the knee joints. Objectively there is sodium. What is its mechanism of action? have caused this complication?
swelling, redness, hyperthermia in the region of A. Cyclooxigenase inhibition A. Celecoxib
these joints. Laboratory testing revealed positive B. Phospholipase inhibition B. Indomethacin
acute phase reactants. What drugs should be used C. Opiate receptor activation C. Aspirin
to treat this patient? D. Opiate receptor block D. Phenylbutazone
A. Anti-inflammatory drugs E. Cyclooxigenase activation E. Ibuprofen
B. Narcotic analgesics
C. Antidepressants A 39-year-old patient with arthritis of the A patient suffering from arthritis is prescribed a
D. Antibiotics temporomandibular joint has been administered COX-2 selective inhibitor with anti-inflammatory
E. Sulfonamides diclofenac sodium. It must be kept in mind that the action. Specify this drug:
side effect of prolonged use of this drug is: A. Celecoxib
B. Butadion (Phenylbutazone) C. Menadione (Vicasolum) D Tavegyl
C. Dimexid D. Analgin (Metamizole) E Dimedrol
D. Indometacin E. Paracetamol
E. Analgin (Metamizole) Loratadine
A patient suffering from acute vascular purpura is A patient was prescribed loratadine to
To prevent possible negative effect upon the prescribed a firstgeneration antihistamine with treat allergic cheilitis. What is the mechanism
gastric mucsa a patient with rheumatoid arthritis local anaesthetic, antispasmodic, and sedative of action of this drug?
was administered a nonsteroid anti-inflammatory action. Specify this drug: A. Blockade of H1-histamine receptors
drug - a COX-2 selective inhibitor. Specify this A. Dimedrol (Diphenhydramine) B. Blockade of adrenergic receptors
drug: B. Diazolin (Mebhydrolin) C. Increases activty of monoamine oxidase
A. Celecoxib C. Dithylin D. Suppresses activity of Na+/K+-ATPase
B. Analgine D. Droperidol E. Suppresses activity of choline esterase
C. Acetylsalicinic acid E. Dibazol (Bendazol)
D. Butadion A 30-year-old driver complains of allergic rhinitis
E. Ibuprofen After using a toothpaste a 27-year-old patient has that usually excerbates in spring. He has been
developed Quincke’s edema. Administer a drug administered an antihistamine drug with a slight
Butadion (Phenylbutazone) from the group of histamine H1-receptor sedative effect and 24-hour period of action. Which
A patient with maxillofacial joint arthritis has antagonists for the treatment of this condition: of the listed drugs has been administered?
cometoadentist. The dentist prescribed an ointment A. Dimedrol A. Loratadine
with an antiinflammatory agent that is a pyrazolone B. Paracetamol B. Dimedrol
derivative. Name this agent: C. Digoxin C. Heparin
A. Butadion (Phenylbutazone) D. Chlorpromazine D. Vicasol
B. Mefenamic acid E. Analgin E. Oxytocin
C. Ibuprofen
D. Indometacin Ketotifen A 12-year-old child presents with intolerance to
E. Diclofenac sodium A 45-year-old woman suffers from allergic some foodstuffs. Their consumption causes an
seasonal coryza caused by the ambrosia allergic reaction in form of itching skin eruption.
Dimedrol (Diphenhydramine) blossoming. What medicine from the stabilizer of What anti-histaminic drug should be administered
A 26-year-old woman presents with skin rashes the adipose cells group can be used for prevention that won’t have any negative impact on the child’s
and itching after eating citrus fruits. Prescribe her a of this disease? school studies (with no sleepiness effect)?
drug that is an H1histamine receptor antagonist: A Ketotifen A. Loratadine
A. Dimedrol (Diphenhydramine) B Diazoline B. Dimedrol
B. Acetylsalicylic acid C Phencarol C. Sodium diclofenac
D. Aminophylline B. Antiserotonin
E. Mesatonum Pentoxylum C. Antihistamine
In the framework of complex treatment of D. Locally irritating
A woman works as railway traffic controller. She gingivitis a patient has been administered a drug E. Antikinine.
suffers from seasonal vasomotor rhinitis and gets that stimulates leucopoiesis, accelerates wound
treatment in the outpatient setting. She was healing, enhances the growth and proliferation of A 45-year-old woman suffers from seasonal
prescribed an antihistamine that has no effect upon cells, has the anti-inflammatory effect. It is applied allergic rhinitis caused by the ambrosia
central nervous system. What drug is it? for treatment of leukopenias of different genesis, in blossoming. What medicine from the stabilizer of
A Loratadine the dental practice it is used for treatment of the adipose cells group can be used for prevention
B Dimedrol inflammatory diseases of the oral mucosa. Identify of this disease?
C Promethazine the drug: A. Diazoline
D Suprastin A. Pentoxylum B. Tavegyl
E Tavegil B. Mercaptopurine C. Phencarol
C. Methotrexate D. Dimedrol
A patient has allergic rhinitis with profuse mucous D. Cyanocobalamin E. Kromolin sodium*.
discharges, itching, frequent sneezing. What drug E. Coamide
should be chosen if you know that it selectively A doctor administered a patient with allergic
blocks histamine receptors? A patient with continious bronchopneumonia was dermatitis a H1 -histamine blocker as a part of
A. Loratadine admitted to the therapeutic department. Antibiotic complex treatment. Name this medication:
B. Mesatonum therapy didn't give much effect. What medication A. Prednisolone
C. Adrenaline hydrochloride for improvement of immune state should be added B. Loratadine*
D. Naphthizin to the complex treatment of this patient? C. Cromolyn sodium
E. Prednisolone A. Timaline* D. Hydrocortisone
B. Analgin E. Adrenaline.
Methyluracil C. Sulfocamphocaine
A patient has a slowly healing fracture. What D. Paracetamol A 12 year old child has intolerance to some
medicine can be used to accelerate formation of E. Benadryl. foodstuffs. Their consumption causes an allergic
connective tissue matrix? reaction in form of itching skin eruptions. What
A. Methyluracil A patient with rheumatoid arthritis who had been antihistaminic drug should be admistered so that
B. Prednisolone treated with indometacin has got signs of the child could attend school?
C. Cyclophosphan gastropathy. What activity of the drug can this A. Loratadine*
D. Methotrexate complication be connected with? B. Dimedrol
E. Cyclosporine A. Anticyclooxygenase* C. Ephedrine
D. Aminophylline C. It must be preceded by administration of B. Indomethacin*
E. Diclofenac. ADH C. Analgin
D. It must be performed on the background of D. Prednisolone
Signs of gastropathy occurred in the patient with mineralocortcoid administration E. Dexametasone.
rheumatoid arthritis who was treated with E. It must be preceded by growth hormone
indometacin. With what activity of the drug can administration. In complex treatment of patient with bronchial
this complication be connected? asthma cromolyn-sodium was included as
A. Antikinine A 45-year-old woman suffers from allergic antiallergic agent. What is the mechanism of this
B. Antiserotonin seasonal coryza caused by the ambrosia drug action?
C. Antihistamine blossoming. What medicine from the stabilizer of A. Inhibition of histaminic H1 receptors
D. Anticyclooxygenase* the mast cells group can be used for prevention of B. Inhibition of histaminic H2 receptors
E. Local irritating. this disease? C. Prevention of mast cell degranulation*
A. Tavegyl D. Inhibition of antigen-antibody interaction
A female patient consulted a doctor about pain and B. Diazoline E. Blockade of D2 receptors.
limited movements in the knee joints. Which of the C. Phencarol
following nonsteroid anti-inflammatory drugs D. Ketotifen* A patient who had myocardial infarction was
should be administered taking into consideration E. Dimedrolum. administrated 75 mg of acetylsalicylic acid a day.
that the patient has a history of chronic What is the purpose of this administration?
gastroduodenitis? Patient was on glucocorticoids for a long time, A. Coronary vessels dilatation
A. Celecoxib* discontinuation of usage caused exacerbation of the B. Temperature reduction
B. Diclofenac sodium illness, decreased BP, weakness. How can you C. Inflammation reduction
C. Promedol explain it? D. Pain relief
D. Acetylsalicilic acid A. Sensitization E. Redaction of thrombocyte aggregation*.
E. Butadiounum . B. Adaptation to the medicine
C. Hyperproduction of ACTH Continuous use of certain drug may cause
The patient suffering from arthritis was treated D. Insufficiency of adrenal glands* osteoporosis, erosion of gastric mucosa,
with prednisolone for long period. Doctor decided E. Cumulation. hypokalemia, retention of sodium and water in the
to discontinue prednisolone administration and organism, decreased concentration of corticotropin
change it for another drug. What is the right way of Patient suffering from rheumatoid arthritis was in blood. What drug is it?
this drug discontinuation? treated with potent nonsteroidal anti-inflammatory A. Reserpine
A. It must be preceded by corticotropin drug. The treatment was complicated by gastric B. Digoxine
administration* bleeding. What drug was used? C. Hypothiazide
B. It must be done in a vary short time A. Paracetamol D. Prednisolone*
E. Indomethacin. A patient suffering from rheumatism was treated
Immune stimulant of animal origin was included in with prednisolone for two months. After the abrupt
A 33 –year-old female patient, who long treated for the complex treatment of patients with severe discontinuation of the drug administration patient’s
chronic polyarthritis, complains of high blood bronchopneumonia. Which of these drugs belong blood pressure dropped dramatically, there was a
pressure, changes in distribution of body fat, to this group of drugs? severe weakness, tachycardia, reappeared joint
menstrual disorders. Admission of which drug is A. Timalin* pain. Which condition was developed in patient
related to these complaints? B. Pirogenal due to prednisolone discontinuation?
A. Prednisolone* C. Prodigiozan A. Orthostatic collapse
B. Beclomethasone D. Levamisole B. Withdrawal*
C. Phenylbutazone E. Sodium nucleinat. C. Allergic reaction
D. Sinaflan D. Anaphylactic shock
E. Indomethacin. Patient has got severe trauma in car accident due to E. Hyperglycemic coma.
this kidney transplant was performed in emergency
What pharmacological property of dimedrolum procedure. Which of the following drugs should be A patient with rheumatoid arthritis was on non-
(diphenhydramine) is used in the hives treatment? included in the post-operative therapy for steroidal anti-inflammatory drug - diclofenac
A. Interference with formation of the antigen- successful engraftment of the transplanted organ? sodium. After some time worsening of associated
antibody complex A. Pirogenal disease occurred in the patient that required
B. Elimination of histamine action on the cell* B. Levamisole withdrawal of medication. What disease was
C. Sedative effect on the central nervous C. Timalin associated with rheumatoid arthritis?
system D. Prodigiozan A. Coronary artery disease
D. Violation of antibody synthesis E. Azathioprine*. B. Bronchial asthma
E. Inhibition of biologically active substances C. Gastric ulcer*
release by mast cell. Anaphylactic shock developed in patient after D. Diabetes mellitus
administration of lidocaine. Which of the following E. Hypertention.
The patient who received steroid anti-inflammatory medications must choose a doctor to bring the
drugs had a complication in the form of patient out of shock? A patient with rheumatoid arthritis was on
hypokalemia. Which of these drugs should be used A. Acetylsalicylic acid diclofenac sodium. After blood test the doctor
for correction of potassium level? B. Corticotropin canceled the drug administration. What the
A. Sodium chloride C. Azathioprine abnormality in blood test was the base for
B. Magnesium sulfate D. Adrenaline* diclofenac sodium discontinuation?
C. Panangin* E. Aminopyrine. A. Enhancement of hemocoagulation
D. Calcium lactate B. Eosinophilia
E. Calcium chloride. C. Leukopenia
D. Leukocytosis In the complex therapy of patient with allergic A woman, who had undergone mastectomy due to
E. Decrease in blood coagulation*. dermatitis doctor prescribed H1-histamine breast cancer, was prescribed a course of radiation
blockers. Point out the drug. therapy. What vitamin preparation has marked
The 12-year-old child has intolerance of some food A. Hydrocortisone antiradiationeffectduetoitsantioxidant activity?
stuffs. Their use causes an allergic reaction in the B. Prednisolone A. Tocopherol acetate
form of itchy skin rash. What antihistamines C. Cromolyn sodium B. Ergocalciferol
should be appointed, not to interfere with school D. Loratadine* C. Riboflavin
activities of the child (not cause drowsiness)? E. Adren D. Cyanocobalamin
A. Loratadine* E. Folic acid
B. Mezaton VITAMIN DRUGS (27)
C. Diclofenac sodium There are various diseases that cause sharp
D. Aminophylline Tocopherol increase of active oxygen, leading to cell
E. Diphenhydramine. Parodontosis is treated by means of antioxidants. membranes destruction. Antioxidants are used to
Which of the following natural compounds is used prevent it from happening. The most potent natural
To prevent transplant rejection after organ as an antioxidant: antioxidant is:
transplantation immunosuppressive hormone A. Tocopherol A. Alpha-tocopherol
therapy is compulsory. What group of hormones is B. Thiamine B. Glucose
used for this purpose? C. Gluconate C. Vitamin D
A. Catecholamines D. Pyridoxine D. Fatty acids
B. Thyroid E. Choline E. Glycerol
C. Glucocorticoids*
D. Mineralocorticoids Activation of free radical processes is a universal There are various diseases that cause sharp
E. Sex hormones. mechanism that triggers cell death. What inhibitors increase of active oxygen, thus leading to cell
of this process should be administered as a part of membranes destruction. Antioxidants are used to
Student addressed the doctor to appoint a drug for therapeutic interventions intended for the treatment prevent it from happening. The most potent natural
the treatment of allergic rhinitis occurred in lime of generalized periodontitis? antioxidant is:
blossoming. Which drug can be used? A. Tocopherol, ascorbate A. α-tocopherol
A. Loratadine* B. Riboflavin, pyridoxine B. Glucose
B. Losartan C. Calciferol, naphthoquinone C. Fatty acids
C. Noradrenaline gidrotartrat D. Thiamin, folate D. Glycerol
D. Inderal E. Cobalamin, pantothenic acid E. Vitamin D
E. Ambroxol.
Cholecalciferol; Ergocalciferol
A 10 month old child has high excitability, sleep Retinol acetate D. Phenylin (Phenindione)
disturbance, amyotonia, retarded dentition, teeth In order to speed up healing of the thermal injury it E. Contrykal (Aprotinin)
erupt with inadequate enamel calcification. These is required to prescribe a drug that facilitates
changes are caused by deficiency of the following epithelization of skin and mucous membranes. A 30-year-old patient after a case of viral hepatitis
vitamin: What drug is it? type B has developed complaints of continuous
A. Cholecalciferol A. Retinol acetate nasal hemorrhages. What drug would be the most
B. Riboflavin B. Tocopherol acetate advisable for treatment of this condition?
C. Thiamine C. Nicotinic acid A. Menadione (Vicasolum)
D. Retinol D. Ergocalciferol B. Asparcam
E. Nicotinamide E. Ascorbic acid C. Nadroparin calcium (Fraxiparine)
D. Folic acid
A patient has enamel erosion. What vitamin should A patient with parodontosis was prescribed a fat- E. Dipiridamol
be administered for its treatment? soluble vitamin that actively participates in redox
A. D processes in the organism. This antioxidant is a Thiamine
B. C growth factor, has antixerophthalmic action, and Treatment of many diseases involves use of
C. K contributes to maintenance of normal vision. In cocarboxylase (thiamine pyrophosphate) for
D. B1 dental practice it is used to accelerate mucosal re- supplying cells with energy. What metabolic
E. PP epithelization during parodontosis. Name this process is activated in this case?
substance: A. Oxidizing decarboxylation of pyruvate
A child with signs of rickets has been prescribed a A. Retinol acetate B. Glutamate deamination
certain liposoluble vitamin drug by the pediatrician B. Ergocalciferol C. Amino acids decarboxylation
and dentist. This drug affects the metabolism of C. Tocopherol acetate D. Decarboxylation of biogenic amines
phosphorus and calcium in the body and facilitates D. Menadione (Vicasolum) E. Detoxication of harmful substances in liver
calcium accumulation in bone tissue and dentine. If E. Cyanocobalamin
its content in the body is insufficient, there develop Pyridoxine (Pyridoxal phosphate)
disorders of ossificationprocess, dental structure, Menadione (Vicasolum) A patient has been prescribed pyridoxal phosphate.
and occlusion. Name this drug: Preoperative examination revealed prothrombin What processes are corrected with this drug?
A. Ergocalciferol deficiency in the blood of the patient. What drug A. Transamination and decarboxylation of
B. Retinol acetate should be preliminarily prescribed to mitigate amino acids
C. Tocopherol acetate blood loss in the pati- ent during the surgery? B. Oxidative decarboxylation of keto acids
D. Menadione (Vicasolum) A. Vicasol (Menadione) C. Deaminization of amino acids
E. Thyroidin B. Thrombin D. Synthesis of purine and pyrimidine bases
C. Aminocapronic acid E. Protein synthesis
E. Pyridoxine C. Hypervitaminosis vit. D*
Ascorbic acid D. Hypovitaminosis vit. A
Examination of a child who hasn’t got fresh fruit There is an inhibited coagulation in the patients E. Hypervitaminosis vit. K.
and vegetables during winter revealed numerous with bile ducts obstruction, bleeding due to the low
subcutaneous hemorrhages, gingivitis, carious level of absorbtion of a vitamin. What vitamin is in The retardation of fontanelle closure and teeth
cavities in teeth. What vitamin combination should deficiency? appearance were observed in child. The increase in
be prescribed in this case? A. Carotene activity of alkaline phosphatase and decrease in the
A. Ascorbic acid and rutin B. E concentration of citrate in the blood were found.
B. Thiamine and pyridoxine C. K* This violation is related to deficiency of vitamin:
C. Folic acid and cobalamin D. A A. Vitamin D*
D. Riboflavin and nicotinamide E. D. B. Vitamin A
E. Calciferol and ascorbic acid C. Ascorbic acid
A patient suffers from vision impairment - D. Vitamin E
Cyanocobalamin hemeralopy (night blindness). What E. Vitamin K.
A 13-year-old girl has been prescribed a certain vitaminpreparation should be administered the
drug for treatment of megaloblastic anemia. This patient in order to restore his vision? The patient with myocardiodystrophy appointed
drug stimulates a transfer from megaloblastic A. Thiamine chloride vitamin preparation with a potent antioxidant
haemopoiesis to normoblastic, participates in B. Tocopherol acetate effect. Which of these drugs has such action?
synthesis of purine and pyrimidine bases, activates C. Retinol acetate* A. Rutin
proteine and methionine synthesis. What drug does D. Vicasol B. Pyridoxine
the patient take? E. Pyridoxine. C. Menadione
A. Cyanocobalamin Patients after long-term complex treatment that D. Riboflavin
B. Haemostimulinum included vitamin drug began to complain of E. Tocopherol*.
C. Rosehip tea muscular weakness, loss of appetite, nausea, and
D. Erythropoietin diarrhea. On examination tachycardia, A woman addressed the pediatrician concerning
E. Ferric sulfate hypertension, weakening of the heart sounds were illness of the 8 months child manifested by
revealed. In the urine protein, red blood cells, sweating, increased size of the crown, retarded
A patient with megaloblastic anemia was taking a hypercalciuria were revealed, in the blood - dentition (only 2 of the tooth), and anxiety. What
water-soluble vitamin. Name this substance: increasing the concentration of Ca and P was medication should be assigned at the first place?
A. Cyanocobalamin found. What phenomenon was observed in a A. Calcium pangamas
B. Thiamine chloride patient? B. Folic acid
C. Tocopherol acetate A. Hypovitaminosis vit. C C. Cyanocobalamin
D. Ascorbic acid B. Hypervitaminosis vit. A D. Thiamin
E. Ergocalciferol*. A. Levamisole for a long time. She was admitted to the hospital
B. Retabolil for general health aggravation, arrhythmia, nausea,
The woman had several spontaneous miscarriages. C. Prednisolone reduced diuresis, insomnia. What is the primary
The lack of which vitamin can cause this? D. Retinol acetate* action to be taken?
A. Thiamine bromide E. Methyluracil . A. To withhold digitoxin
B. Vitamin D B. To reduce digitoxin dosage
C. Vitamin K CARDIOTONIC DRUGS INCLUDING C. To administer strophanthine intravenously
D. Vitamin E* CARDIAC GLYCOSIDES. D. To administer digoxin
E. Ascorbic acid. ANTIARRHYTHMIC DRUGS (32) E. To give an intravenous injection of calcium
gluconate solution
The patient complains of increased vascular Digoxin
fragility (touch to the skin are bruises), bleeding A patient suffers from chronic left-ventricular A patient with chronic heart failure had been taking
from the gums. Appointment of ascorbic acid insufficiency. What drug should be prescribed? digitoxin for several months, during digitalization
reduced these symptoms. Specify the mechanism A. Digoxin the following symptoms developed: headache,
of action of the drug on vascular permeability. B. Bemegride nausea, diarrhea, loss of appetite, impaired color
A. Stimulates the synthesis of corticosteroids C. Etimizol vision, bradycardia. What antidote should be
B. Stabilizes the cell membrane D. Vinpocetine administered to reduce the intoxication symptoms?
C. Improves oxidation-inflammatory processes E. Pyracetam A. Unithiol
D. Increases collagen production* B. Atropine sulfate
E. Stimulates the conversion of folic acid to When treating a patient with chronic cardiac failure C. Prednisolone
folinic. a doctor detected bradycardia and deterioration of D. Adrenalin hydrochloride
the patient’s general state. Such condition is caused E. Naloxone
The patient has dementia, diarrhea and dermatitis. by cumulative effect of a drug. Which drug of
Which of the following drugs should be included in those listed below has cumulative action? Corglycon
complex treatment of this patient? A. Digoxin A patient with symptoms of acute heart failure,
A. Nicotinamide* B. Diphenhydramine (Dimedrol) namely pallor, acrocyanosis and rapid shallow
B. Cobalamin C. Hydrochlorothiazide breathing, has been delivered to the emergency
C. Calcium pantothenate D. Isosorbide department. Which of these drugs is indicated in
D. Riboflavin E. Retinol acetate this case?
E. Thiamine nitrophosphate. A. Corglycon
Digitoxin B. Digitoxin
In order to accelerate healing of a radiation ulcer a A 65-year-old patient with chronic heart failure has C. Cordiamine
vitamin drug was administered. What drug was it? been taking digitoxin in self-administered dosages D. Nitroglycerine
E. Adrenalin hydrochloride E. Anti-shock Which of the following drugs is most likely to
produce positive inotropic and negative
Strophanthin A patient with ventricular arrhythmia was admitted chronotropic effects?
A 55-year-old mae patient with acute heart failure to the cardiological department. What drug should A. Nitroglycerin
has been administered a quick-relief cardiac be administered? B. Furosemide
glycoside. Which of the following drugs has been A. Amiodarone C. Diltiazem
given to the patient? B. Amlodipine D. Procainamide
A. Strophanthin C. Drotaverine E. Digoxin.*.
B. Adonisidum D. Aminazine
C. Digitoxin E. Proserin Choose the inotropic drug for treatment of patient
D. Celanid suffering from congestive heart failure, not
E. Milrinone Verapamil responding to digitalis.
A paroxysm of tachycardia occurred in a patient A. Phenylephrine
Amiodarone undergoing a dental procedure. Which of the B. Dobutamine*
A patient in a cardiological department has following drugs should be used to relieve it? C. Clonidine
arrhythmia. A doctor administered him amyodaron. A. Verapamil D. Noradrenaline
What is the main mechanism of amyodaron’s B. Isadrinum E. Salbutamol.
antiarrhythmic action? C. Atropine
A. It blocks mostly potassium channels D. Dipheninum A patient who has been suffering from cardiac
B. It inhibits cholinoreceptors E. Nitroglycerine insufficiency for several months has been taking
C. It stimulates histamine receptors digoxin on an outpatient basis. At a certain stage of
D. It activates serotonin receptors A patient complains about retrosternal pain, treatment there appeared symptoms of drug
E. It alters myocardium susceptibility to the dyspnea and palpitation. After examination he was overdose. What phenomenon underlies the
acetylcholine diagnosed with coronary heart disease and development of this complication?
prescribed verapamil. What is the mechanism of its A. Functional cumulation
A female patient suffering from coronary artery action? B. Tachyphylaxis
disease has been prescribed amiodarone that has A. It blocks calcium channels C. Habituation
antianginal action. What other action does this drug B. It blocks α-adrenoreceptors D. Sensibilization
have? C. It blocks β-adrenoreceptors E. Material cumulation*.
A. Antiarrhythmic D. It blocks potassium channels
B. Analgesic E. It blocks sodium channels A 50 y.o. patient with chronic cardiac insufficiency
C. Local anaesthetic and tachyarrythmia was prescribed cardiotonic
D. Anti-inflammatory drug. What drug was prescribed?
A. Dobutamine A. Verapamil lower dose of digoxin. In spite of this the
B. Mildronate B. Amiodarone* bradycardia was soon transformed into A-V-block.
C. Amyodarone C. Quinidine Choose the necessary drug for abolishing of this
D. Digoxin* D. Procainamide manifestation of glycoside toxicity.
E. Dopamine. E. Lidocaine. A. Potassium chloride
B. Phenytoin
A patient suffering from chronic cardiac Patient suffering from tachyarrhythmia was treated C. Atropine*
insufficiency was recommended to undergo a with calcium channel blocker. What drug was D. Lidocaine
prophylactic course of treatment with a used? E. Propranolol.
cardiological drug from the group of cardiac A. Niphedipine
glycosides that is to be taken enterally. What drug B. Verapamil* A pregnant woman addressed her doctor
was recommended? C. Labetolol complying of frequent heartbeats. Which of
A. Corglycon D. Alprenolol antiarrhythmic agents is contraindicated in this
B. Cordiamin E. Nicardipine. case?
C. Strophanthine A. Atenolol
D. Cordarone A 52 year old patient with supraventricular B. Propranolol*
E. Digoxin*. tachyarrhythmia was using antiarrhythmic drug of C. Metoprolol
class III. For a long period it maintained the normal D. Verapamil
A patient that entered the admission office had the cardiac rhythm, but with time dispnoe started to E. Diltiazem.
following signs of acute cardiac insuffiency: occur while going upstairs. Patient addressed
paleness, acrocyanosis, frequent shallow doctor. The examination revealed pulmonary Patient with chronic congestive heart developed
respiration. What drug is indicated in this case? sclerosis. Which of the antiarrhythmic drug can tachyarrhythmia. Which of the listed drugs is
A. Corglycon* cause such side effect? contraindicated in this case?
B. Adrenaline hydrochloride A. Verapamil A. Quinidine*
C. Nitroglycerine B. Amiodarone* B. Lidocaine
D. Cordiamin C. Bretylium C. Timecaine
E. Digitoxin. D. Lidocaine D. Phenytoin
E. Procainamide. E. Ajmaline.
To maintain normal sinus rhythm in patient with
atrial fibrillation the doctor prescribed an Digoxin in tablets was prescribed to patient with In the treatment of patient suffering from
antiarrhythmic drug. After laboratory testing of chronic congestive heart failure. After 1 month of congestive heart failure resistant to digitalis
thyroid function administration of this drug was treatment decrease in heart rate was noted doctor adrenomimetic was used as inotropic agent. Point
withdrawn. What drug was initially prescribed? advised the patient to continue treatment with out which of the following drugs was administered.
A. Phenylephrine inhibitor for hypertension. You have decided to E. Lisinopril
B. Partusisten use phenytoin instead of procainamide. What is
C. Dobutamine* the reason? A patient suffers from chronic left-ventricular
D. Xylometazoline A. The total anesthetic effect of procaninamide insufficiency. What medication should be
E. Naphazoline would aggravate the hypertension administered?
B. The anticholinergic effect of procainamide A. Digoxin*
A 62-year-old patient, prone to would aggravate glaucoma* B. Bemegride
bronchoconstriction, addressed his doctor with C. The local aneathetic effect of C. Aethimizolum
complaints of tachyarrhythmia troubling him procainamaide would potentiate diabetes D. Vinpocetine
several times a day. The doctor prescribed D. The hypertensive effects of procainamide E. Pyracetam
anaprilin (propranolol). Administration of drug would aggravate the hypertension
provoked severe attack of bronchospasm. What is E. The cholinergic effects of procainamide Patient suffering from acute cardiac failure was
the mechanism of this side effect? would aggravate the diabetes treated with nonglycoside inotropic drug inhibiting
A. Stimulation of M1- cholinoceptors phosphodiesterase III. Which drug was used?
B. Inhibition of α-adrenoceptors A patient with chronic cardiac insufficiency has A. Mildronate
C. Inhibition of β2 - adrenoceptors* been treated with cardiotonic drugs and a B. Dopamine
D. Increase in n. vagus tone thiazide diuretic, but in spite of it there are C. Dobutamine
E. Stimulation of leukotriene release still edemata and risk of ascites. What medication D. Amyodarone
should be prescribed to amplify diuretic effect of E. Amrinone*
A patient with fibrillation, who has had bronchial the applied drugs?
asthma in his anamnesis, should be administered A. Furosemide Patient complains of weakness, dyspnea, low
antiarrhythmic drug. Which preparation from the B. Spironolactone* extremities oedema. Diagnosis: chronic cardiac
suggested drugs is contraindicated for this patient? C. Amyloride insufficiency. What medicine should be prescribed
A. Ajimaline D. Clopamide first of all?
B. Verapamil E. Mannitol A. Caffeine
C. Novocainamide B. Digitoxin*
D. Nifedipine A patient takes digoxin for treatment of cardiac C. Papaverine
E. Anapriline (Propranolol)* insufficiency. What diuretic may increase digoxin D. Propranolol
toxicity due to the intensified excretion of K+ ions? E. Raunatin
A 55-year-old patient with persisting ventricular A. Spironolactone
arrhythmia was admitted to the hospital. The B. Hydrochlorothiazide* ANTIHYPERTENSIVE DRUGS, DIURETIC
patient is taking Timolol drops for glaucoma, daily C. Panangine DRUGS (50)
insulin injections for diabetes mellitus, and an ACE D. Siliborum
Magnesium sulfate A patient with essential hypertension has been forced diuresis was induced. What substance was
For relief of hypertensive crisis a doctor prescribed captopril. What is its mechanism of used to perform this procedure?
administered a patient a drug that apart from action? A. Furosemide
antihypertensive effect has also sedative, A. Inhibition of angiotensin-converting B. Hydrochlorothiazide
spasmolytic and anticonvulsive effect. The drug enzyme activity C. Omeprazole
was taken parenterally. When it is taken enterally it B. β-adrenoreceptor block D. Spironolactone
acts as a laxative and cholagogue. What drug was C. α-adrenoreceptor block E. Dithylinum (Suxamethonium chloride)
administered? D. Angiotensin II receptor block
A. Magnesium sulfate E. Peripheral vasodilatating effect Spironolactone
B. Dibasolum Despite the administration of cardiotonics
C. Reserpine Hydrochlorothiazide and thiazide diuretic a patient with chronic heart
D. No-spa To treat chronic heart failure a patient takes failure has persistent edemas and the risk of ascites
E. Apressin digoxin. What diuretic can increase digoxin arose. What medication should be administered to
toxicity due to increased excretion of K+ ions? enhance the
To terminate hypertensive crisis the patient was A. Hydrochlorothiazide diuretic effect of the administered drugs?
administered solution of magnesium sulfate. What B. Spironolactone A. Spironolactone
route of drug administration should be chosen? C. Panangin B. Furosemide
A. Intravenous D. Silibor C. Amiloride
B. Duodenal E. Lisinopril D. Clopamide
C. Rectal Furosemide E. Manithol
D. Oral During narcosis a patient developed a
E. Intra-arterial risk of cerebral edema.What drug should be The alternate usage of dichlotiazide, etacrin acid
administered in this case? and lasex did not cause marked diuretic effect in
Amlodipine A. Furosemide the patient with marked peripheral edema. The
A patient has arterial hypertension. What long- B. Dopamine aldosterone level in the blood is increased. Indicate
acting drug from the group of calcium channel C. Phenazepam which medicine should be prescribed:
blockers should be prescribed? D. Triamterene A. Spironolactone
A. Amlodipine E. Sodium bromide B. Mannit
B. Octadine C. Clopamid
C. Pyrroxanum Resuscitation unit received a patient with acute D. Urea
D. Atenolol poisoning caused by unidentified medicine. To E. Amilorid
E. Reserpine quickly excrete the poisonfrom the patient’s body,
Captopril
A patient with edemata was prescribed a K+- pains in joints. Which of the listed side effects is Allopurinol was prescribed to decrease the level of
retaining diuretic - aldosterone antagonist. What the most probable cause of patient’s complains? urinary acid. Competitive inhibitor of what enzyme
drug is it? A. Hyperglycemia is allopurinol?
A. Spironolactone B. Hyperuricemia* A. Xanthine oxidase*
B. Digoxin C. Drug hypersensitivity B. Guanine deaminase
C. Procainamide hydrochloride D. Hyperkalemia C. Adenosine deaminase
D. Clonidine E. Hypokalemia. D. Hypoxanthine phosphoribosiltransferase
E. Alopurinole E. Adenine phosphoribosiltransferase.
A patient suffering from hypertension was treated
Allopurinole with hydrochlorothiazide. With time he addressed A doctor administered Allopurinol to a 26-year-old
A 42-year-old man with gout presents with high doctor complaining of worsening of his condition. young man with the symptoms of gout. What
content of uric acid in blood. The patient was On examination of the patient hypokalemia was pharmacological action of Allopurinol ensures
prescribed allopurinol to lower the concentration of diagnosed. Choose the drug, which has to be added therapeutical effect?
uric acid. Allopurinol is a competitive inhibitor of aiming at enhancement of diuretic effect and A. By inhibiting leucocyte migration into the
the following enzyme: abolishing of hypokalemia. joint
A. Xanthine oxidase A. Acetazolamide B. By general analgetic effect
B. Adenosine deaminase B. Furosemide C. By increasing uric acid excretion
C. Adenine phosphoribosyltransferase C. Ethacrynic acid D. By general anti-inflammatory effect
D. Hypoxanthine phos horibosyltransferase D. Spironolactone* E. By inhibiting uric acid synthesis*.
E. Guanine deaminase E. Chlorothiazide.
The alternate usage or dichlotiazide, etacrin acid
Point out antihypertensive agent that increases salts A patient with acute poisoning was admitted to and lasex did not influence diuretically upon the
and H2O excretion and can causes hyperglycemia emergency department. It was established that patient with marked peripheral edemata. The
and uric acid retention? poisonous substance was excreted by kidney. aldosterone rate in the blood is increased. Indicate
A. Nifedipine Choose the best diuretic for forced diuresis. which medicine should be prescribed:
B. Propranolol A. Acetazolamide A. Urea
C. Prazosin B. Amiloride B. Amilorid
D. Hydrochlorothiazide* C. Spironolactone C. Clopamid
E. Acetazoleamide (Diacarbum). D. Furosemide* D. Spironolacton*
E. Hydrochlorothiazide. E. Mannit.
A 67- year-old man treated with
hydrochlorothiazide for control of mild A 42-year-old man suffering from gout has A patient with chronic cardiac insufficiency has
hypertension visited his doctor complaining of increased level of urinary acid in blood. been treated with cardiotonic drugs and a thiazide
diuretic, but in spite of it there are still edemata and complaining of pain in joints. Which diuretic was glomerulonephritis. What diuretic should be
risk of ascites. What medication should be used? administered for forced diuresis?
prescribed to amplify diuretic effect of the applied A. Spironolactone A. Furosemide*
drugs? B. Hydrochlorothiazide* B. Hydrochlorothiazide
A. Spironolactone* C. Clonidine C. Chlorthalidone
B. Furosemide D. Triamterene D. Cyclometazide
C. Manitole E. Enalapril E. Acetazolamide
D. Clopamide
E. Amyloride. A patient with chronic cardiac insufficiency has A hypertensive patient was treated with the drug
been treated with cardiotonic drugs and a suppressing the formation of angiotensin II and
A patient takes digoxin for treatment of cardiac thiazide diuretic, but in spite of it there are preventing degradation of bradykinine. Point out
insufficiency. What diuretic may increase digoxin still edema and risk of ascite. What medication the drug realizing antihypertensive effect by these
toxicity due to the intensified excretion of K+ ions? should be prescribed to amplify diuretic effect of mechanisms.
A. Lisinopril the applied drugs? A. Nifedipine
B. Panangine A. Spironolactone* B. Guanethidine
C. Siliborum B. Furosemide C. Clonidine
D. Spironolactone C. Amyloride D. Enalapril*
E. Hydrochlorothiazide* D. Clopamide E. Propranolol.
E. Mannitol
Highly potent diuretic drug was prescribed to the Diuretic drug was prescribed to the patient with A hypertensive patient was fond of meat dishes and
patient with pulmonary edema in the course of hypertension in the course of complex treatment. In did not keep proper diet. In the course of complex
complex treatment. In a few days signs of a few days BP decreased but signs of hypokaliemia antihypertensive treatment an acute attack of gout
hypokaliemia were developed. What drug could developed. What drug could cause such developed. Which of the following agents is
cause such complications? complications? responsible for worsening of patient condition?
A. Furosemide (Lasix)* A. Spironolactone A. Hydrochlorothiazide*
B. Spironolactone B. Clophelin B. Prazosin
C. Acetazolamide C. Lasix (Furosemide)* C. Allopurinol
D. Triamterene D. Triamterene D. Acetazolamide
E. Enalapril E. Enalapril E. Propranolol.

For treatment of hypertensive patient doctor A 65 year old female patient suffers from chronic The blood pressure of hypertensive patient is
prescribed diuretic agent. It resulted in sufficient renal insufficiency accompanied by evident successfully controlled by administration of
control of BP, but soon patient addressed doctor, edemata caused by chronic
enalapril. Which mechanism is responsible for Doctor substituted captopril for losartan. What is A patient with hypertensic crisis was admitted to
antihypertensive effect of this drug? the mechanism of the last drug action? the cardiological department, he was injected
A. Inhibition of peripheral alpha1 - adrenoceptors A. Inhibition of renin release intravenously with an antihypertensive drug - salt
B. Blockade of angiotensin II receptors B. Inhibition of ACE of an alkaline-earth metal. What drug was injected?
C. Inhibition of renin release C. Inhibition of angiotensinogen conversion to A. Benzohexamethonium
D. Inhibition of kallikrein – kinin system angiotensin I B. Potassium chloride
E. ACE inhibition and stimulation of kalliкrein – D. Blockade of angiotensin II receptors* C. C Sodium hydrocarbonate
kinin system*. E. Increase in bradykinin level. D. Magnesium sulfate*
E. Caleium lactate.
Metabolism can render pharmacological activity to A patient with II stage hypertension has been
some initially inert substances (pro-drug). Point out taking one of hypotensive medications for the For treatment of hypertensive patient doctor
the substance, which is pro-drug, used for purpose of treatment. After a time arterial pressure prescribed antiadrenergic drug, first administration
treatment of hypertension. decreased, but the patient started complaining of of which in therapeutic dose caused “first dose
A. Levodopa flaccidity, sleepiness, indifference. A bit later he syncope”. Which drug was most probably used?
B. Enalapril* felt stomach pain. He was diagnosed with ulcer. A. Metoprolol
C. Verapamil What hypotensive medication has the patient been B. Atenolol
D. Reserpine taking? C. Prazosin*
E. Propranolol. A. Captopril D. Reserpin
B. Dibazoie E. Propranolol
Hypertensive patient was treated with the drug that C. Reserpine*
decreases vascular tone. His treatment was D. Furosemide A 43 year-old male patient is suffering from
complicated by persistent dry cough. Which drug E. Verapamil. hypertension. His blood pressure was successfully
most probably was used? controlled by monotherapy with adrenoblocker.
A. Papaverine A patient taking clonidine for essential With time treatment was complicated with diarrhea
B. Phentolamine hypertension treatment was using alcohol that and impotence. The patient addressed his doctor
C. Lisinopril* caused intense inhibition of central nervous and the drug was changed for another
D. Prazosin system. What may it be connected with? adrenoblocker. Which of adrenoblockers can cause
E. Clonidine. A. Intoxication above listed side effects?
B. Idiosyncrasy A. Metoprolol
For treatment of hypertensive patient doctor C. Cumulation B. Salbutamol
prescribed captopril, but soon patient addressed D. Effect potentiating* C. Propranolol
doctor, complaining of dry cough and rashes. E. Effect summation. D. Dobutamine
E. Prazosin*
A patient with moderate hypertension was treated B. Low density lipoproteins (LDL)*
Which of the following antiadrenergic drugs used with the drug that realizes its effect by decrease in C. Very low-density lipoproteins (VLDL)
in the treatment of hypertension is contraindicated noradrenaline release from sympathetic nerve D. High-density lipoproteins (HDL)
in patients prone to bronchospasm? endings. While treatment patient experienced E. Intermediate density lipoproteins (IDL)
A. Prazosin drowsiness. What the drug was used?
B. Metoprolol A. Prazosin A 64 –year-old patient addressed doctor
C. Reserpine B. Reserpin* complaining of vertigo and ringing in ears. On
D. Atenolol C. Drotaverine examination the disturbance of brain blood
E. Propranolol (Anaprilinum)* D. Atenolol circulation was diagnosed. Doctor prescribed the
E. Dichlorothiazide drug that is vinca alkaloid derivative. Which drug
Point out antihypertensive agent that increases salts was prescribed?
and H2O excretion and can cause hyperglycemia Phenofibrate was prescribed for a patient with a A. Cinnarizine
and uric acid retention? history of angina pectoris and high plasma B. alfa-Tokopherol
A. Nifedipine cholesterol level. What is the mechanism of action C. Vinpocetine*
B. Propranolol of this drug? D. Xantinol nicotinate
C. Prazosin A. Activation of lipoprotein lipolysis* E. Parmidin
D. Hydrochlorothiazide* B. Inhibition of lipoprotein lipase
E. Acetazoleamide (Diacarbum) C. Inhibition of hydroxymethyl glutaryl Point out the class of lipoprotein that facilitates the
coenzyme A removal of cholesterol from tissues
A patient suffering from hypertension was treated D. Inhibition of DL oxidation A. Chylomicrons (Chy)
with hydrochlorothiazide. With time he addressed E. Bile acid sequestration B. Low density lipoproteins (LDL)
doctor complaining of worsening of his condition. C. Very low-density lipoproteins (VLDL)
On examination of the patient hypokalemia was In complex treatment of hypertensive patient D. High-density lipoproteins (HDL)*
diagnosed. Choose the drug, which has to be added antiatherogenic agent that decreases synthesis of E. Intermediate density lipoproteins (IDL)
aiming at enhancement of diuretic effect and cholesterol was included. Point out this agent.
abolishing of hypokalemia. A. Cholestyramine A 60 year-old patient with angina pectoris visited
A. Acetazolamide B. Colestipol her doctor for routine examination. Blood sampling
B. Furosemide C. Fenofibrate revealed elevated level of cholesterol in the
C. Ethacrynic acid D. Simvastatin* plasma. Due to this antisclerotic drug was added to
D. Spironolactone* E. Probucol antianginal therapy. The effect of prescribed drug
E. Chlorothiazide is based upon the inhibition of cholesterol
Point out the most atherogenic lipoproteins synthesis by suppression of HMG-CoA reductase
A. Chylomicrons (Chy) activity. What the drug was prescribed?
A. Nicotinic acid C. Reserpine E. Verapamil
B. Probucol D. Propranolol
C. Lovastatin* E. Aliskiren* A 57-year-old patient with coronary artery disease
D. Clofibrate has been administered an anti-anginal agent that
E. Colestipol A 66 year old female patient got intravenous activates guanylate cyclase and accumulates
injection of magnesium sulfate solution for the myocardial cGMP. What drug is it?
Patient with moderate hypertension was treated purpose of elimination of hypertensive crisis. But A. Isosorbide mononitrate
with the drug, decreasing heat work and renin arterial pressure didn't go down and after B. Dipyridamole
release, and causing coldness of hands as a side repeated introduction of the same C. Panangin
effect. What the drug was used? preparation there appered sluggishness, slow D. Validol
A. Prazosin response, inhibition of consciousness and E. Verapamil
B. Propranolol* respiration. What preparation is antagonist of
C. Spironolacton magnesium sulfate and can eliminate symptoms of Nitroglycerine
D. Atenolol its overdose? A patient with ischemic heart disease has been
E. Dichlorothiazide A. Calcium chloride* administered an anti-anginal drug that reduces the
B. Potassium chloride myocardial oxygen consumption and improves
Blood pressure of hypertensive patient is C. Sodium chloride blood supply of myocardium. What drug is it?
successfully controlled by administration of D. Activated carbon A. Nitroglycerine
enalapril. Which mechanism is responsible for E. Potassium permanganate B. Validol
antihypertensive effect of this drug? C. Propranolol
A. Inhibition of peripheral alpha –1 ANTIANGINAL DRUGS. HYPOLIPIDEMIC D. Promedol
adrenoceptors DRUGS (27) E. Retabolil
B. Blockade of angiotensin II receptors
C. Inhibition of renin release Isosorbide mononitrate A 60-year-old patient has taken a drug to relieve
D. Inhibition of kallikrein – kinin system A patient has coronary heart disease. For its angina pectoris attack; in several minutes pain felt
E. ACE inhibition and enhancement of treatment he was prescribed an antianginal drug in the breastbone area abated, but it was followed
kalliкrein – kinin system* that activates guanylate cyclase and accumulates by feeling of vertigo, headache, tinnitus, and
cyclic guanosine monophosphate in the hyperemia of face. What drug has he taken?
A 60 year-old patient suffering from hypertension miocardium cells. What drug is it? A. Nitroglycerine
was prescribe drug inhibiting RAS that inhibits A. Isosorbide mononitrate B. Validol
rennin directly. Point out this drug. B. Dipiridamol C. Nifedipine
A. Enalapril C. Panangine D. Verapamil
B. Verapamil D. Validol E. Amiodarone
B. Increase in myocardial oxygen very low density lipoproteins. What drug should be
A man with ischemic heart disease has been taking consumption prescribed?
his medicine too often throughout a day, which C. Dilatation of peripheral vessels A. Fenofibrate
resulted in poisoning. Examination detects D. Obstruction of coronary vessels B. Famotidine
cyanosis of skin and mucosa, sharp drop of blood E. Compression of coronary vessels C. Amiodarone
pressure, tachycardia, and respiratory depression. D. Lisinopril
Blood methemoglobin is high. What type of Myocardial infarction E. Dobutamine
medicine did the patient overdose on? A patient has myocardial infarction with
A. Organic nitrates thrombosis of the left coronary artery. What A patient with angina pectoris was helping himself
B. α-adrenergic blockers pharmocological preparation group should be used with nitroglycerin. One day the patient used
C. Calcium channel blockers to reestablish blood flow? nitroglycerin several times because of frequent
D. Adenosine-based drugs A. Fibrinolysis activators episodes of pain. So as the last sublingual
E. Myotropic antispasmodics B. Narcotic analgesics administration of nitroglycerin did not relieve pain,
C. β-adrenergic blockers the patient addressed his doctor. Doctor advised
A 60-year-old patient consulted a doctor about D. Angiotensin-converting enzyme inhibitors him to withdraw nitroglycerin for 8 hours and
retrosternal pain arising immediately after physical E. Glucocorticoids prescribed another drug. After 8 hours the effect of
exercise. He was prescribed nitroglycerin. The nitroglycerin was restored. Which phenomenon did
medication relieved retrosternal pain but the patient Lovastatin take place in this case?
got acute headache. What is the likely mechanism Among the specific hypocholesterolemic drugs the A. Increase in nitroglycerin metabolism
of this side effect? most effective are those blocking the synthesis of B. Increase in protein binding
A. Intracranial pressure rise endogenic cholesterol in liver. Which of the below C. Decrease of absorption
B. α-adrenoreceptor block listed drugs has such mechanism of D. Drug dependence
C. Phosphodiesterase block hypocholesterolemic action? E. Tolerance *.
D. Reduced accumulation of calcium ions A. Lovastatin
E. Inhibited formation of mediators in brain B. Probucol A patient with Raynaud disease was treated with
C. Clofibrate niphedipine. What is the mechanism of action of
After a serious psychoemotional stress a 48-year- D. Linaethol this drug?
old patient suddenly developed acute heart ache E. Allilcepum A. Blockade of B-adrenoceptors
irradiating to the left arm. Nitroglycerine relieved B. Blockade of alpha adrenoceptors
the pain attack after 10 minutes. What is the Fenofibrate C. Decrease in neurotransmitter amount
leading pathogenetic mechanism of this process A patient with ischemic heart disease presents with D. Calcium channels’ blockade*
development? increased blood plasma content of triglycerides and E. Blockade of central M-cholinoceptors.
A. Spasm of coronary arteries
To prevent the onset of myocardial infarction in a A 72-year woman suffering from angina pectoris insufficiency. What medicine should be prescribed
patient with angina pectoris aspirin was used. Due was treated by propranolol (anaprilinium). On the first of all?
to which of listed effects aspirin is beneficial in background of the treatment cough with A. Digitoxin*
this case? bronchospastic components was developed. Doctor B. Caffeine
A. Antiinflammatory replaced propanolol with metoprolol. This resulted C. Papaverine
B. Antiplatelet* in abolishing of respiratory disturbances. What is D. Propranolol
C. Analgesic the reason for the beneficial effect of metoprolol? E. Raunatin
D. Antipyretic A. Block of M-cholinoceptors
E. Ulcerogenic. B. Absence of action upon β2- adrenoceptors* Myocardial infarction patient was treated with
C. Stimulating of M-cholinoceptors streptokinase. What is the mechanism of
After a tooth extraction a patient felt persistent pain D. Inhibition of m-cholinireceptors thrombolytic action of this drug?
behind his breast bone. After sublingual intake of E. Absence of action on α- adrenoceptors A. Formation of active complex with
an antianginal drug the pain behind the breast bone proactivator of plasminogen*
disappeared, but the patient complained of A patient with myocardial infarction was treated B. Direct activation of plasminogen
headache and dizziness. What drug are these with intravenous infusion of heparin. What is the C. Prevention of prothrombin conversion to
properties typical for? goal of this drug administration? thrombin
A. Validol A. To cause the lysis of thrombus directly D. Prevention of fibrin formation
B. Verapamil B. To transform plasminogen into plasmin E. Direct lysis of thrombus
C. Nitroglycerin* C. To prevent further thrombosis*
D. Metoprolol D. To prevent platelets activation 72-year-old patient with angina pectoris was
E. Propranolol. E. To decrease the area of necrosis prescribed dipyridamole. Administration of drug
resulted in worsening of patient condition and
In complex treatment of a patient with angina It is necessary to prevent the onset of myocardial doctor substituted dipyridamole by long acting
pectoris non- narcotic analgesic exerting infarction in patient with angina pectoris. Choose organic nitrate. What was the most probable cause
antiplatelet activity was included. Point out this the drug with antiplatelet and vasodilating effects. of worsening?
agent. A. Ticlopidine A. Lack of coronary vessels dilation
A. Meperidine B. Dipiridamole* B. Increase in oxygen demand
B. Morphine C. Aspirin C. “Coronary steal phenomenon”*
C. Tramadol D. Nifedipine D. Decrease in arterial pressure
D. Metamizole E. Verapamil E. Decrease in venous pressure
E. Aspirin*
Patient complains of weakness, dyspnea, low
extremities oedema. Diagnosis: chronic cardiac
A patient who had myocardial infarction was inhibition. Blood has increased concentration of second day stenocardia attacks resumed. What can
prescribed 75 mg of acetylsalicylic acid a day. methemoglobin. The drug the patient had taken explain inefficiency of the prescribed drug?
What is the purpose of this administration? relates to the following group: A. Tachyphylaxis*
A. Reduction of platelet (thrombocyte) A. Organic nitrates* B. Cumulation
aggregation * B. Alpha-adrenoceptor blockers C. Sensibilization
B. Inflammation reduction C. Calcium channel blockers D. Idiosyncrasy
C. Pain relief D. Adenosine drugs E. Dependence
D. Temperature reduction E. Myotropic spamolytics
E. Coronary vessel dilatation DRUGS AFFECTING RESPIRATORY
A patient suffers from stenocardia and takes SYSTEM (31)
Anapriline (Propranolol) therapy caused positive isosorbide mononitrate. He was prescribed a
effect in the dynamic of the disease of a 44-year- complementary drug with disaggregating effect. Glaucine
old woman suffering from stenocardia (angina What drug is it? A child suffers from dry cough. What
pectoris). What is the main mechanism of the A. Acetylsalicylic acid* non-narcotic antitussive drug will relieve the
effect of this medicine? B. Nitroglycerine patient’s condition?
A. Blockade of β-adrenoreceptors and C. Propranolol A. Glaucine hydrochloride
decrease myocardial requirements to the oxygen.* D. Nifedipine B. Codeine phosphate
B. Decrease of oxidative exchange in E. Validol C. Morphine hydrochloride
myocardium due to enzyme blockade of Krebs' D. Potassium iodide
cycle A patient with coronary artery disease was E. Althaea officinalis root extract
C. Decreased power inputs of myocardium due admitted to the cardiological department. For
to reduced loading stenocardia (angina pectoris) prevention a drug A patient has acute laryngotrachei-tis with
D. Increased oxygen supply to the from the group of beta-adrenoceptor blockers was nonproductive cough that is very exhaustive.
myocardium administered. What drug is it? Prescribe an antitussive drug:
E. Decreased need in increasing of oxygen A. Metoprolol* A. Glaucine
supply to the myocardium B. Atropine sulfate B. Ambroxol
C. Morphine hydrochloride C. Mucaltin
A patient suffering from coronary artery disease D. Oxytocin D. Herba Thermopsidis
had taken a certain drug many times a day in order E. Furosemide E. Acetylcystein
to arrest stenocardia attacks. Overdose of this drug
finally caused intoxication. Objectively: cyanotic A patient with frequent attacks of stenocardia was Acetylcysteine
skin and mucous membranes, dramatic fall in the prescribed sustak-forte to be taken one tablet twice A patient with chronic bronchitis has been
arterial pressure, tachycardia, respiration a day. At first the effect was positive but on the administered an expectorant that disintegrates
disulphide bonds of sputum glycosaminoglycan mucolytic agent that stimulates surfactant mucolytic drug was prescribed. This drug was
thus reducing its viscosity. The patient has been synthesis. What mucolytic agent was prescribed? especially beneficial for the child due to its ability
also warned about possible bronchospasm. What A. Ambroxolum to stimulate production of surfactant. Point out this
drug has been administered? B. Sodium hydrocarbonate drug
A. Acetylcysteine C. Morphine hydrochloride A. Codeine
B. Libxine D. Glaucin B. Ambroxolum*
C. Bromhexine E. Theophylline C. Oxeladine
D. Thermopsis herb D. Ethylmorphine
E. Sodium hydrocarbonate Analeptic remedy of reflective type from the N- E. Potassium iodide.
cholinomemetics group was given to the patient for
A patient suffering from chronic bronchitis takes a restoration of breathing after poisoning with carbon Severe attack of bronchospasm and cough occurred
synthetic mucolytic drug that stimulates sputum monoxide. What medicine was prescribed to the in a patient with bronchial asthma. Choose the best
liquefaction. What drug is it? patient? drug for quick relief of the patient condition.
A. Acetylcysteine A. Adrenalin hydrochloride A. Ipratropium bromide
B. Diazepam B. Lobeline hydrochloride* B. Orciprenaline sulfate
C. Heparin C. Mesaton C. Atropine
D. Furosemide D. Pentamin D. Adrenaline*
E. Enalapril E. Atropine sulfate. E. Noradrenaline.

A patient suffering fromacute bronchitis with In complex therapy of bronchial asthma myotropic A patient with bronchial asthma had been taking
difficult expectoration was prescribed spasmolytic drug was added. Soon after its usage, tablets which caused insomnia, headache,
acetylcysteine. What drug action will provide nervousness and sleep disturbances occurred. increased blood pressure. What medecine can
curative effect? Which of the following drugs can cause above- cause such complications?
A. Mucoproteins depolymerization mentioned side effect? A. Ephedrine*
B. Activation of bronchial ciliated epithelium A. Papaverine B. Chromolin sodium
C. Reflex stimulation of bronchiolar peristalsis B. Platyphilline C. Euphyline
D. Alkalinization of sputum C. Aminophylline * D. Adrenaline
E. Stimulation of the bronchial glands D. Drotaverine E. Izadrine.
E. Ephedrine .
Ambroxolum A 13-year-old girl with history of asthma
A female patient suffering from acute bronchitis A 5 year-old –child was suffering from bronchitis, complained of cough, dyspnea and wheezing. Her
complains about respiratory obstruction and cough accompanied with a productive cough with thick symptoms became so severe that her parents
with thick viscous sputum. She was prescribed a sputum. In addition to antimicrobial treatment a brought her to the emergency room. Physical
examination revealed diaphoresis, dyspnea, Severe attack of bronchospasm and cough occur in A. Acetylcysteine*
tachycardia and tachypnea. Her respiratory rate patient with bronchial asthma. Choose the best B. Diazepam
was 42/min. pulse rate was 110 beats per minute, drug for quick relieve of patient condition. C. Heparin
and blood pressure was l30/70 mm Hg. Choose A. Ipratropium bromide D. Furosemide
from the following list the most appropriate drug to B. Orciprenaline sulfate E. Enalapril.
reverse the bronchoconstriction rapidly: C. Atropine
A. Methylprednidsolone D. Adrenaline* A patient suffering from bronchial asthma didn't
B. Salbutamol* E. Noradrenaline. inform his doctor that he had attacks of angina
C. Beclomethasone pectoris(stenocardia). Doctor administered him a
D. Cromolyn Patient with bronchial asthma was taking tablets, drug, which usage resulted in less frequent attacks
E. Ipratropium. which caused insomnia, headache, increased blood of bronchial asthma, but stenocardia (anginal)
pressure. What medicine can cause such attacks became more frequent. What medication
Apnoic newborn was resuscitated by complications? was administered?
administration of respiratory stimulant.Which A. Izadrine A. Isadrin (Isoprenalin)*
agent is the drug of first choice in this case? B. Adrenaline B. Salbutamol
A. Bemegridum C. Chromolin sodium C. Aminophylline
B. Aethimisolum* D. Euphyline D. Cromolyn sodium
C. Lobeline E. Ephedrine*. E. Phenotherol.
D. Codeine
E. Camphor. In complex therapy of bronchial asthma myotropic A 70-year-old man, who suffered from chronic
spasmolytic drug was included. Soon after its bronchitis, was prescribed medicine for the cough -
In complex treatment of patient with bronchial usage nervousness and sleep disturbances occurred. codeine. What is the mechanism of antitussive
asthma cromolyn-sodium was included as Which of the following drugs can cause above- effect?
antiallergic agent. What is the mechanism of this mentioned side effect? A. Central*
drug action? A. Papaverine B. Reflex
A. Inhibition of histaminic H1 receptors B. Platyphilline C. Competitive
B. Inhibition of histaminic H2 receptors C. Aminophylline* D. Local effect
C. Prevention of mast cell degranulation* D. Drotaverine E. Peripheral effect.
D. Inhibition of antigen-antibody interaction E. Ephedrine.
E. Blockade of D2 receptors. In long lusting usage of fenoterol in bronchial
A patient suffering from chronic bronchitis takes a asthma its bronchodilating effect is gradually
synthetic mucolytic drug that facilitates the sputum decreasing. What is probable cause of tolerance
thinning. What drug is it? development?
A. Worsening of absorption A. Antidepressants should be administered the patient in the first
B. Decrease in numbers of beta- B. Tranquilizers place?
adrenoceptors* C. Antipsychotics A. Adrenaline*
C. Enhancement of binding with plasma D. Analeptics* B. Acyclovir
albumin E. Analgesics. C. Prednisone
D. Enhancement of elimination D. Famotidine
E. Decrease in numbers of Gs-proteins. Asthmatic patient was assigned to inhalation of E. Fenoterol.
0.5% solution (isoprenalin) isadrinum.
To terminate an attack of asthma patients used Bronchospasm was relieved, but the patient began A 25 y.o. patient in the dental office suddenly
inhalation of the drug that is used by inhalation as to complain of pain in the heart and heartbeat. This developed an attack of asthma. The doctor gave the
bronchodilators, and in injections it is used in is due to the stimulation of: patient salbutamol to inhale. What is the
obstetric practice to prevent miscarriages. What A. Beta2-adrenergic mechanism of action of this drug?
drug was used by patient? B. Alpha1-adrenoceptor A. Stimulates the alpha-adrenergic
A. Fenoterol* C. M-cholinergic B. Blocks histamine H1 receptors
B. Atenolol D. Beta1-adrenergic* C. Blocks M-cholinergic
C. Adrenaline E. Synthesis of acetylcholine. D. Stimulates beta2-adrenergic receptors*
D. Ephedrine E. Blocks phosphodiesterase.
E. Euphylline. Patient suffering from bronchitis with poorly
separated sputum started taking acetylcysteine. For patient suffering of pneumonia and cough with
In patient with bronchial asthma attacks usually Name the mechanism of mucolytic action of the thick poorly separated sputum doctor prescribed
occur at night, they are accompanied by drug. expectorant drug that activates the synthesis of
bradycardia, spasmodic pains in the intestines, A. Reflex stimulates the secretion of bronchial surfactant in the lungs. What the drug was
diarrhea. Preparation of which group can eliminate glands prescribed?
these symptoms? B. Stimulates the cough center A. Acetylcysteine
A. H2-histaminoblockers C. Increases the secretion of bronchial mucus B. Infusion of herb Thermopsis
B. N-cholinoblockers D. Depolymerizes sputum* C. Glaucin
C. M cholinoblockers* E. Increases bronchial contraction. D. Trypsin
D. Alpha-blockers E. Lasolvan (Ambroxol)*.
E. Beta-blockers. A patient with severe attacks of asthma that was
not stopped by earlier effective beta-adrenoceptor To suppress dry cough in patient with chronic
A patient with respiratory depression was admitted agonists was admitted to the intensive care unit bronchitis doctor prescribed antitussive - glaucine
to hospital. Drugs of which pharmacological group Diagnosis: asthma, asthmatic status. What drug hydrochloride. What is the mechanism of action of
stimulate breathing in such case? this drug?
A. Resorptive central* membrane cytoreceptors should be blocked in the B. Sodium sulphate
B. M-cholinergic blockade course of the patient’s complex therapy: C. Castor oil
C. Reflex A. H2-histamine D. Atropine
D. Antienzymatic B. H1-histamine E. Novocaine amide
E. Peripheral. C. a-adrenergic receptor
D. β-adrenergic receptor Contrykal (Aprotinin)
Exhausting cough with fetid odor appeared in E. α2-adrenergic receptor Name the drug that inhibits excretory
patient with lobar pneumonia. Which of the function of pancreas during treatment of
following agents should be used in this case? Famotidine acute pancreatitis:
A. Famotidine A 36- year-old patient has gastric ulcer (with A. Contrykal (Aprotinin)
B. Mesaton increased acidity). Which of the listed below drugs B. Allochol
C. Fenoterol will reduce the secretion of hydrochloric acid with C. Раnzynorm
D. Acyclovir a minimum of side effects? D. Pancreatin (Mezym forte)
E. Trypsin*. A. Famotidine E. Festal
B. Atropine
DRUGS AFFECTING GASTROINTESTINAL C. Pirenzepine A patient with acute pancreatitis presents with
TRACT (38) D. Almagel significantly increased urine diastase content. What
E. – proteolysis inhibitor must be included into complex
Ranitidine therapy of this patient?
A patient with acute duodenal ulcer was admitted To treat peptic ulcer disease of the stomach a A. Contrykal (Aprotinin)
to the hospital. Analysis of gastric juice revealed patient has been prescribed famotidine. Specify the B. Festal
hyperfunction of secretion and acid-forming in mechanism of action of this drug: C. Pancreatine
stomach. Choose a drug that can reduce secretory A. H2 histamine receptors block D. Digestal
function of stomach due to inhibition of H2- B. Effect on ion channels of cell membranes E. Mezym forte
receptors: C. Antienzyme action
A. Ranitidine D. Physicochemical interaction A patient diagnosed with acute pancreatitis was
B. Extract of dry belladonna E. Effect on cell membrane transport system admitted to the surgical department. Which drug
C. Atropine administration would be pathogenetically
D. Methacin Bisacodyl grounded?
E. Platyphyllin An elderly patient has chronic constipations A. Contrykal
induced by large intestine hypotonia. What drug B. Tripsin
A patient has addressed a doctor with complaint of should be administered? C. Chymotripsin
gastric ulcer exacerbation. The following A. Bisacodyl D. Pancreatin
E. Fibrinolysin D. Blockade of H1 histamine receptors
E. Blockade of H2 histamine receptors*. A patient addressed doctor with complaints of
Tripsin heartburn often troubling him. On examination
A patient with skin burns was delivered to a A young woman with breast cancer after surgery ulcerative esophagitis was diagnosed and inhibitor
hospital. To clean the wound from necrotic tissues was treated with a chemotherapeutic complex. At of proton pomp was prescribed. Which of the
and mucus the doctor prescribed an enzymatic drug the very beginning of treatment the patient following drugs was prescribed?
for topical treatment. Name this drug: experienced much distressing nausea and vomiting. A. Cimetidine
A. Tripsin Choose the best agent to relieve patient’s condition B. Ranitidine
B. Pancreatin A. Hyoscine (Aeronum) C. Omeprazole*
C. Asparaginase B. Diphenhydramine (Dimedrolum) D. Misoprostol
D. Pepsin C. Validolum E. Metronidazole.
E. Streptokinase D. Chlorpromazine (Aminazinum)
E. Metoclopramide*. A 37-year-old man was admitted to the surgical
Lydase department with the symptoms of acute
Enzymes and other active substances regulating Patient with gastric peptic ulcer was treated with pancreatitis: vomiting, diarrhea, bradycardia,
connective tissue density and permeability are omeprazole. This agent is capable of decreasing in hypotention, weakness, dehydration of the
being produced in connective tissue cells. What HCl production because it organism. What medicine should be used first of
enzyme drug is used to make the connective tissue A. Neutralizes gastric acid all?
growths looser and more permeable? B. Blocks H2 –histamine receptors A. Ephedrine
A. Lydase C. Inhibits the H+, K+-ATPase proton pump* B. Platyphylline
B. Amylase D. Coats gastric mucosa C. No-spa
C. Lipase E. Blocks muscarinic receptors. D. Etaperazine
D. Cocarboxylase E. Contrycal*.
E. Cholinesterase A 45-year-old woman presents to the emergency
room with severe epigastric pain. On the base of A liquidator of a breakdown at a nuclear power
Patient with gastric peptic ulcer addressed his patient examination and laboratory findings an plant who was irradiated complained about
doctor. After examination doctor prescribed acute pancreatitis is diagnosed. Which of the drugs vomiting that occurs all of a sudden. What
famotidine aiming at inhibition of gastric acid is to be administered in this case? medication should be prescribed?
secretion. What is the mechanism of this drag A. Pancreatin A. Metoclopramide*
action? B. Mezym forte B. Aeron
A. Inhibition of proton pump C. Contrycal* C. Reserpine
B. Neutralization of gastric acid D. Morphine D. De-Nol
C. Prevention of histamine release E. Fentanyl. E. Atropine.
A. Nonsteroidal anti-inflammatory drugs A patient with exacerbation of duodenal peptic
Examination of a 35 year old patient revealed high B. Calcium channel blockers ulcer was admitted to hospital The analysis showed
acidity of gastric juice. What receptors should be C. Steroidal anti-inflammatory drugs an increased gastric secretion and acid-forming
blocked in order to reduce it? D. Blockers H1 histamine receptor function of the stomach. Choose a product that will
A. alpha 1-adrenoreceptors E. H2 histamine receptors lockers*. reduce the secretory function of the stomach by
B. beta2-adrenoreceptors blocking histamine H2-receptor?
C. beta1 -adrenoreceptors For patients with peptic ulcer the drug inhibiting A. Famotidine*
D. alpha2-adrenoreceptors the growth and reproduction of H.pylori was B. Atropine Sulfate
E. Histamine*. prescribed as a causal treatment. Name the drug: C. Plathyphylline
A. Corglyconum D. Dry extract of belladonna
For treatment of biliary tract dyskinesia ( B. Glauvent E. Metacin.
hypokinetic type) doctor prescribed cholagogue, C. Metronidazole*
referring to the group holekinetics. Indicate the D. Furosemide A pregnant woman is suffering from heartburn.
drug E. Prazosin. Which agent is the best?
A. Magnesium sulfate (gavage)* A. Ranitidine
B. Alloholum A 46 years old patient was complained of stomach B. Sodium bicarbonate
C. Cholenzymum pain, heartburn. During examination increased C. Omeprazole
D. Oxafenamid gastric acidity was revealed. What should be D. Almagel*
E. Cholosasum. assigned to neutralize the acidity of gastric juice? E. Bismuth sulfate.
A. Papaverine
Aged patient is suffering from constipation for a B. Atropine Sulfate The doctor prescribed bisacodyl for patient
long time. Which synthetic laxatives should be C. Almagel* suffering from chronic constipation. What
proposed in this case? D. Neostigmine pharmaceutical group is the drug?
A. Atropine E. Benzohexonium. A. Narcotic analgesics
B. Bisacodyl* B. Laxatives*
C. Castor oil Patients with peptic ulcer of the duodenum uses C. Antihypertensives
D. Liquid paraffin famotidine. What is the group of this drug? D. Blockers H2-histamine receptor
E. Magnesium sulfate. A. Beta-agonists E. Diuretics.
B. Alpha-adrenergic agonists
A patient suffering from gastric ulcer with C. Antiholinesterase A 45 –year-old women is suffering from chronic
hyperacidic syndrome was admitted to hospital. D. H2-blockers of histamine receptors* pancreatitis. Doctor prescribed her complex
The drug of which group should be used in the E. Irritant. preparation containing pancreatic enzymes and
treatment of the patient? dried bile. Point out prescribed drug.
A. Allocholum C. Liv-52 proton pump (H +-K +-ATPase) used in the
B. Pancreatin D. Siliborum combined therapy.
C. Aprotinin E. Papaverine. A. Gastrocepin (pirenzepine)
D. Festal* B. Omeprazole*
E. Cholenzymum. 25 year-old man was hospitalized to C. Misoprostol
gastroenterology department with complaints of D. Atropine sulfate
A patient with cholecystitis relieves periodical "hungry" pain, a feeling of heaviness in the E. Almagel.
attacks of pain with myotropic spasmolytic drug. stomach, heartburn. Doctor prescribed gastrocepin
Which of listed agents is used? (pirenzepine). What effect of gastrocepin A patient with chronic hyperacidic gastritis to
A. Drotaverine(No-spa)* (pirenzepine) was the base for choice of a relieve heartburn is using antacid, it gives prompt
B. Ephedrine physician? effect, but it is accompanied by unpleasant feeling
C. Pancreatin A. Relaxation of biliary tract muscle of fullness in the stomach. Which of the drugs can
D. Aprotinin B. Relaxation of smooth muscles of the cause this side effect?
E. Atropine. stomach A. Magnesium trisilicate
C. Reduction of gastric acid secretion and B. Aluminum hydroxide
Patient addressed doctor complaining of gastrin* C. Pepsin
periodically occurring dyspepsia. Laboratory tests D. Destruction of Helycobacter pylori D. Magnesium oxide
did not reveal pathogens that can be responsible for E. Reduction of trypsin secretion . E. Sodium bicarbonate*.
diarrhoea. Choose the drug the most beneficial in
this case. Usage of sodium bicarbonate in peptic ulcer with A complex treatment of gastric peptic ulcers
A. Atropine high acidity gives temporary relief. What is the includes famotidine. What is the mechanism of this
B. Platyphylline cause of short lasting effect of drug in this case? drug action?
C. Papaverine A. The formation of carbon dioxide A. M-cholinergic blockade
D. Bisacodyl stimulating a secondary hypersecretion* B. The blockade of proton pump (H +, K +-
E. Loperamide*. B. Neutralization of hydrochloric acid ATPase)
C. Stimulation of histamine H2-receptors C. Blockade of H2 histamine receptors*
Patient with chronic hepatitis was prescribed D. Stimulation of the gastric M-cholinergic D. Antacid effect
complex preparation with hepatoprotecting receptors E. Local anesthetic effect on the mucous
properties. Preparation contains essential E. Accumulation of histamine. membrane of the stomach..
phospholipids and vitamins. Point out this
preparation The patient has hyperacidic gastritis. Specify the A patient has complex treatment of gastric peptic
A. Legalon drug with the mechanism realized by blocking of ulcers; concomitant disease is glaucoma. Drugs of
B. Essentiale*
which group in this connection should not be topically. What drug can be applied for this A. Methotrexate
included in antiulcer therapy? purpose? B. Prednisolone
A. Myotropic antispasmodics A. Heparin ointment C. Pentoxyluv*
B. Blockers H2-histamine receptor B. Salicylic ointment D. Mercaptopurine
C. Antacid C. Troxevasin ointment E. Cyanocobalamin.
D. Cholinoblockers* D. Butadion ointment
E. Gastroprotectors. E. Thrombin Severe anemia occurred in patients after gastric
resection on account of ulceration. What medicine
Almagel was prescribed for patient with gastric A 46-year-old female patient needs a surgery in the has to be assigned along with improvement of
ulcer. What is the mechanism of drug action? maxillofacial region. It is known that the patient is nutrition?
A. The neutralization of hydrochloric acid* disposed to increased hemocoagulation. What A. Ferroceronum
B. Local anesthetic action natural anticoagulant can be used in order to B. Cyanocobalamin*
C. The blockade of the H2 histamine receptor prevent thrombosis? C. Methyluracilum
D. M-cholinergic blockade A. Heparin D. Iron sulfate
E. Adsorption of alkaline foods. B. Hirudin E. Coamidum.
C. Sodium citrate
DRUGS AFFECTING BLOOD SYSTEM (34) D. Fibrinolysin Blood test revealed anemia in the patient with
E. None of the listed drugs megaloblastic forms of red cells, due to this
Acetylsalicilic acid cyanocobalamin was assigned for treatment. At
A patient suffering from stenocardia takes 100 mg A patient with myocardium infarction was what type of anemia this vitamin preparation is
of acetylsalicilic acid daily. What is the effect of admitted to the resuscitation department. What indicated?
acetylsalicilic acid in this patient? drug should be injected to the patient in order to A. Hyperchromic anemia*
A. Inhibition of thrombocyte aggregation prevent thrombosis? B. Hypoplastic
B. Inhibition of blood coagulation A. Heparin C. Hemolytic
C. Dilatation of coronary vessels B. Chingamin D. Aplastic anemia
D. Prothrombin rate reduction C. Thyroxine E. Hypochromic anemia.
E. Cholesterol rate reduction D. Biseptol-480
E. Dimedrol Hypochromic anemia appeared in patient due to
Heparin internal bleeding. Which of these drugs can
A patient complains about shin pain which is The patient was using levomycetin for a long time provide rapid recovery of red blood cells?
getting worse during walking. Objectively: there is without control, his examination revealed a A. Cobamamid
an edema and reddening along the vein. A doctor decrease in leukocyte count in the blood. Which B. Heparin
administered a direct coagulant to be applied drug should be used for stimulation leukopoiesis? C. Sirepar
D. Furosemide D. Sodium oxalate
E. Ferum-Lek*. Patients suffering from angina and receiving E. Fibrinogen*.
isosorbide mononitrate, was appointed an
After radiation therapy the myelo depressive additional drug with antiplatelet effect. Identify the Anticoagulant therapy with heparin was performed
reaction in the form of leukopenia appeared in drug: in patient with myocardial infarction. What
patient. Which of the following drugs should be A. Validolum preparation should be used to continue treatment
applied to stimulate leukopoiesis? B. Nitroglycerine after withdrawal of heparin to prevent recurrence
A. Vitogepat C. Nifedipine of thrombosis?
B. Ferropleks D. Aspirin* A. Neodikumarinum*
C. Hemostimulin E. Inderal. B. Calcium chloride
D. Sodium nucleinat* C. Furosemide
E. Nicotinic acid. The child emerged bleeding from the nose. Which D. Fibrinolysin
of the following drugs can stop it? E. Sodium hydrocitrate.
In the patient that had long history of stomach A. Thrombin*
disease hyperchromic anemia was found. Which of B. Fibrinolysin Analysis of the patient's blood showed an increased
the following agents is to be prescribed for this C. Vicasolum (Menadione) number of red blood cells and hemoglobin that is
disease? D. Potassium chloride characteristic of polycythemia. Which of these
A. Ascorbic acid E. Ethamzylatum. drugs should be prescribed for a patient?
B. Cyanocobalamin* A. Kobamamid
C. Unithiolum Massive bleeding began in woman after separation B. Contrycal
D. Ferum-Lek of the placenta. Which of the drug you need to help C. Vitogepatum
E. Oxyferriscorbone sodium. in this case? D. Imiphosum*
A. Thrombin E. Pentoxylum.
Iron deficiency anemia is very common among B. Fibrinolysin
pregnant women, complicating pregnancy, C. Aminocapronic acid* To stop heavy uterine bleeding ergot alkaloid that
childbirth, adversely affects the fetus and newborn. D. Fepromaronum increases myometrium tone was selected. Which of
Propose for the treatment such anemia preparation E. Vicasolum (Menadion). listed agents is of this group?
of ferrous iron for oral administration. A. Oxytocin
A. Ferkoven Gastric bleeding appeared in the patient. Which of B. Ergometrine*
B. Ferrum-lek these drugs can stop it? C. Neostigmin
C. Hemostimulin A. Folic acid D. Aminocapronic acid
D. Koamidum B. Thrombin E. Fenoterol.
E. Iron ferrous lactate*. C. Sodium citrate
Direct acting anticoagulant was prescribed for C. Inhibition of the synthesis of prothrombin B. Vicasolum*
patients prone to thrombosis. Point out the drug: in the liver C. Calcium chloride
A. Aminocaproic acid D. Reduction of the number of calcium ions in D. Fibrinogen
B. Vicasolum (Menadione) the blood E. Aminocapronic acid.
C. Neodikumarin E. Influence on the retraction of the clot.
D. Sinkumar For the prevention of myocardial infarction doctor
E. Heparin*. In the laboratory experimental studies in vitro and prescribed nonsteroidal anti inflammatory drug
in vivo antagonist of direct anticoagulant was with antiplatelet effect - aspirin. Choose the dose
Before tooth extraction a patient was advised to selected for the clinical use. What could be the and regimen at which the drug will exert
take a certain drug for hemorrhage prevention. preparation of these? antiplatelet effect.
What drug was advised? A. Protamine sulfate* A. 0.1 three times a day
A. Heparin B. Sodium citrate B. 0.25 three times a day
B. Vicasolum (Menadione)* C. Contrycal C. 1.0 three or four times a day
C. Asparcam D. Menadione D. 0.5 three times a day
D. Magnesium sulfate E. Calcium chloride. E. 0.1-0.2 once a day for a long time*.
E. Dimedrolum.
Massive uterine bleeding in woman was stopped For the treatment of hypochromic anemia patient
A patient with essential hypertension was by introducing aminocaproic acid. Which of listed takes iron ferrous sulfate. Specify one of the side
recommended a drug that prevents thrombosis. mechanisms contributed to hemostatic effect of this effects of this drug.
This drug exerts antiplatelet and coronary dilating drug? A. The darkening of the teeth*
effects. What drug is it? A. Inhibition of fibrinolysis* B. Dyspepsia
A. Heparin B. Activation of fibrinolysis C. Allergy
B. Dipiridamole* C. Increased prothrombin in the blood D. Itching
C. Protamine sulfate D. Increasing calcium in the blood E. Dizziness.
D. Neodicumarin E. Increased synthesis of prothrombin in the
E. Syncumar. liver. Vicasolum (Menadione) has been prescribed for
patient a week before tonsillectomy to prevent
In acute thrombosis intravenous heparin rlieves the 8 year-old child is preparing for tonsillectomy. bleeding. Specify the mechanism of action of this
patient's condition. Which of these mechanisms is Blood analysis showed clotting time increased to 7 drug.
responsible for effect of heparin? minutes. What medication should be included in a A. Inhibits fibrinolysis
A. Dissolution of fresh thrombus complex of drugs for preparation period of 5 days B. Activates fibrinolysis
B. Inhibition of the transformation of before surgery in the first place? C. Increases the synthesis of clotting factors in
prothrombin to thrombin in the blood* A. Etamsylate the liver*
D. Inhibits ACE A patient with a history of angina pectoris and high Choose the hormonal drug for enhancement of
E. Causes vasoconstriction. plasma cholesterol level was prescribed uterine contraction during labor
phenofibrate. What is the mechanism of action of A. Dinoprost (PGF2 )
Indirect acting anticoagulant warfarin was this drug? B. Oxytocin*
prescribed for patient to prevent thrombosis. What A. Activation of lipoprotein lipase* C. Ergometrine
is the mechanism of action of this drug?. B. Inhibition of lipoprotein lipase D. Salbutamol
A. Inhibition of fibrinolysis C. Inhibition of hydroxymethyl glutaryl E. Partusisten
B. Activation of fibrinolysis coenzyme A
C. Decrease in the synthesis of clotting factors D. Inhibition of LDL oxidation To stop postpartum bleeding doctor prescribed
in the liver* E. Bile acid sequestration. ergot alkaloid derivative. Point out this drug
D. Inhibition of ACE A. Oxytocin
E. Inhibition of platelets aggregation. Patient suffering from pernicious anemia is treated B. Methylergometrine*
with folic acid. Which agent is to be added to make C. Neostigmin
On discontinuation of heparin treatment of patient effective the treatment of this disease? D. Dinoprostone (PGE2)
with myocardial infarction administration of A. Retinoli acetas E. Qinine
warfarin was started. This drug exerts its B. Cyancobalamin*
anticoagulant effect by C. Thiamine bromide Pregnant woman developed rhythmic uterine
A. Blocking calcium binding to clotting factors D. Retabolil contractions at the physiological term of delivery.
B. Forming a complex with clotting factors E. Insulin. But contractions were not progressing in force and
C. Breaking down thrombin frequency. Cervix of uterus was neither softening
D. Inhibiting of pro-clotting factor synthesis in A patient ill with essential hypertension was nor opening. Chose the drug, which is the most
the liver* recommended a drug that prevents thrombosis. It is beneficial in this case.
E. Depolymerization of fibrin. to be taken parenterally. What drug is it? A. Oxytocin
A. Protamine sulfate B. Pituitrin
A patient with myocardial infarction was treated B. Amben C. Ergometrine
with intravenous infusion of heparin. What is the C. Heparin* D. Dinoprostone*
goal of this drug administration? D. Neodicumarin E. Partusisten
A. To cause the lysis of thrombus directly E. Syncumar.
B. To transform plasminogen into plasmin A pregnant woman developed rhythmic uterine
C. To prevent further thrombosis* DRUGS ACTING ON MYOMETRIUM. contractions at the physiological term of delivery.
D. To prevent platelets activation ANTIGOUT DRUGS. (5) But contractions were not progressing in force and
E. To decrease the area of necrosis. frequency. Choose the drug for stimulation of
uterine contraction.
A. Oxytocin* B. Insulin Sulfocamphoric acid)
B. Ergometrine C. Prednisolone D. Cerigel
C. Partusisten D. Sibazon E. Chlorhexidine (bi)gluconate
D. Physostigmine E. Heparin
E. Castor oil. A patient suffering form stomatitis was prescribed
Chloramine oral rinsing. Which antiseptic from the oxidant
Intake of oral contraceptives containing sex Having completed work in a laboratory, a student group is the most suitable for this purpose?
hormones inhibits secretion of the hypophysiae must tidy up the workspace, perform disinfection A. Potassium permanganate
hormones. Secretion of which of the indicated of the workbench and tools. What chemicals B. Boric acid
hormones is inhibited while taking oral should be used for disinfection? C. Alcoholic iodine solution
contraceptives with sex hormones? A. Chloramine D. Ethyl alcohol
A. Somatotropic B. Ether E. Chloramine
B. Oxytocin C. Formalin
C. Vasopressin D. Hydrochloric acid A patient was admitted to a hospital with poisoning
D. Follicle-stimulating* E. Chloroform with unsound food. His stomach was lavaged with
E. Thyrotropic. solution of potassium permanganate. What is its
Hydrogen peroxide mechanism of action?
ANTISEPTICS AND DISINFECTANTS (32) A doctor has made a diagnosis of gingivitis and A. Release of atomic oxygen
recommended the patient to rinse the oral cavity B. Release of chlorine
Alcohol solution of iodine with an oxidizing agent. Specify this agent: C. Release of iodine
Name the halogen-containing antiseptic A. Hydrogen peroxide D. Disturbance of synthesis of respiratory
with fungicidal properties, which is used to treat B. Boric acid chain enzymes
dermatomycosis: C. Salicylic acid E. Destruction of bacteria membranes
A. Iodine solution D. Phenol
B. Formalin solution E. Brilliant green Formaldehyde
C. Methylene blue For the purpose of disinfection of nonmetallic
D. Brilliant green Potassium permanganate surgical instruments the formaldehyde solution was
E. Boric acid solution A patient is diagnosed with acute morphine used. What group does this anticeptic preparation
hydrochloride intoxication. Prescribe the oxidizing belong to according to its chemical structure?
A 46-year-old patient consulted a doctor about agent for gastric lavage: 1. Aliphatics
pustular rash on the skin of the limbs. What A. Potassium permanganate 2. Aromatics
antiseptic should be administered to the patient? B. Chloramine 3. Alcohols
A. Alcohol solution of iodine C. Sulfocamphocainum (Procaine + 4. Halogenated compounds
5. Detergents Patient with abscess of the cut wound applied to D. Aggressive action of hydrogen peroxide on
the traumatological department. Doctor for the tissues
For the preparation of a patient`s burn skin surface cleaning of the wound from the pus washed it with E. Formation of albuminates.
a certain medication was used. Its antiseptic action 3% hydrogen peroxide. Foam was absence. What
is provided by free oxygen that segregates in is the cause of absence of the drug activity? Patient with burns was treated by application of
presence of organic substances. Choose the right A. Pus in the wound antiseptic containing halogen and polyvinyl
answer: B. Low concentration of H2O2 pirrolidon. Point out this preparation.
A. Potassium permanganate* C. Inherited insufficiency A. Iodovidon*
B. Chlorhexidine phosphatdehydrogenase of erythrocyte B. Halozan
C. Sodium bicarbonate D. Shallow wound C. Chlorhexidine
D. Boric acid E. Inherited insufficiency of catalase*. D. Chloramine
E. Furacilin. E. Pantocide.
A group of students were traveling by countryside
Patient suffering from tonsillitis was recommended collecting medicinal plants. It was a hot day and The patient was admitted to the infectious disease
to gargle throat with preparation of nitrofurans. the students spent the entire drinking water. The clinic with complaints of vomiting, diarrhea, fever,
What drug was recommended? team leader offered the water of the pond, using the after errors in diet. What antiseptic should be used
A. A solution of hydrogen peroxide, chlorine containing tablet for its disinfection. Point for gastric lavage (washing out)?
B. Solution of furacilinum,* the drug. A. Ethanol
C. Solution of silver nitrate, A. Chloramine-B B. Hydrogen peroxide
D. Chlorhexidine B. Chlorhexidine C. Methylene blue
E. Alcoholic solution of iodine. C. Halozan* D. Activated char coal
D. Brilliant Green E. Potassium permanganate*.
Burned skin surface was treated with antimicrobial E. Furacilinum.
preparation. Its antiseptic properties are provided A patient has purulent wound with necrotic
by atomic oxygen that is formed in presence of A nurse washed the patient’s contaminated wound content. What medication should be used to
organic substances. What preparation was applied? with 3% hydrogen peroxide solution that causes cleanse the wound?
A. Potassium permanganate* formation a lot of foam. What is the mechanism of A. Hydrogen peroxide*
B. Furacillin antiseptic effect of this drug? B. Furacilinum (Nitrofurazone)
C. Chlorhexidine bigluconate A. The formation of molecular oxygen* C. Sulfacyl sodium
D. Alcoholic iodine solution B. The formation of atomic form of oxygen D. Alcohol solution of iodine
E. Boric acid . C. The interaction of hydrogen peroxide with E. Aetacridini lactas.
fibrinolysine
In the patient with syphilis treated by, bismuth To prepare the operative field surgeon used a C. Furacilinum*
preparations gray patches on the mucous solution of iodine in alcohol. What is the chemical D. Boric acid
membrane of the mouth and symptoms of kidney group of this antiseptic preparation? E. Aethacridini lactates.
disturbances appeared. Which agent is advisable A. Heavy metals
for the treatment of bismuth poisoning? B. Halogen-containing compounds* Teenager addressed dermatologist complaining of
A. Naloxone C. Alcohols acne. Doctor prescribed him an antiseptic agent
B. Unitiol* D. Detergents that gradually releases oxygen and is traditionally
C. Methylene blue E. Aliphatic substance. used locally for treatment of acne. Point out
D. Nalorphine prescribed drug.
E. Bemegride. For disinfecting of metallic instruments in the A. Hydrogen peroxide
surgical department the formaldehyde solution is B. Potassium permanganate
Dentist handled herpetic lesions of 7 years old used. What is the chemical group of this antiseptic C. Benzoyl peroxide*
child with halogen antiseptic that has preparation? D. Boric acid
antimicrobial, antifungal and antiviral properties. A. Halogenated compounds E. Methylenum coeruleum.
Name the drug. B. Aromatic substance
A. Alcohol solution of iodine* C. Detergents To prevent pyodermia in 4-year-old girl with
B. Potassium permanganate D. Aliphatic agent* chickenpox mother was recommended to apply a
C. Formaldehyde E. Alcohols. preparation of a group of dyes. Point out this
D. Silver nitrate preparation.
E. Brilliant Green. After extirpation of the tooth in the patient A. Chlorhexidine
bleeding from the hole emerged. What drug from B. Alcoholic solution of iodine
The patient with gingivitis after using applications the group of antiseptics should be used in this C. Furacilinum
was assigned to rinse oral cavity with antiseptic case? D. Alcoholic solution of of Brilliant Green*
agent realizing its effect by atomic oxygen drug A. A solution of hydrogen peroxide* E. Chloramine.
exerts deodorant and astringent effects. Identify the B. Solution of epinephrine hydrochloride
drug. C. Solution of brilliant green Before the operation surgeon used degmicidum for
A. Ethanol D. Iodinol disinfecting his hands. Point the group of this
B. Chlorhexidine bigluconate E. A solution of ethyl alcohol. antimicrobial agent
C. Hydrogen peroxide A. Group of detergent*
D. Sodium bicarbonate Nitrofurane antiseptic was prescribed for mouth B. Group of dyes
E. Potassium permanganate* wash to patient with stomatitis. Point out this agent C. Halogen containing antiseptic
F. Hydrogen peroxide. A. Hexamethylen tetraminum D. Oxidizing agent
B. Aethonium E. Group of nitrofuranes .
B. Boric acid D. Streptomycin sulfate
Doctor prescribed oral rinse for patient with C. Alcoholic solution of iodine E. Chloramphenicol
stomatitis. Which antiseptic of oxidazinggroup is D. Chlorhexidine*
most suitable for this? E. Ethanol. A 29-year-old patient with bacterial pneumonia has
A. Boric acid been administered penicillin. What is the
B. Potassium permanganate* Surgeon was urgently summoned to the operating mechanism of its antimicrobial action?
C. Alcoholic solution of iodine room. Choose the antiseptic used for surgeon hands A. Inhibition of cell walls synthesis in the
D. Chloramine in emergency. microorganisms
E. Ethanol. A. Boric acid B. Inhibition of intracellular protein
B. Ethyl alcohol 96%* synthesis
Gastric lavage with a solution of potassium C. Ethacridine lactate C. Inhibition of cholinesterase activity
permanganate was performed in patient due to food D. Hydrogen peroxide D. Inhibition of SH-groups of
poisoning. Which group of antiseptics does this E. Ethyl alcohol 70%. microorganism enzymes
drug belong to? E. Antagonism of PABA
A. Aromatic substances ANTIBIOTICS (49)
B. Halogens Amoxicillin
C. Acid Benzylpenicillin A patient has been diagnosed with bacillary
D. Oxidizing agents* A patient suffering from syphilis was prescribed a dysentery. What drug of those listed below should
E. Detergents. drug the action of which based upon disturbed be prescribed?
generation of murein leading to death of the A. Amoxicillin
Solution of carbolic acid was used for disinfecting causative agent. What drug is it? B. Benzylpenicillin sodium salt
of instruments. To which group does this antiseptic A. Benzylpenicillin sodium salt C. Isonicotinic acid hydrazide (Isoniazid)
belong? B. Bijochinol D. Itraconazole
A. Oxidizing agents C. Ciprofloxacin E. Acyclovir
B. Phenols* D. Azithromycin
C. Halogens E. Doxycycline hydrochloride Oxacillin
D. Acid A 43-year-old patient is to be administered an
E. Detergents. A patient with streptococcal infection of gums was antibiotic from the penicillin group which would
prescribed a drug that contained beta-lactam ring in be resistant to penicillinase. What drug can be
For disinfecting of hand before operation surgeon its structure. Which drug relates to this group? recommended?
used chlorine containing antiseptic, that also A. Benzylpenicillin A. Oxacillin
widely used in dentistry. Point out this agent. B. Rifampicin B. Amoxicillin
A. Potassium permanganate C. Erythromycin C. Carbenicillin
D. Azlocillin B. Tetracyclines C. Take before eating
E. Ampicillin C. Sulfanilamides D. Course of treatment should not exceed 1
D. Chloramphenicol group day
Carbenicillin disodium E. Macrolides E. Do not take with vitamin preparations
A 60-year-old patient was admitted to the surgical
department because of infection caused by blue pus Doxycycline For the treatment of periostitis a 35-year-old
bacillus (Pseudomonas aeruginosa) which is A 26-year-old female patient with bronchitis has patient should be administered an antibiotic with a
sensative to penicillin antibiotics. Indicate which of been administered a broad spectrum antibiotic as a high ability to penetrate into the bone tissue.
the given penicillins has marked activity to the causal treatment drug. Specify this drug: Specify this drug:
Pseudomonas aeruginosa? A. Doxycycline A. Doxycycline hydrochloride
A Carbenicillin disodium B. Interferon B. Streptomycin
B Benzylpenicillin C. BCG vaccine C. Erythromycin
C Phenoxymethylpenicillin D. Ambroxol D. Chloramphenicol
D Oxacillin E. Dexamethasone E. Kanamycin
E Methicillin
During examination of first-grade pupils a dentist A patient with mandibular osteomyelitis has been
Cephalosporin antibiotic revealed that one child had brown-yellow teeth and administered an antibiotic from the tetracycline
For infection prevention a patient who underwent two split teeth. According to the child, previously group. Specify this drug:
appendectomy was prescribed a cephalosporin he had been treated for pneumonia with "some" A. Doxycycline hydrochloride
antibiotic. Antimicrobial activity of these pills. What drug might have had such a negative B. Rifampicin
antibiotics is called forth by the disturbance of the impact on teeth? C. Streptomycin
following process: A. Doxycycline D. Oxacillin
A. Microbial wall formation B. Oxacillin E. Amikacin
B. Nucleic acid synthesis C. Erythromycin
C. Ribosomal protein synthesis D. Ampicillin A stomatologists examined first-grade pupils and
D. Energy metabolism E. Biseptol revealed that one of children had yellowish brown
E. Cholinesterase block teeth, two of them were split. Heretofore the pupil
A patient with acne is prescribed doxycycline was treated with "some pills"on account of
A patient suffers from a severe life-threatening hydrochloride. What should the patient be warned pneumonia. What medication could have had such
generalised septic infection. What group of against, regarding administration of this drug? a negative effect upon teeth?
chemotherapeutical drugs should be prescribed in A. Avoid prolonged exposure to the sun 1. Doxycycline
this case? B. Take with large amount of liquid, 2. Oxacillin
A. Cephalosporines preferably milk 3. Erythromycin
4. Ampicillin B. Erythromycin the given penicillins has marked activity to the
5. Biseptol C. Benzylpenicillin sodium salt Pseudomonas aeruginosa?
D. Isoniazid A. Methicillin
Erythromycin E. Ciprofloxacin B. Phenoxymethylpenicillin
An infectious patient manifests sensibilization to C. Carbenicillin disodium*
penicillin. Which of the following antibiotics is the Gentamicin D. Oxacillin
safest to be applied in this case? Pathogenic staphylococcus was obtained from the E. Benzylpeniciilin.
A. Erythromycin purulent wound of the patient. Its antibiotic
B. Bicillin sensitivity was determined to be as follows: A 50-year-old patient with typhoid fever was
C. Ampicillin penicillin growth inhibition zone - 8 mm; oxacillin treated with Levomycetin, next day his condition
D. Amoxicillin - 9 mm, ampicillin - 10 mm, gentamicin - 22 mm, became worse, temperature rose to 39,6°C. What
E. Oxacillin lincomycin - 11 mm. What antibiotic should be caused the complication?
chosen for treatment in this case? A. The effect of endotoxin agent*
Azithromycin A. Gentamicin B. Reinfection
5-year-old child has been diagnosed B. Oxacillin C. Secondary infection addition
with acute right distal pneumonia. Sputum C. Ampicillin D. Irresponsiveness of an agent to the
inoculation revealed that the causative D. Penicillin levomycetin
agent is resistant to penicillin and senstive E. Lincomycin E. Allergic reaction.
to macrolides. What drug should be prescribed?
A. Azithromycin A patient suffers from severe postoperative
B. Tetracycline Lincomycin hydrochloride pseudomonadous infection. What of the following
C. Gentamycin To treat osteomyelitis, a patient was prescribed an antibiotics should be administered in this case?
D. Streptomycin antibiotic that easily penetrates into bone tissue. A. Erythromycin
E. Ampicillin Name this drug: B. Doxycycline
A. Lincomycin hydrochloride C. Cephazolin
B. Streptomycin sulfate D. Benzylpenicillin
A 30-year-old patient with pneumonia has been C. Cefazolin E. Amicacin sulfate*.
administered a 3-day course of an antibiotic from D. Polymyxin B
the group of azalides that has bactericidal effect, E. Amphotericin B Car driver has got into accident and was admited to
prolonged action, the ability to bind to phagocytic emergancy department. Surgical invasion in
cells and accumulate in the infection foci. What A 60-year-old patient was admitted to the surgical abdominal cavity was successfuly performed, but
drug has been administered? department because of infection caused by blue pus postoperative period was complicated by
A. Azithromycin bacillus (Pseudomonas aeruginosa) which is peritonitis. Bacterial analysis revealed
sensitive to penicillin antibiotics. Indicate which of
Pseudomonas aeruginosa. For treatment of patient E. Streptomycin. D. Furazolidone
you may propose any antibiotic leasted below with E. Ampicillin.
axception Patient with tuberculosis was on the complex
A. Azlocillin treatment including antibiotics. At the end of Patient with pneumonia, prone to use alcohol, was
B. Genamycine first course doctor noticed significant decrease in treated with antibiotic of wide spectrum. On the
C. Cefotaxim patient hearing. Which of the following drugs can third day of treatment after alcohol usage severe
D. Imipenem (Tienam) cause such side effect? antabus-like reaction occurs. What was the group
E. Tetracycline*. A. Isoniazid of antibiotic used for treatment of patient?
B. Rifampicin A. Biosynthetic penicillin
A 6-year boy was admitted to hospital with C. Streptomycin* B. Aminoglycoside
pneumonia. Treatment with amoxycycline was not D. Paraaminosalicylic acid C. 3rd generation cephalosporin*
effective. Bacterial analysis revealed Micoplasma E. Pirazinamide . D. Penicilinse resistant penicillin
pneumoniae. Choose the most suitable drug for E. Tetracycline.
treatment of this child. Patient, with severe pneumonia was treated with
A. Tetracycline the III generation cephalosporin. Doctor warned Patient condition after long term atimicrobial
B. Azithromycine* him about the danger of alcohol usage while treatment was complicated by pseudomembranous
C. Bicillin 5 treatment. What the drug was used for treatment of enterocolitis. For it treatment vancomycin was
D. Nystatin patient? prescribed. What is the mechanism of its
E. Oxacillin. A. Carbenicilin antimicrobial effect?
B. Cefotaxim* A. Disturbances of structure and function of
A 20 years old man with gonorrhea was treated C. Cefalexin cell membrane
with penicillin G in combination with probenecid. D. Gentamycin B. Inhibition of cell wall synthesis*
His state was improved, but 1 week later the E. Doxycycline. C. Inhibition of protein synthesis
patient was still complaining of a persistent D. Inhibition of nucleic acid synthesis
urethral discharge and pain on urination. Patient was admitted to the infection unit with E. Inhibition of folic acid metabolism.
Laboratory test reveals chlamidia in discharge. diagnosis of bacterial dysentery. On laboratory
Which of the listed drugs effective both in studies it was revealed that causative element is Patient was admitted to the infection unit with
gonorrhea and chlamidial infection must be used sensitive to the many antimicrobial medicines, but diagnosis of bacterial dysentery. On laboratory
for further treatment patient has anemia. What medicine is contra- studies it was revealed that causative element is
A. Amoxicillin indicated to the patient? sensitive to the many antimicrobial medicines, but
B. Doxycycline* A. Levomycetin* patient has anemia. What antibiotic can be
C. Oxacillincin B. Phthalazol recommended for this patient?
D. Gentamycin C. Enteroseptol A. Levomycetin
B. Amoxicillin* B. Penicillins
C. Erythromycin C. Tetracyclines Patient has primary syphilis. What is the most
D. Cotrimoxazol D. Macrolides effective antibiotic in this case?
E. Oxacillin. E. Aminoglycosides*. A. Penicillin*
B. Nystatin
A 5-year-old child has been diagnosed with acute A patient with bacterial pneumonia was prescribed C. Amphotericin
right distal pneumonia. Sputum inoculation benzylpenicillin. What is the mechanism of its D. Streptomycin
revealed that the causative agent is resistant to antibacterial effect? E. Kanamycin.
penicillin, but it is senstive to macrolides. What A. Abnormal permeability of cytoplasmic
drug should be prescribed? membrane Why tetracycline is not recommended in pregnant
A. Azithromycin* B. Inhibition of intracellular protein synthesis women?
B. Tetracycline C. Inhibition of synthesis of microbial cell A. Due to ototoxicity
C. Gentamycin wall* B. Due to the depressing effect on the
D. Streptomycin D. Inhibition of SH-groups of microorganism respiration of the fruit
56. E. Ampicillin . enzymes C. Due to the ability of the drug to increase
E. Antagonism with p-amino-benzoic acid . uterine tone
A 19 year old woman suffers from primary D. Due to the teratogenic effect of the drug*
syphilis. Doctor administered her complex therapy A patient with gastric peptic ulcer was on complex E. Due to the ability to cause anemia in
that includes benzylpenicillin sodium salt. What is treatment that includes antibiotic of macrolide pregnant.
the mechanism of action of this drug? group. Point out this antibiotic.
A. It blocks RNA synthesis A. Clarithromycin* In the patient, who has long used tetracycline, there
B. It blocks synthesis of cytoplasm proteins B. Tetracycline was mucosal candidiasis. What medication should
C. It blocks thiol enzymes C. Gentamycin be appointed to treat it?
D. It blocks synthesis of peptidoglycan of D. Streptomycin A. Nystatin*
microbal wall* E. Ampicillin . B. Nizoral
E. It blocks DNA synthesis . C. Griseofulvin
Antibiotic used for the treatment of typhoid fever D. Furadonin
A patient underwent appendectomy. In the is: E. Nitrofungin.
postoperative period he has been taking an A. Oxacillin
antibiotic. The patient complains about hearing B. Erythromycin Mother of 2 years old child addressed dentist with
impairment and vestibular disorders. What group C. Cephalexin tooth abnormality in her child. On child
of antibiotics has such side-effects? D. Benzylpenicillin examination the destruction of the incisors, yellow
A. Cephalosporins E. Levomicetin*. enamel, the brown rim on the necks of the teeth
ware revealed. Mother informed doctor that she A 6 year child was admitted to the hospital with A. Polymyxin
had infection and used antibiotics. Which of the diagnosis of candidiasis caused by Candida B. Lincomycin
drugs has a strong teratogenic effect, disrupting the albicans, the mucous membrane of cheeks, palate C. Tetracycline*
development of teeth? and tongue was cowered by fur of white and D. Oxacillin
A. Polymyxin yellow color. Which of the drugs can be used to E. Penicillin G.
B. Erythromycin treat the child?
C. Tetracycline* A. Gentamicin Patient with staphylococcal sepsis was treated with
D. Cefrenol B. Ketoconazole* benzylpenicillin. Therapy was ineffective. What
E. Ampiox. C. Tetracycline preparation of penicillin group should be
D. Cefran prescribed for the patient to continue treatment?
The patient of 42 years old for the treatment of E. Penicillin. A. Oxacillin*
bacterial pneumonia was appointed ampicillin. B. Polymyxin
Specify the mechanism of bactericidal action of the A patient with impaired hearing has severe C. Erythromycin
drug bacillary infection. Which group of antibiotics is D. Phtalazol
A. Inhibition of intracellular protein synthesis contraindicated in this case? E. Aztreonam.
B. Inhibition of the synthesis of the cell wall of A. Aminoglycosides*
the microorganism* B. Penicillins Child of 5 years with a staphylococcus infection
C. Violation of the permeability of the C. Cephalosporins was assigned penicillin antibiotic without testing
cytoplasmic membrane D. Tetracyclines pathogen susceptibility to drugs of this group.
D. Inhibition of SH - groups of enzymes of E. Rifamycins. Which drug of penicillin group is resistant to beta-
microorganisms lactamase and effectively influences on the
E. Antagonism with paraaminobenzoic acid. A patient diagnosed with purulent pleurisy caused penicillin-resistant staphylococci?
by penicillin resistant staphylococcus. What A. Carbenicillin
Infectious patient is sensitized to benzylpenicillin. preparation of penicillin resistant to beta-lactamase B. Ampicillin
Which of the following antibiotics would be the may be used for the treatment of this patient? C. Bicillin-5
safest in this case? A. Ampicillin D. Phenoxymethylpenicillin
A. Amoxicillin B. Benzylpenicillin E. Oxacillin*.
B. Ampicillin C. Phenoxymethylpenicillin
C. Erythromycin* D. Augmentin* SYNTHETIC CHEMOTHERAPEUTIC
D. Oxacillin E. Carbenicillin. DRUGS INCLUDING
E. Bicillin 1. FLUOROQUINOLONES (24)
Use of which broad-spectrum antibiotic is
contraindicated in liver disease? Sulphanilamides
Certain infections caused by bacteria are treated A 7 vear old child is ill with bronchitis. It is
with sulphanilamides that block the synthesis of Co-trimoxazole necessary to administer him an antibacterial drug.
bacterial growth factor. What is the mechanism of After the second abortion a 23-year-old woman has What drug of fluoroquinolone group is CONTRA-
these drugs action? been diagnosed with toxoplasmosis. Which drug INDICATED at this age?
A. They are antivitamins of p-aminobenzoic should be used for toxoplasmosis treatment? A. Cyprofloxacin*
acid A. Co-trimoxazole B. Sulfadimethoxine
B. They inhibit the folic acid absorption B. Itraconazole C. Ampiox
C. They are allosteric enzyme inhibitors C. Mebendazole D. Ampicillin
D. They are involved in redox processes D. Azidothimidine E. Amoxicillin.
E. They are allosteric enzymes E. Acyclovir
Patient with pneumonia has intolerance to
A 24-year-old patient with catarrhal tonsillitis has Ciprofloxacin antibiotics. Which of the combined sulfanilamide
been administered a drug from the group of A patient has been diagnosed with sepsis. It was medicines should be prescribed to the patient?
sulfonamides. Specify the mechanism of decided to treat him with a drug from the A. Sulfacyl sodium
sulfonamide antibacterial action: fluoroquinolone group. Specify this drug: B. Aethazol
A. Competitive antagonism of PABA A. Ciprofloxacin C. Biseptol*
B. Disruption of the cell wall protein synthesis B. Cefpirome D. Streptocid
C. Reduction of membrane permeability C. Metronidazole E. Sulfadimethoxine.
D. Inhibition of sulfhydryl groups of thiol D. Ampicillin
enzymes E. Cephalexin A patient consulted a stomatologist about purulent
E. Protein coagulation inflammation of his gums. What drug will be the
Nitrofurans most effective if it is suspected that a causative
Sulfanilamides are applied as antimicrobal agents A patient undergoing treatment with nitrofurans agent is an anaerobe?
in clinical practice. Sulfanilamide treatment, has imbibed a small amount of alcohol, which A. Co-trimoxazole
however, can result in hemolytic anemia resulted in severe poisoning. The poisoning B. Gentamicin
development in patients that suffer from genetic developed due to: C. Oxacillin sodium
defect of the following enzyme of pentose A. Accumulation of acetic aldehyde D. Metronidazole*
phosphate metabolism in erythrocytes: B. Allergic reaction E. Nitroxoline .
A. Glucose-6-phosphate dehydrogenase C. Neural disorder
B. Pyruvate kinase D. Cardiovascular collapse A patient with pneumonia had a complex treatment
C. Hexokinase E. Disturbed renal function that included sulfonamide preparation. What
D. Transketolase should doctor recommend to prevent cristaluria?
E. Transaldolase A. Decrease the dose of sulfonamide
B. Drinking of alkaline solution* B. Furazolidonum
C. Administration of drugs before meal C. Emetine hydrochloride A Bactrim was prescribed for 35-years old patient
D. Drinking of acidic solution D. Metronidazole* with bronchitis. What is the mechanism of action
E. Use of vitamins. E. Levomycetin. of this drug?
A. Sequential block of two stages of bacterial
Gonorrhoea was revealed in the patient on What is the mechanism of antimicrobial effect of folate metabolism*
bacterioscopy of the smear from urethra. Taking sulfonamide? B. Inhibition of bacterial cell wall synthesis
into account that medicines for gonorrhea are A. Inhibition of protein synthesis C. Inhibition of protein synthesis
fluorquinolones, patient should be prescribed: B. Inhibition of bacterial cell wall synthesis D. Inhibition of cell membrane synthesis
A. Ciprofloxacin* C. Competitive antagonism with para- E. Inhibition of nucleic acid synthesis.
B. Furazolidone aminobensoic acid*
C. Fluorouracil D. Inhibition of cell membrane synthesis For treatment of typhoid fever ciprofloxacin was
D. Urosulfan E. Inhibition of nucleic acid synthesis. prescribed. What is mechanism of this drug action?
E. Cefazoline. A. Inhibition of bacterial cell wall synthesis
Combined sulfonamide preparation with B. Inhibition of DNA gyrase*
Patient with pneumonia has intolerance to bactericidal effect was prescribed for treatment of C. Inhibition of protein synthesis
antibiotics. Which of the combined sulfanilamide sore throat in 13 years old child. Point out this D. Competitive antagonism with
medicine should be prescribed to the patient? drug. paraaminobenzoic acid
A. Trimethoprim A. Biseptol* E. Inhibition of nucleic acid synthesis.
B. Aethazol B. Eneroceptol
C. Natrium sulfacyl C. Sulfalen Due to which side effect ciprofloxacin is
D. Streptocid D. Aethasolum contraindicated in children?
E. Biseptol*. E. Urosulfanum. A. Hepatotoxicity
B. Glucosuria
A 30-year-old patient complains about having A HIV patient has bacterial dysentery. On the C. Cartilage damage*
abdominal pain and diarrhea for 5 days body result of antimicrobial sensitivity test doctor D. Nausea
temperature rise up to 37,5o C along with chills. prescribed him cotrimoxazol (bactrim,biseptol). E. Insomnia.
The day before patient had been in forest and What type of antibacterial action has this drug?
drunk from open water reservoir. Laboratory A. Virostatic A doctor prescribed sulfonamide drug to patient
analysis enabled to make the diagnosis of amebic B. Bacteriostatic with acute bronchitis. After urinalyses he changed
dysentery. What is the drug of choice for its C. Fungicidal it for antibiotic of penicillin group. The possibility
treatment? D. Fungistatic of which side effect of sulfonamide was the base
A. Phthalazol E. Bactericidal*. for changing it by other drug?
A. Mental confusion E. It interfere with sulfonamide distribution. B. Streptomycin
B. Hyperglycemia C. Ampicillin
C. Allergic reaction Synthetic antimicrobial drug of quinolones with D. Phthalylsulfathiazole
D. Crystalurea* moderate activity against mycobacteria E. Trimethoprim/sulfamethoxazole (Biseptol)
E. Glucosurea . tuberculosis was recently included in second-line
antitubercular drugs. Point this drug. A female who had been continuously taking
Urinalyses revealed crystaluria in patient who used A. Furozalidone antibiotics for an intestinal infection developed a
synthetic antimicrobial drug for treatment of B. Sulfadimetoxine complication manifested by inflammation of the
bronchitis. Which group of antimicrobial drug can C. Biseptol oral mucosa and white deposit. Bacteriological
cause such side effect? D. Isoniazid study of the deposit samples revealed yeast fungi
A. Sulfonamides* E. Ciprofloxacin*. Candida albicans. Which of the following
B. Fluorochinolons medications is indicated for the treatment of this
C. Tetracyclines ANTIFUNGAL, ANTITUBERCULOTIC, complication?
D. Penicillins ANTIVIRAL, AND ANTISPIROCHETAL A. Fluconazole
E. Macrolydes. DRUGS (33) B. Biseptol
C. Tetracycline
Sulfonamides are the drugs of broad antimicrobial Nystatin D. Furazolidone
spectrum that includes some protozoa. For A patient developed burning sensation in the oral E. Polymyxin
complex treatment of which protozoal infection is cavity and white fuzzy coating on the tongue. Oral
it used? thrush is diagnosed. What drug of those listed Itraconazole
A. Tuberculosis below should be used? A patient who has been taking tetracycline for a
B. Malaria* A. Nystatin long time has developed candidosis of mucous
C. Amebiasis B. Amphotericin membranes. What drug shoul administered for its
D. Otitis C. Griseofulvin treatment?
E. Bronchitis. D. Tetracycline A. Itraconazole
E. Gentamicin B. Griseofulvin
Why procaine is not compatible with C. Nitrofungin
sulfonamides? Fluconazole D. Amphotericin
A. It binds with sulfonamide An 18-year-old patient has developed E. Nitrofurantoin
B. It forms paraaminobenzoic acid when candidiasis after the case of pneumonia
metabolized.* treated with β-lactam antibiotic. What antimycotic Acyclovir
C. It inhibits bacterial cell wall synthesis agent should be prescribed? A patient has herpetic conjunctivitis. What
D. It interferes with sulfonamide absorption A. Fluconazole etiotropic drug should be administered?
A. Acyclovir A. Isoniazid B. Benzylpenicillin sodium salt
B. Ampicillin B. Rifampicin C. Benzylpenicillin potassium salt
C. Methisazonum C. Ciprofloxacin D. Biseptol-480
D. Furagin D. Sodium salt of benzylpenicillin E. Cefazolin
E. Tetracycline E. Alcohol iodine solution
To treat tuberculosis, an antibiotic that colors urine
A patient consulted a dentis about a lesion of his A patient suffering form tuberculosis was treated red is prescribed. Name this antibiotic:
oral mucosa. He was diagnosed with herpetic with rifampicin, which caused drug resistance of A. Rifampicin
stomatitis. Which of the following drugs will have tuberculosis mycobacteria. In order to reduce B. Erythromycin
an effect on etiotropic factor? mycobacteria resistance, rifampicin should be C. Amoxicillin
A. Acyclovir combined with the following drug: D. Nitroxoline
B. Dimedrol A. Isoniazid E. Cefotaxime
C. Paracetamol B. Acyclovir
D. Levamisole C. Intraconazole Streptomycin
E. Furacilinum D. Metronidazole A patient being treated for tuberculosis is suffering
E. Amoxicillin from hearing deterioration. What drug causes this
A patient with herpetic stomatitis was prescribed complication?
acyclovir for topical application. What is its Following treatment with a highly-efficient anti- A. Streptomycin
mechanism of action? tuberculosis drug a 48-year-old female developed B. Isonicotinic acid hydrazide (Isoniazid)
A. It inhibits synthesis of nucleic acids of optic nerve neuritis, memory impairment, cramps. C. Kanamycin sulphate
viruses Which of these anti-TB drugs had the patient D. Ethionamide
B. It inhibits virus penetration into cells taken? E. Rifampicin
C. It inhibits virus maturation A. Isoniazid
D. It increases the resistance of B. PASA A 35-year-old man under the treatment for
macroorganism cells to the viruses C. Rifampicin pulmonary tuberculosis has acute-onset of right big
E. It inhibits virion assembly D. Ethambutol toe pain, swelling, and low-grade fever. The gouty
E. Kanamycin sulfate arthritis was diagnosed and high serum uric acid
Isoniazid level was found. Which of the following
After 4 months of treatment for tuberculosis the Rifampicin antituberculosis drugs are known for causing high
patient began complaining of toes and fingers After starting treatment for pulmonary tuberculosis uric acid levels?
numbness, sensation of creeps. He was diagnosed a patient complained about red tears and urine. A. Rifampicin
with polyneuritis. What antituberculous drug might What drug could cause such changes? B. Aminosalicylic acid
have caused these complications? A. Rifampicin C. Thiacetazone
D. Pyrazinamide* Patient with tuberculosis was on the complex A patient has herpetic rash. What medication
E. Cycloserine. treatment including antibiotics. At the end of first should be administered?
course doctor noticed significant decrease in A. Acyclovir*
A patient was diagnosed with active focal patient hearing. Which of the following drugs can B. Gentamycin
pulmonary tuberculosis. What drug should be cause such side effect? C. Clotrimazole
prescribed in the first place? A. Isoniazid D. Benzylpenicillin sodium salt
A. Ethionamide B. Rifampicin E. Biseptol.
B. Isoniazid* C. Streptomycin*
C. Ethoxide D. Paraaminosalicylic acid Point out the antiviral agent used in influenza.
D. Sulfalen E. Pirazinamide . A. Oseltamivir.*
E. Cyclocerine. B. Streptomycine
One of the problems in the treatment of C. Gentamycin
A patient suffers from pulmonary tuberculosis. tuberculosis is caused by existence of D. Clotrimazole
During treatment neuritis of visual nerve occurred. metabolically dormant forms of mycobacterium E. Pyridoxine.
What drug has caused this side effect? (persister). Which of the drugs is effective against
A. Isoniazid all subpopulations of Mycobacteria tuberculosis A patient with lung tuberculosis is treated with a
B. Ethambutol* and causes sterilizing effect? drug that has a wide antimicrobial spectrum;
C. Kanamycin A. Streptomycin mycobacteria of tuberculosis are highly sensitive to
D. Rifampicin B. Isoniazid it. Its effect is realized by depression of bacterial
E. Streptomycin . C. Kanamycin RNA synthesis. Drug is staining body liquids
D. Rifampicin* (urine, sputum, tears) in red color. What the drug is
Patient suffering from tuberculosis was treated E. Paraaminosalicylic acid. used?
with isoniazid. In the course of treatment A. Streptomycin
peripheral neuritis was developed. What is the The patient suffering from tuberculosis was treated B. Isoniazid
mechanism of this side effect? with combination of antituberculous drugs, C. Ethambutol
A. Direct toxic effect of drug on peripheral including isoniazid. Which drug should be added to D. Rifampicin*
nerves prevent neurological side effects of isoniazid? E. Paraaminosalicylic acid.
B. Inhibition of myoneural junctions A. Streptomycin
C. Interference with pyridoxine metabolism* B. PASA Treatment of tuberculosis is realized by means of
D. Interference with folic acid synthesis C. Rifampicin combination chemotherapy, including agents with
E. Inhibition of sodium channels. D. Pyridoxine* different mechanisms of action. Which of the
E. Ascorbic acid. antituberculosis drugs inhibit the transcription of
DNA into RNA of mycobacteria?
A. Rifampicin* who confirmed gynecomastia. Identify the drug, to be integrated instead of deoxyguanosine into
B. Isoniazid which he used. viral DNA. Name the drug.
C. Streptomycin A. Cycloserine A. Interferon
D. Ethionamide B. Ethambutol B. Azidothymidine
E. PAS. C. Isoniazid* C. Acyclovir*
D. Rifampicin D. Midantan
The patient went to a doctor complaining of red E. Streptomycin sulfate . E. Ethambutol.
staining of urine and tear. From his case history it
is known that he is treated on pulmonary As a result of prolonged use of broad spectrum A 20 years old patient with primary syphilis is
tuberculosis. Which of anti-TB drugs is antibiotics intestinal candidamycosis was receiving combined therapy, which includes the
responsible for this phenomenon? developed. What should be prescribed to treat it? sodium salt of benzylpenicillin. What is the
A. Isoniazid A. Interferon mechanism of action of this drug?
B. Ethionamide B. Remantadin A. Blockade of DNA synthesis
C. Ethambutol C. Itraconazole* B. RNA synthesis blockade
D. Rifampicin* D. Rifampicin C. Blockade of the SH- groups of enzymes
E. Streptomycin . E. Gramicidin. D. Blockade of the synthesis murein of cell
wall of microorganisms*
How do you explain the fact that the in treatment A patient with a diagnosis of AIDS was admitted E. Blockade of protein synthesis in cytoplasm.
of tuberculosis with isoniazid dose is adjusted to the infectious disease hospital. To improve the
individually, with mandatory measurement of its immune status of patient, stabilizing body weight A patient addressed doctor with complaints on the
concentration in the urine after the first drug? and improwement of his condition, antiviral agent - damage of skin between the fingers of foot,
A. Genetically determined differences in rate an antagonist of thymidine, which blocks the DNA exudation and itching. Doctor prescribed paste
of drug acetylating* polymerase of HIV was prescribed. Select the containing antifungal agent of wide spectrum.
B. Occurrence of hyperglycemia as a side designated agent. Point out this drug
effect A. Azidothymidine* A. Nystatin
C. The development of renal failure B. Acyclovir B. Clotrimazole*
D. Irritant action of the drug C. Interferon C. Streptomycin
E. Development of hemolytic anemia. D. Rimantadine D. Co-trimoxazol
E. Midantan. E. Levorin.
TB patient after prolonged treatment with effective
anti-TB drug called attention to breast Patients with herpes doctor prescribed an antiviral This drug is used for treatment of tuberculosis, rate
enlargement. On this occasion he went to a doctor, drug, which the main mechanism of action is of its inactivation in the liver is genetically
inhibition of viral DNA polymerase, and the ability
determined and is different in different persons. E Emetine hydrochloride patient was diagnosed with trichomonal
Point out this drug gingivostomatitis. What drug should be chosen for
A. Acyclovir In order to prevent wound infection associated with his treatment?
B. Azidothymidine surgical procedures a patient was given a synthetic A. Metronidazole
C. Ciprofloxacin antiprotozoan drug with a high activity against B. Ampicillin
D. Isoniazid* Helicobacter pylori. Specify this drug: C. Doxycycline hydrochloride
E. Rifampicin. A. Metronidazole D. Gentamicin sulfate
B. Doxycycline hydrochloride E. Nystatin
ANTHELMINTIC AND ANTIPROTOZOAL C. Chingamin Quinine
DRUGS (26) D. Acyclovir It was necessary to determine absolute gustation
E. Isoniazid thresholds of a healthy man for different
Metronidazole substances. The lowest threshold will be observed
Systemic amebiasis with involvment of intestines, To prevent wound infection associated with for the following substance:
liver, lungs was diagnosed in a 52-year-old surgical procedures a patient was given a synthetic A. Quinine
patient. What drug should be prescribed? antiprotozoal drug demonstrating high activity B. Sodium chloride
A Metronidazole against Helicobacter pylori. Specify this drug: C. Glucose
B Quiniofone A. Metronidazole D. Saccharose
C Tetracycline B. Doxycycline hydrochloride E. Citric acid
D Quingamine C. Chingamin
E Enteroseptol D. Acyclovir To prevent of 4-day malaria a 42-year-old patient
E. Isoniazid was prescribed primaquine. On the 3-rd day from
A 30-year-old patient complains about having the begin of treatment there appeared stomach and
abdominal pain and diarrhea for five days; body A dentist has detected symptoms of parodontosis in heart pains, dyspepsia, general cyanosis,
temperature rise up to 37,5oC along with chills. a patient. What antiprotozoal drug should be hemoglobinuria. What is the cause of side effects
The day before a patient had been in a forest and prescribed? of the preparation?
drunk from an open water reservoir. Laboratory A. Metronidazole A. Drug potentiation by other preparations
analyses enabled to make the following diagnosis: B. Levamisole B. Cumulation of the preparation
amebic dysentery. What is the drug of choice for C. Griseofulvin C. Delayed urinary excretion of the
its treatment? D. Mykoseptin preparation
A Metronidazole E. Furazolidone D. Decreased activity of microsomal liver
B Furazolidonum enzymes
C Levomycetin A patient consulted a dentist about itching and E. Genetic insufficiency of glucose 6-
D Phthalazol burning in the oral cavity; high temperature. The phosphate dehydrogenase*.
drug is contra-indicated because the drug inhibits B. Chloroquine
A 52-year-old patient has the following diagnosis: metabolism of ethyl alcohol. What drug is it? C. Pyrimethamine
systemic amebiasis with involvement of intestines, A. Clonidine D. Metronidazole*
liver, lungs. What drug should be prescribed? B. Metronidazole* E. Monomecinum.
A. Enteroseptol C. Reserpine
B. Tetracycline D. Diazepam The patient has a mixed helminthic invasion:
C. Quiniofone E. Aminazine. intestinal ascariasis and liver trematodes. What
D. Quingamine anthelminthic drugis the most advisable for the
E. Metronidasol*. Before the doctor’s business trip to foreign country treatment of this patient?
he was proposed histoschizontocidal antimalarial A. Mebendazole*
A patient consulted a doctor about bowels drug for the personal prevention of malaria. Which B. Pyrantel B.
disfunction. The doctor established symptoms of drug has got a specialist? C. Chloxylum
duodenitis and enteritis. Laboratory examination A. Mefloquine D. Piperazine adipate
helped to make the following diagnosis: lambliosis. B. Chloroquine E. Levamisole.
What medication should be administered? C. Quinine
A. Tetracycline D. Doxycycline The drug has a devastating effect on the
B. Monomycin E. Primaquine*. erythrocytic forms of malaria parasites, dysenteric
C. Metronidazole* amoeba. It is used to treat and prevent malaria, the
D. Erythromycin The patient with lovered immunity was examined treatment of amoebiasis and connective tissue
E. Chingamin. concerning helminthiasis. Laboratory studies have diseases. Identify the drug.
found ascariasis. Which drug should be prescribed? A. Emetine hydrochloride
A patient consulted a stomatologist about purulent A. Niclosamide B. Chloroquine*
inflammation of his gums. What drug will be the B. Piperazine C. Tetracycline
most effective if it is suspected that a causative C. Furazolidone D. Erythromycin
agent is an anaerobe? D. Gentamicin E. Quinine.
A. Oxacillin sodium E. Levamisole*.
B. Gentamicin The patient addressed doctor complaining of
C. Nitroxoline The patient addresed her gynecologist complaining epigastric discomfort, nausea, loss of appetite. The
D. Co-trimoxazole of heavy vaginal discharge with an unpleasant study of duodenal content revealed Giardia
E. Metronidazole*. odor. After the smears bacterioscopy resistant to metronidazole. Which drug should be
trichomoniasis was diagnosed. What the drug is appointed?
A patient ill with amebiasis was prescribed a indicated in this case? A. Rifampicin
certain drug. The use of alcohol together with this A. Sulfadimezin B. Chloroquine*
C. Metronidazole E. Nystatin. The drug with histoschizontocydal and
D. Isoniazid gametocydal action was used for causal
E. Acyclovir . For treatment of neck phlegmon caused by prophylaxis of malaria. Point out this drug.
anaerobic infection antimicrobial drug of imidasole A. Metronidasole
Antiprotozoal agent of wide spectrum was included group was included. Point out this drug B. Primaquine*
in complex treatment of patient with gastric peptic A. Norfloxacin C. Chloroquine
ulcer. Doctor warned patient about prohibition of B. Clindamycin D. Tetracycline
alcohol usage during treatment with this drug. C. Metronidazole* E. Emetine hydrochloride .
What the drug was used? D. Tetracycline
A. Chloramphenicol E. Ampicillin. Point out the antimalarial drug that exert
B. Metronidazole* erythrocytic schizontocidal effect by changing pH
C. Cotrimoxazole A 30-year-old patient complains about having in parasitic vesicles and prevention of haeme
D. Chloroquine abdominal pain and diarrhea for five days body transformation to hemozoin.
E. Praziquantel. temperature rise up to 37,5oC along with chills. A. Primaquine
The day before a patient had been in a forest and B. Chloroquine*
Patient suffering from severe malaria caused by drunk from an open water reservoir. Laboratory C. Metronidazole
plasmodium falciparum resistant to chloroquine analyses enabled to make the following diagnosis: D. Sulfadoxine
was treated with the oldest antimalarial drug, amebic dysentery. What is the drug of choice for E. Pyrimethamine.
obtained from the bark of cinchona tree. Point out its treatment?
this drug. A. Metronidazole* ANTITUMOR DRUGS (21)
A. Primaquine B. Furazolidonum
B. Proguanil C. Levomycetin Methotrexate
C. Pyrimethamine D. Phthalazol Leukoses are treated with anti- metabolite
D. Quinine* E. Emetine hydrochloride . methotrexate. What vitamin is its antagonist?
E. Emetine. A. Folic acid
Amebiasis of hepatic localization was diagnosed in B. Cyanocobalamin
Point out the drug of first choice for treatment of patient. Choose the drug for treatment of patient. C. Phyllochinone
patient suffering from kala-azar (visceral A. Norfloxacin D. Piridoxine
leishmaniasis) B. Clindamycin E. Rutin
A. Metronidazole C. Chloroquine*
B. Sodium stibogluconate* D. Tetracycline Continuous treatment of cancer patients with
C. Tetracycline E. Ampicillin. methotrexate over time reduces the target cell’s
D. Pyrimethamine
sensitivity to the drug. In this case gene A. Methyluracil D. Competitive inhibition of nucleic acid
amplification of the following enzyme is observed: B. Retinol acetate* metabolism in tumors
A. Dihydrofolate reductase C. Levamisole E. Cytorecptors blockade on the cell
B. Thiaminase D. Prednisolone membrane.
C. Deaminase E. Retabolil.
D. Thioredoxin reductase Thioredoxin In the intensive care unit a child was enrolled with
reductase An oncological patient was prescribed pronounced signs of acidosis, to relieve his
E. - methotrexate. With the lapse of time target cells of condition immediate drip of infusion was started.
the tumour lost susceptibility to this drug. There is Which of the following drugs should be used in
An oncological patient had been administered change of gene expression of the folowing enzyme: acidosis?
methotrexate. With time target cells of the tumour A. Thiaminase A. Potassium Chloride
lost sensitivity to this drug. At the same time the B. Deaminase B. Sodium Chloride
change in gene expression of the following enzyme C. Dehydrofolate reductase* C. Sodium bicarbonate*
is observed: D. Folate decarboxylase D. Glucose
A. Dehydropholate reductase E. Folate oxidase. E. Magnesium sulfate.
B. Thiaminase
C. Deaminase Patient after surgery for breast cancer Patients with chronic heart failure was using
D. Pholate oxidase chemotherapy with antiestrogen agent was digoxin. To increase the result of treatment
E. Pholate decarboxylase prescribed. Which of the following drugs belongs additional drug was included in patient therapy. It
to antiestrogen with anticancer activity? resulted in the development of symptoms of
A patient who was previously ill with mastectomy A. Tamoxifen* intoxication. Which drug can cause increase in
as a result of breast cancer was prescribed B. Cyclophosphamide cardiac glycoside toxicity?
radiation therapy. What vitamin preparation has C. Chlorbutin A. Potassium chloride
marked radioprotective action caused by D. Cisplatin B. Magnesium chloride
antioxidant activity? E. Fosfestrol. C. Asparkam
A. Ergocalciferol D. Glucose
B. Tocopherol acetate* What is the basic mechanism of anticancer action E. Calcium chloride*.
C. Thiamine chloride of ethyleneimine derivative?
D. Ribof1avin A. Alkylation of RNA and DNA of tumor A patient with acute poisoning was admitted to
E. Folic acid. cells* emergency department. It was established that
B. Inhibition of cell division in metaphase poisonous substance was excreted by kidney.
In order to accelerate healing of a radiation ulcer a C. Formation of stable complexes with DNA Choose the best diuretic for forced diuresis.
vitamin drug was administered. What drug is it? of tumor cells A. Acetazolamide
B. Amiloride A patient with myocardium infarction was treated D. Phtorafur
C. Spironolactone with heparin. On 6th day of treatment nasal E. Cytarabine.
D. Furosemide* bleeding occurred. Overdose of heparin was
E. Hydrochlorothiazide. diagnosed. Point out the antidote of heparin Patient with gastric cancer is treated with drug that
A. Acetylcysteine interferes with synthesis of nucleic acids due to its
Choose the most beneficial agent for stomach wash B. Protamine sulfate* resemblance with pirimidine. Point out this drug
in patient poisoned with alkaloid C. Magnesium sulfate A. Mercaptopurine
A. Physiological solution D. Unithiolum B. Methotrexate
B. Chlorhexidine E. Deferoxamine. C. Phluorouracil (Phtoruracil)*
C. Sodium bicarbonate D. Neomycin
D. Potassium permanganate* Patient suffering from cancer of skin is treated with E. Bleomycin.
E. 5% solution of glucose. anticancer antibiotic. Point out this drug
A. Neomycin Highly dehydrated patient with cholera was
Point out the antidote used in poisoning with salts B. Danazol admitted to specialized department. Which agent is
of heavy metals C. Mercaptopurine necessary to use to normalize the volume of
A. Penicillamine D. Phtoruracil circulating blood?
B. Unithiolum* E. Bleomycin*. A. Solution KCl 4%
C. Acetylcysteine B. Solution of calcium chloride 10%
D. Magnesium sulfate Radio protector was given to a worker of a nuclear C. Solution of glucose 40%
E. Atropine. power station. What mechanism is considered to be D. Solution calcium gluconate
responsible for radioprotection? E. Solution of NaCl 0,9%*.
A patient in comatose state was admitted to A. Increasing of respiration
emergency department. His respiration was B. Inhibiton of free radicals formation* A patient working at a chemical plant was admitted
abnormal (Cheyn-Stoke’s respiration). Poisoning C. Activation of oxidation reactions to the toxicological department with mercury
with opioid analgesic was diagnosed. After D. Prevention of tissue's hypoxia poisoning. What medicine should be used?
intravenous administration of antidote respiration E. Increasing of tissue blood supply. A. Isonitrozin
was restored. What agent was used as antidote? B. Naloxone
A. Pralidoxime Patient suffering from acute leucosis was treated C. Enterosorbent
B. Naloxone* with agent that is competitive antagonist of folic D. Unithiol*
C. Ethanol acid. Point out this agent E. Activated carbon.
D. Acetazolamide A. Phtoruracil
E. Unithiolum. B. Methotrexate* PRINCIPLES OF ACUTE POISONING
C. Mercaptopurine TREATMENT (6)
E. Isonitrosine E. Fibrinogen
A 17-year-old girl has attempted to commit suicide
by overdosing on phenobarbital. Upon arrival at A patient is diagnosed with acute morphine MSQs 1st term – 386
the site an emergency doctor urgently performed hydrochloride poisoning. Choose the oxidant drug
gastric lavage on the patient and introduced to be prescribed for gastric lavage. MSQs 2nd term – 408
bemegride and solution of sodium hydrocarbonate A. Potassium permanganate Total – 794
intravenously. The doctor introduced sodium B. Chloramine
hydrocarbonate to: C. Sulfocamphocainum (Procaine +
A. Increase renal excretion of phenobarbital Sulfocamphoric acid)
B. Normalise blood pressure D. Cerigel
C. Stimulate respiration E. Chlorhexidine digluconate
D. Inactivate phenobarbital
E. Wake up the patient A woman poisoned with unknown substance was
hospitalised in a toxicological
A 50-year-old patient with hypertensive crisis had department. What group of drugs can be
been administered magnesium sulfate, which led to administered to decrease absorption and
abrupt decrease in blood pressure. The side effects introduction of the poison to her body?
of magnesium sulfate can be prevented if the A. Adsorbents
following drug is administered: B. Neuroleptics
A. Calcium chloride C. Antioxidants
B. Trilon B D. Organic nitrates
C. Sodium sulfate E. Cholinesterase inhibitors
D. Sodium bromide
E. Potassium chloride A patient with acute myocardial infarction
has been administered heparin as a part of
A 38-year-old man who poisoned himself with complex therapy. Some time after heparin injection
mercury dichloride was taken to the admission the patient developed hematuria. What heparin
room in grave condition. What antidote should be antagonist should be injected
immediately introduced? to remove the complication?
A. Unithiol A. Protamine sulfate
B. Dipiroxim B. Vicasol
C. Atropine C. Aminocaproic acid
D. Nalorphine D. Neodicumarin

You might also like